Dr. Basil Notes- Nac Osce Vip

March 27, 2018 | Author: amroamasha | Category: Gastroesophageal Reflux Disease, Anemia, Headache, Myocardial Infarction, Thorax


Comments



Description

OSCE guideThird edition Table of contents Table of contents ............................................................................................................................. 2 History taking – Medicine ............................................................................................................... 6 General review:............................................................................................................................ 7 Notes .......................................................................................................................................... 12 Chest pain – ACUTE ................................................................................................................. 14 Chest pain – CHRONIC ............................................................................................................ 16 Headache.................................................................................................................................... 18 Blood results / Macrocytic Anemia / B12 Deficiency................................................................. 21 Difficulty swallowing ................................................................................................................ 23 Elevated liver enzymes .............................................................................................................. 24 Drinking / Alcohol ..................................................................................................................... 26 Fever / Tired .............................................................................................................................. 28 Diarrhea – ACUTE .................................................................................................................... 29 Diarrhea – CHRONIC ............................................................................................................... 30 ASTHMA .................................................................................................................................. 32 COPD management ................................................................................................................... 35 Ankle swelling – Bilateral ......................................................................................................... 36 Ankle swelling – Unilateral ....................................................................................................... 37 Congestive heart failure – CHF ................................................................................................. 38 Heart racing ............................................................................................................................... 40 Fall ............................................................................................................................................. 42 Peripheral vascular disease: ....................................................................................................... 44 Urinary symptoms: .................................................................................................................... 45 Anuria ........................................................................................................................................ 46 Hematuria .................................................................................................................................. 47 Renal stones ............................................................................................................................... 48 Incontinence............................................................................................................................... 50 Lump – Neck Swelling .............................................................................................................. 51 Lump – Breast............................................................................................................................ 52 Dizziness.................................................................................................................................... 55 INR – Counselling ..................................................................................................................... 57 Patient is receiving blood – counsel for adverse reactions ........................................................ 59 Counselling – Ventilator............................................................................................................ 61 Ethical questions ........................................................................................................................ 62 HIV post-test counselling .......................................................................................................... 63 Lung Nodule .............................................................................................................................. 64 High Creatinine.......................................................................................................................... 66 Impotence / Erectile Dysfunction .............................................................................................. 67 Rheumatology – History Taking................................................................................................ 68 Multiple Sclerosis ...................................................................................................................... 69 Obesity....................................................................................................................................... 70 Epilepsy Counselling ................................................................................................................. 71 Medical note .............................................................................................................................. 73 Pre-diabetes – Counselling ........................................................................................................ 74 Emergency Medicine..................................................................................................................... 75 Emergency Room ...................................................................................................................... 76 Trauma ....................................................................................................................................... 77 OSCE-guide-III.doc Page 2 of 255 Unconscious Patient – Neuro..................................................................................................... 81 Unconscious Patient – Diabetic ................................................................................................. 82 Unconscious Seizing Patient – DT / Epilepsy / Brain tumour / …............................................ 84 Heart Attack – Chest Pain (MI or Heart Block) ........................................................................ 86 Case 1: Chest pain with initial normal ECG.............................................................................. 87 Case 2: Chest pain with STEMI ................................................................................................ 89 Case 3: Chest pain – v fibrillation / v tachy............................................................................... 90 Case 4: Chest pain – v fibrillation – intoxicated patient ............................................................ 90 Heart Block................................................................................................................................ 91 Headache.................................................................................................................................... 92 Acute Abdominal Pain............................................................................................................... 93 Upper GIT bleeding ................................................................................................................... 98 Lower GIT bleeding................................................................................................................... 99 ECG ......................................................................................................................................... 100 Phone calls ............................................................................................................................... 103 Physical Examination .................................................................................................................. 106 Medical Physical Exam ........................................................................................................... 107 Abdominal examination:.......................................................................................................... 109 Liver Examination ................................................................................................................... 111 Nasal bleeding  Hematological Examination....................................................................... 112 Chest Examination................................................................................................................... 114 Pneumonia ............................................................................................................................... 115 Cardiac Examination – Essential HTN case ............................................................................ 117 Secondary Hypertension .......................................................................................................... 119 Hypertension............................................................................................................................ 120 SOB – shortness of breathe...................................................................................................... 121 DVT ......................................................................................................................................... 122 Peripheral Arterial Disease Examination................................................................................. 125 Diabetic Foot ........................................................................................................................... 126 Neurological Examination ....................................................................................................... 127 Cranial Nerves Examination.................................................................................................... 129 Tremors.................................................................................................................................... 131 Thyroid Exam .......................................................................................................................... 132 Dermatomes ............................................................................................................................. 133 Neck Examination.................................................................................................................... 134 Carpal Tunnel Syndrome ......................................................................................................... 135 Hand Laceration / Wrist Laceration......................................................................................... 136 Back Pain ................................................................................................................................. 137 Acute Back Pain....................................................................................................................... 138 Chronic Back Pain ................................................................................................................... 139 Back Joint Examination ........................................................................................................... 140 Ankle Twist ............................................................................................................................. 141 Shoulder Joint .......................................................................................................................... 142 Elbow....................................................................................................................................... 143 Hip Joint................................................................................................................................... 144 Knee Joint ................................................................................................................................ 145 Obstetrics and Gynecology.......................................................................................................... 146 History taking – OB-GYN ....................................................................................................... 147 OB/GYN cases......................................................................................................................... 147 MGOS history questions:......................................................................................................... 148 History of pregnant lady – third trimester................................................................................ 149 OSCE-guide-III.doc Page 3 of 255 Vaginal Discharge.................................................................................................................... 150 Vaginal Bleeding – Non-Pregnant / Not-Known Pregnant...................................................... 151 Vaginal Bleeding – Pregnant / Ante-Partum Hemorrhage....................................................... 152 Abnormal Uterine Bleeding (AUB)......................................................................................... 153 Amenorrhea ............................................................................................................................. 155 Infertility .................................................................................................................................. 156 Counselling – pre-eclampsia.................................................................................................... 157 Caesarean Section – Counselling – wants to have CS............................................................. 159 Caesarean Section – Counselling – does not want to have CS ................................................ 161 OCPs / Contraception Counselling .......................................................................................... 163 HRT counselling...................................................................................................................... 166 Needle Stick Counselling – HIV ............................................................................................. 168 Counselling – PAP smear ........................................................................................................ 170 Antenatal Counselling.............................................................................................................. 172 Endometriosis .......................................................................................................................... 174 Woman wanting an abortion.................................................................................................... 175 Osteoporosis – Counselling / OR / Short Case ........................................................................ 176 Pediatrics ..................................................................................................................................... 177 History taking – Pediatrics....................................................................................................... 178 Jaundice ................................................................................................................................... 182 IUGR........................................................................................................................................ 184 Crying Baby............................................................................................................................. 185 Chronic Cough – Asthma ........................................................................................................ 186 Anemia..................................................................................................................................... 188 Vomiting .................................................................................................................................. 189 Diarrhea ................................................................................................................................... 191 Mother worried about her child weight ................................................................................... 193 Fever ........................................................................................................................................ 195 Runny Nose / Flu / URTI......................................................................................................... 196 Rash ......................................................................................................................................... 197 Delayed Speech........................................................................................................................ 198 Seizing child counselling ......................................................................................................... 199 ADHD counselling .................................................................................................................. 200 Vaccination counselling........................................................................................................... 202 Child with DM counselling...................................................................................................... 204 Bed wetting counselling / Nocturnal Enuresis......................................................................... 205 Breast feeding counselling....................................................................................................... 207 Psychiatry .................................................................................................................................... 209 Mental status exam – the psychiatry interview........................................................................ 210 DSM-IV-TR............................................................................................................................. 213 History taking – Psychiatry...................................................................................................... 214 Psychosis.................................................................................................................................. 216 Schizotypal personality disorder.............................................................................................. 218 Panic attack.............................................................................................................................. 219 Tiredness OR weight loss ........................................................................................................ 221 Sleep / fatigue notes................................................................................................................. 222 Insomnia .................................................................................................................................. 223 Domestic Violence – Spouse Abuse ........................................................................................ 224 Child Abuse ............................................................................................................................. 226 Domestic abuser....................................................................................................................... 228 Depression ............................................................................................................................... 230 OSCE-guide-III.doc Page 4 of 255 Depression management / counselling..................................................................................... 230 Dysthymia................................................................................................................................ 231 Premenstrual Dysphoric Disorder (PMDD)............................................................................. 232 Abdominal Pain / Headache .................................................................................................... 233 Somatoform disorders DD ....................................................................................................... 234 Drug seeker .............................................................................................................................. 236 Lithium discontinuity............................................................................................................... 238 Manic patient ........................................................................................................................... 240 Suicidal attempt ....................................................................................................................... 241 Eating disorder......................................................................................................................... 243 Mini-mental status exam:......................................................................................................... 245 Dementia.................................................................................................................................. 246 Delirium................................................................................................................................... 248 Case 1: Dad has not been himself / not sleeping well.............................................................. 248 Case 2: DT ............................................................................................................................... 249 Smoking Cessation – counselling ............................................................................................ 250 Refusal to treatment – counselling........................................................................................... 251 Truth telling ............................................................................................................................. 252 Organ Donation........................................................................................................................ 253 OCD......................................................................................................................................... 254 NOTES .................................................................................................................................... 255 OSCE-guide-III.doc Page 5 of 255 History taking – Medicine History taking – Medicine OSCE-guide-III.doc Page 6 of 255 . doc Page 7 of 255 . stroke. heart attack.History taking – Medicine General review:       Introduction: Chief complaint History of present illness Past medical history Family history Social history Introduction Chief complaint Analysis of CC HPI Impact Red flags TIME: Os Cf D Character: PQRST ↑ ↓  Chronic diarrhea: dehydration  Anemia: fatigue  Cancer: metastasis Constitutional symptoms Risk factors Differential diagnosis Allergy Medications PMH: diseases (DM. HTN. cancer) LMP / Last tetanus shot Events: hospitalization / surgery PMH A M P L E FH Any long term disease Any specific disease SH How do you support yourself financially? With whom do you live? SAD OSCE-guide-III. Examples: . In the next few minutes I will do a physical exam on your (e. during which I will ask you to do some movements that may cause some discomfort and may be some pain.g. Start to ask based on the age: MALE FEMALE > 65 yrs  Do you take meds on regular basis? Do you have a list of it? Or the bottles? Do you take sleeping pills?  Do you have difficulties with sleeping?  Do you have difficulties with your balance1. I am the physician in charge today. any falls?  Do you have difficulties with urination (incontinence / retention)?  Do you have changes in your vision / hearing?  Do you have changes in your mood / memory? 50s  Do you have problem drinking  use CAGE  Depression2  identify through social history  ED / Impotence 30s  Psychiatric problems  SAD  social history Teen / 20s  Premature ejaculation  Abortion  STDs  STDs  Eating disorders 2.History Taking: Good evening Mr ….doc Page 8 of 255 . - Physical examination: Good evening Mr …. o < 18 years: use first name o > 18 years: use Mr / Mrs / Ms . I am Dr …. If you have any concerns or questions. Special conditions:  Fatigue  Insomnia  Headache  Abdominal pain  Vaginal bleeding 1 2 Domestic abuse Normal pressure hydrocephalus: ataxia / incontinence / dementia Common triad association: alcohol / depression / suicide OSCE-guide-III. I am the physician on duty now.Stand at the edge of the chair .Go to the examiner  give stickers  use alcohol rub (disinfective) . please let me know. to figure out a working plan that can help you.I understand that you are here because of ….Chief complaint [A] If the CC not known . I am Dr …. please fell free to stop me and let me know. And I will be telling the findings to the examiner while we proceed. if you feel either.How can I help you today? 1. and I understand that you are here today because of ….Good evening Mr …. I understand that you are here today because of …. shoulder).History taking – Medicine Introduction: . And if you have any concerns. please do not hesitate to stop me.Knock the door . 1. I am the physician in charge now. I am Dr …. In the next few minutes I will ask you some questions about your cc. sharp.History taking – Medicine      Chest pain SOB Heart racing Dizziness Numbness [B] If the CC is known CLEAR  Any pain: headache. tired  When the patient uses medical terms: abortion.Position: where did it start? Can you point with one finger on it? .Can you tell me more about this 2.Course: from that time till now. burning? . Start first open-ended questions: . jaundice. awakening you from sleep? . tightness.Do not interrupt . chest pain  Cough  Fever Clarify: 1. Active listening: . occurring at rest.Onset: o How did it start (sudden / gradual)? o Setting: what were you doing? .Quality: how does it feel like? Squeezing.Do not duplicate 3. Body language: nod your head Panic attack NOT clear  Vague symptoms: dizziness.What do you mean? . is your cc all the time or is it on and off (continuous vs.Duration: o Usually given in the question o If > 24 hours  empathy: were you able to sleep Character: + PAIN: PQRST always ask “from the beginning?” .History of present illness TIME (Os Cf D): .Use closed ended questions 2. stabbing.doc Page 9 of 255 . intermittent)? o Continuous:  From the beginning  Does it ↑ or ↓ or the same?  Frequency: is it your first time? o Intermittent:  Frequency: how often did you have it in the last (…)?  Are these attacks similar or different? • ↑ in duration (longer) or ↑ in severity (more severe) or ↑ in frequency (more often)?  What brings it? On doing certain thing.Radiation: does it shoot anywhere? OSCE-guide-III. palpitation  When the patient uses the words: change / difficulty Clarify: . is it more in morning or towards the end of the day? o Any variation? Triggers: o What brings your headache? o Is it related to: stress / lack of sleep / over sleep / flashing lights / smells? o If female: is it related to your periods? Are you taking any meds or OCPs? o Any diet triggers? + Fluids (e.g. stroke. OSCE-guide-III.COCA±B/D: colour. I am going to ask you more questions to see if you have any other symptoms beside your cc. headache + By causes and consequences: e. Allergy b. 5.Past medical history Because it is the first time I see you. how much do you rate this one? o If bad pain  empathy: this must be difficult Timing: o Does it change with time. I need to ask you some questions about your family medical history.Review of systems: + By differential diagnosis: e.Examples: noise / quiet places / movements / resting / coughing / leaning forward / lying down / [D] ASSOCIATED SYMPTOMS: . heart attack. Macrocytic anemia – Vit B12 deficiency 3. 1 being the mildest pain you have ever had and 10 is the most severe.doc Page 10 of 255 . cancer)?  LMP for females  Events: any history of hospitalization / procedures? 4.Near-by systems 3.Now.g.Causes and consequences 6. and by this I mean your parents and siblings. Medications (OTC.In addition to your cc.g. amount ± blood/discharge What ↑ or ↓: . vomitus): . + By systems: e. Rx meds.Risk factors Now. did you notice any other symptoms? . I need to ask you some questions about your past medical history. high blood pressure.Same system 2.  Allergy / Medications: a. I am going to ask you more questions to see if you have any medical conditions that may explain / cause / predispose your cc. herbs)  Past history of diseases for which you see doctor on regular basis (DM.Family history Because it is the first time I see you. chest pain 1. bleeding.History taking – Medicine - - - Severity: on a scale of 10. odour. contents/consistency. diarrhea.Constitutional symptoms [RED FLAGS] 4. supplements.g.What increases or decreases your cc? . Any long term disease in the family? DM. stroke? 3.Social history 1.SAD: a. With a family  how is the relation with …? Is she/he supportive? 3.How do you support yourself financially? 2. Alone  are you in any relationship? Are you sexually active? b.History taking – Medicine 1. Have you ever tried recreational drugs? OSCE-guide-III.With whom do you live? a. How much? ii.Any specific disease runs in the family? 5. Do you drink alcohol? i. heart attack. For how long? c.How do you describe their general health? 2. HTN. Do you smoke? b.doc Page 11 of 255 . Why are you concerned? Worried / occupied patient! Whenever the patient shows non-verbal clues of being worried / occupied: .In the long cases (10 minutes)  use at least 3 empathy statements - Patient says “I’m not ok / I’m not so good”  I am sorry to hear that Patient says “I fell down”  Oh. you need to confirm special medications. by saying. choices Types of questions you can NOT use: leading questions.Torticollis  do you take anti-psychotics? Do you see psychiatrist/ did he give you any medications MSD (mood / suicide / drinking): whenever you find one.First time: Summarize . heart failure.Can you tell me your concerns! .Asthma  aspirin / β–blockers (HTN.In the short cases (5 minutes)  use at least 1-2 empathy statement . what about … . closed-ended. please bear with me for few minutes and I will give you a pain medication as soon as I can .Diarrhea  what about antibiotics . I am Dr … and we are here to make sure you’re ok I have a concern! Whenever the patient says: “I have a concern”: STOP the interview! . ask him: and what happened recently that made you decide to seek medical advice now? Whenever the patient has something affecting his life / social issue: Refer to social worker / services Do NOT criticize other doctors or the patient OSCE-guide-III.Bleeding  what about aspirin / blood thinners . screen for the others When the patient comes with a chronic long duration complaint.Migraine  what about OCPs? (any birth control pills) . and could not find questions to ask. can you tell me more about these difficulties you are facing Patient is regaining consciousness in the ER  Mr … you have had … and you are in the hospital now. social phobias) .I can see that you are worried / occupied! Would you like to tell me more about your worries or concerns? - Question types: Types of questions you can use: open-ended. thyroid disease. did you hurt yourself / “No” – I am glad to hear that After suicide  It looks like you have gone through difficult times.doc Page 12 of 255 .Third time: PMH / FH / SH Medications: When you ask about the medications and even if the patient says NO. .History taking – Medicine Notes EMPATHY: If during history taking you noticed the patient is in pain  empathy: I can see you are in pain. in certain diseases.Second time: Ask about constitutional symptoms / Review of systems . you are ok now. stacking questions Time usage technique: 1/2/3 If you are stuck during the history taking. g. Treatment: life style / medications (side effects / alternatives / consequences of not receiving treatment) 4. The medical condition is called “…” b. there are too many points regarding this issue that it will be better to discuss it with the specialist.doc Page 13 of 255 . we balance the benefits and the side effects. Break every 30-60 seconds (check & recheck that your patient understands). you are not working alone  We will try to make you as comfortable as possible  Oncology group AND palliative group: Referral: do you have any doctor you are comfortable with? o Summarize /+/ Strategy Counselling: 1. ask the patient: does that make sense? Is this acceptable? Reasonable? Is it clear? 6. … d. may affect ability to do certain things. Preventive measures: e. It is better to refer you to the specialist. if you are not sure about any thing.Do not mislead your patient. o Empathy /+/ Expectations: what are your expectations from today’s visit  You will have tremendous support.At the beginning ask whether your patient has a specific concern . Inform the patient a. General tips for the counselling sessions: . GIT symptoms: Nausea / Vomiting . - - Whenever you hear “car accident”: I am sorry to hear that! Was anyone hurt? I am sorry for that Were you driving or a passenger? If you do not know the answer to a question: This is a good question / point. Investigations that might be needed to conclude the condition OR to look for complications 2.Make it interactive not lecturing .History taking – Medicine For breaking bad news [SPIKES] approach: o Setting o Perception of the patient: what do you know about … o Invitation: how much details you would like to know about … o Knowledge: give knowledge in understandable pieces. modify the poly-pharmacy … 3.Distension / bloating / gas Change in bowel movements: constipation / diarrhea Blood in stools / vomiting blood LIVER: yellowish discoloration / itching / dark urine / pale stools OSCE-guide-III. A good statement to use in different counselling situations: always in medicine. Offer more info: brochures / web sites / support groups 5. and make sure the patient understands this info. I will check it for you and we will discuss it next visit. Consequences / complications of the condition! May happen again.Heart burn / acidic taste in mouth Abdominal pain . Explain the pathophysiology c. say that this is a very good question and you are going to check the answer for him. please fell free to stop me and let me know Analysis of  OsCfD: Onset / setting: what were you doing? CC  PQRST: ─ Position: where did it start? Can you point with one finger on it? ─ Quality: how does it feel like? Squeezing. If you have any concerns or questions. any blisters / skin rash on your skin  DVT: any pain / swelling / redness in your legs / calves? Any recent long travel? OSCE-guide-III. tightness ─ Radiation: does it shoot anywhere? Your jaw.History taking – Medicine Chest pain – ACUTE Introduction: Good evening Mr …. to figure out a working plan that can help you. In the next few minutes I will ask you some questions about your chest pain. analyze (OsCfD)  Do you feel your heart racing?  Did you feel dizzy / light headedness / LOC? Are you tired?  Did you notice swelling in your ankles? Legs? Calf muscles? Near by  CHEST: systems ─ Any cough or phlegm? Chest tightness? Wheezes? ─ Recent fever / flu like symptoms? Muscles/ joint ache?  GIT: ─ Difficulty swallowing (esophageal spasm) ─ Heart burn / acidic taste in your mouth? ─ Any hx of PUD? Reflux? GERD?  Chest wall: any trauma.doc Page 14 of 255 . your shoulders. and I understand that you are here because you have chest pain for the last … minutes. your back?  What ↑ or ↓: ─ Breathing / position ─ How did you come to the clinic today? Ambulance  did they give you aspirin / nitrates? Did it help you? Impact  Atherosclerosis: ─ Hx of stroke? Symptoms of stroke (weakness / numbness / change in vision / difficulty finding words)? ─ Any sexual dysfunction? ─ Do you feel abdominal pain after eating? ─ Do you feel cold extremities ─ Do you feel cold feet? Pain after walking?  CHF: ─ SOB? How many pillows do you use? Do you wake up gasping for air? ─ Any swelling in your LL? How high does it go? Is it related to position? ─ Eye puffiness? Pain on the liver? Red flags Constitutional  Fever / night sweats / chills  How about your appetite? Any weight changes? symptoms  Any lumps or bumps in your body? Risk factors CAD Pericarditis PE Differential Same system  Nausea / vomiting diagnosis  Sweating / feeling tired  SOB  if yes. I am the physician on duty now. I am Dr …. Esophageal spasm Questions:  Investigations: ECG / Cardiac enzymes OSCE-guide-III.Trauma Panic attack Pulmonary embolism Panic attack GIT: GIT: .PUD .Unstable angina .Unstable angina Non-cardiac: Non-cardiac .Pericarditis .History taking – Medicine Risk Factors: ─ CAD (Coronary Artery Disease): MAJOR: o High blood pressure o High blood sugar o High cholesterol: have you got your cholesterol measured before? o Family hx of heart attack at age < 50 yrs o SAD: Smoking / Cocaine MINOR: o Look for obesity o Do you exercise o How about your diet.Herpes zoster pneumothorax .Tension .Esophageal spasm .GERD .Pneumonia .Pleurisy .PUD .Aortic dissection .CAD .Cancer .GERD .doc Page 15 of 255 . do you eat a lot of fast food? o Are you under stress? ─ Pericarditis: o Recent flu like symptoms o Medications (Isoniazide / Rifampicin) o Hx of surgery o Hx of heart attack o Hx of kidney disease / puffy face / frothy urine o Hx of TB o Hx of autoimmune disease ─ Pulmonary Embolism: o Recent long flight o History of malignancy o Family history of blood clots o Female: pregnancy / OCPs / HRT Chest Pain Acute Chronic Minutes – hours Hours – days Intermittent Continuous Cardiac: Cardiac: Cardiac: . your back?  What ↑ or ↓: ─ Breathing / position ─ Is it related to activity? How many blocks were you able to walk? And now? ─ How about rest? And during night? ─ When was the last attack  Triggers Angina GERD   Exertion  Golf (leaning forward) Stress (emotional)  Coffee / dairy products Cold air  Smoking / Alcohol Heavy meals  Heavy / late meals Sexual activity  Pregnancy (progestin) Impact Effect  Atherosclerosis  Chronic cough  CHF  Change in the voice Red flags Constitutional  Fever / night sweats / chills symptoms  How about your appetite? Any weight changes?  Any lumps or bumps in your body? Risk factors CAD Differential Same system  Nausea / vomiting diagnosis  Sweating / feeling tired  SOB  if yes.doc Page 16 of 255 . your shoulders. how do you feel now? OsCfD: Onset / setting: what were you doing? PQRST: ─ Position: where did it start? Can you point with one finger on it? ─ Quality: how does it feel like? Squeezing. analyze (OsCfD)  Do you feel your heart racing?  Did you feel dizzy / light headedness / LOC? Are you tired?  Did you notice swelling in your ankles? Legs? Calf muscles? Near by  CHEST: systems ─ Any cough or phlegm? ─ Chest tightness? Wheezes? ─ Recent fever / flu like symptoms? Muscles/ joint ache?  GIT: ─ Difficulty swallowing (esophageal spasm) ─ Heart burn / acidic taste in your mouth? ─ Any hx of PUD? Reflux? GERD?  Chest wall: any trauma.History taking – Medicine Chest pain – CHRONIC Chest pain for 6 weeks UNSTABLE ANGINA /+/ GERD – GIT CAUSES OF CHEST PAIN Intro Analysis of CC … But first I would like to ask you. tightness ─ Radiation: does it shoot anywhere? Your jaw. any blisters / skin rash on your skin  DVT: any pain / swelling / redness in your legs / calves? Any recent long travel? PMH FH SH      OSCE-guide-III. fatigue. is this IHD? Is his heart endangered? ─ This is quite a reasonable concern? What made you think about that? ─ Especially you have many risk factors that may predispose to heart attack. e.5% of users in clinical trials). pantoprazole o Side effects of PPIs:  In general. decreased vitamin B12 absorption may occur with long-term use of protonpump inhibitors and may lead to Vitamin B12 deficiency  Infrequent adverse effects include rash. Then we know for sure the condition of your heart.doc Page 17 of 255 . and anxiety OSCE-guide-III.g. flatulence. GERD stands for Gastro-Esophageal Reflux Disease. ─ On the other hand.g.History taking – Medicine Counselling: Concern: The patient has a concern. we may send you to have a stress ECG. it is less likely your condition is due to heart problem. constipation. we trace your heart while you are exercising. proton pump inhibitors are well tolerated. exercise / diet / smoking / cholesterol. Long-term use is associated with hypomagnesemia  Because the body uses gastric acid to release B12 from food particles. we will do some lab works and an electrical tracing for your heart (ECG). then if we find that we still need. abdominal pain. But we still need to check your heart more. in which. nausea. the most likely diagnosis of what you have is a medical condition called “GERD”. we would like to take measures to try to decrease your risk of developing heart attack. diarrhea. Right now the physical exam is ok. e. any idea about that? Do you know anything about GERD? ─ Explain with a drawing: the esophagus (food pipe) / lower esophageal sphincter / physiologic mechanism to keep it competent / in GERD  weak sphincter  acid refluxes / irritates the esophagus / impact (short term and long term) ─ Treatment: o Avoid triggers o Life style modifications:  Raise the head of the bed  Smaller meals  Do not eat late  ↓ smoking o Medications: proton pump inhibitors (PPIs). and the incidence of short-term adverse effects is relatively uncommon  Common adverse effects include: headache (in 5. itch. and dizziness. ─ However. Drugs) 3. were you able to sleep o Triggers:  What brings your headache?  Is it related to: stress / lack of sleep / flashing lights / smells / diet?  If female: is it related to your periods? Are you taking any OCPs? ─ What ↑ or ↓? Lying down / coughing / resting in quiet room / … Associated symptoms / differential diagnosis: 1.Subarachnoid hemorrhage: ─ Very acute /+/ Very severe headache / the worst headache ─ History of aneurysm or polycystic kidney disease ─ Visual changes (pupil changes) ─ Your heart is beating slow 4. ─ Cranial nerves: o Any change in smelling perception? o Any difficulty in vision / vision loss? o Any difficulty in hearing / buzzing sounds? o Difficulty finding words? Aphasia? o Difficulty swallowing? ─ Brain: o Any dizziness / light headedness / LOC? o Any tremors / jerky movements / hx of seizures? ─ Personality and cognition: o Any memory / mood / concentration problems? o Did anybody tell you that you there is a change in your personality recently? ─ UL/LL: o Any weakness / numbness / tingling in your arms / legs o Any difficulty in your balance / any falls? ─ Spine: o Any difficulty with urination / need to strain to pass urine? o Any change in bowel movements? OSCE-guide-III. you suspect particular cause. temporal arteritis  go to TA block then return to complete the neurological screening. Alcohol.Infection: ─ Fever / night sweats / chills / constitutional symptoms ─ Neck pain / stiffness ─ Recent flu like symptoms / skin rash ─ Bothered by light ─ Nausea / vomiting 2.doc Page 18 of 255 .Neurological screening: If while you are doing the neurological screening.g. e.Subdural hematoma: ─ Trauma / fall ─ SAD (Smoking.History taking – Medicine Headache HPI: ─ OsCfD: gradual onset / all the time / increasing / for few days ─ PQRST: temporal area / vague deep pain / severe o Severe:  empathy: this must be difficult. HTN: ─ Were you diagnosed before with high blood pressure? ─ Do you know your blood pressure? Have you had it checked before? ─ Salty food? Family history of HTN / heart disease? ─ Any history of repeated headaches? 7.Temporal arteritis: ─ Age > 55 years ─ When you touch this part of your head. Taper after ESR decreases < 50 mm/hr and stop if ESR normalizes (< 20 mm/hr) OSCE-guide-III. 15 mg/day (for long periods of time).Extra-cranial causes of headache: ─ Eyes: any hx of glaucoma. immediately start high dose prednisone. 1 mg/kg OD (to prevent blindness) then maintain dose daily (in divided doses). is it painful. red eye. then taper prednisone dose after symptoms resolve. Polymyalgia Rheumatica: ─ Constitutional symptoms + Fatigue ─ Age > 50 yrs ─ ESR > 50 mm/hr Treatment: Corticosteroids. pain in your eyes? Do you usually wear eyeglasses? Do you see well? Any vision problems? When was last time you saw your optometrist? ─ E – do you have any pain / discharge in your ears? ─ N – nasal discharge / sinusitis / hx of facial pain? ─ T – any teeth pain / difficulty swallowing? 8. is it painful? Can you comb your hair? ─ Do you feel cord-like structure? ─ Do you have any visual disturbances / impairment? ─ When you are chewing.doc Page 19 of 255 . cramps in your jaws? ─ Any weakness / numbness in your shoulders / hips? ─ Is there any cough? Mild fever? 6.Medications: ─ Do you take any nitrates? ─ Do use too much of advil (or other NSAIDs)? For how long? ─ Were you used to take large amounts of coffee and then you stopped abruptly? ─ OCPs? Temporal Arteritis: Investigations: ─ TA biopsy ─ Doppler ─ ESR ─ CT head Treatment: If suspect GCA (Giant Call Arteritis).History taking – Medicine 5. Physiotherapy / ms massage / heat compresses (neck) OSCE-guide-III.History taking – Medicine PRIMARY HEADACHE Intermittent / episodic Headache Tension Duration Days Quality Pressing / tightening / bilateral Place Associated symptoms Aggravating factors Others Band around the head Stress Physical injury Treatment .Acetaminophen .Acetaminophen .NSAIDs .doc Migraine Hours Mostly unilateral / pulsating / interferes with daily activities Mostly unilateral Photophobia / phonophobia Physical activity / motion Light / sound  Family history  Types: + Classical: with aura + Non-classical: no aura Acute phase: .Ca channel blockers .Triptans (somatriptan) Page 20 of 255 .NSAIDs (ibuprofen) Cluster Minutes Comes in series / severe pain / hyperaesthesia Around the eyes / nose Red eyes / lacrimation / rhinorrhea / sweating Smoking / alcohol Smell / exercise .Triptans / ergotamine Prophylactic: .Oxygen .NSAIDs .Remove precipitant . I am Dr …. xray. anemia): ─ Why have you done this test? ─ Is it the first time to have it? ─ Who ordered this test for you? Why? ─ When did you have it? 2. we need to do more investigations.First let me ask you few questions about the lab test itself (this applies to any blood work. I need first to ask you some questions to help me get better understanding and interpretation of these results. I have it and I am going to discuss it with you. Most likely diagnosis: pernicious anemia Investigations: ─ CBC / Differential / Peripheral blood film ─ B12 level in the blood / Folic acid level in the blood Introduction Good morning Mr …. could be. Findings: poor diet.History taking – Medicine Blood results / Macrocytic Anemia / B12 Deficiency Cases: ─ ─ Middle age man received blood report showing Macrocytic anemia Elderly (65 years old) man presenting with ataxia. jaundice. I understand that you are here today (OR we called you to come) to get the results of your blood tests (OR x-ray) that you have done few days ago. 3. some of them are serious. biopsy. there is different causes for this.doc Page 21 of 255 . to decrease the patient anxiety. Is it OK with you? + If patient anxious about results  tell her/him then continue history 1. because it is the first time that I see you.I would like to ask you some questions to see how did this (anemia) affect you: CONSEQUENCES of anemia: ─ Anemia symptoms: o Did anyone comment that you are pale. dizziness. o If the patient panicked? Is it serious doctor?  Do you have any concerns?  There are different causes that may lead to this result.Give the information: ─ If it is bad news  SPIKES ─ If abnormal blood results or x-ray: explain the results to patient. macrocytic anemia. recently? o Did you notice any ↓ in your activity level? o Heart racing / SOB / chest pain with exercise? o Any dizziness / light headedness / fainting? ─ Neuro symptoms: o Any tingling / numbness / in your feet? o Difficulty in your balance / any falls? o Any difficulty concentrating / memory problems? OSCE-guide-III. HIV testing. However. The blood tests you had show that you have special kind of anemia that we call “Macrocytic anemia” in which the size of the red blood cells (which are a component of your blood) is larger than usual. or rheumatoid disease / lupus? ─ Terminal ileum: o Did you have any bowel surgery before? o Were you diagnosed with “Crohn’s disease” before? Any repeated attacks of diarrhea? Any foul smelling bulky stools? ─ Pancreatic and liver failure: o Any hx of liver / pancreatic disease? o Yellowish discoloration / itching / dark urine / pale stools? ─ Alcohol: o Do you drink alcohol? How much? For how long? ─ Meds: Do you take medications on regular basis? What kind? o Have you ever been diagnosed with epilepsy? Do you take anti-epileptics? o Do you see a psychiatrist? Do you take a mood stabilizer? o Any hx of chemotherapy? Have you ever taken a drug called “methotrexate”? ─ Hematological causes: o Any recent bleeding (nose / gum / coughing / vomiting blood)? Any bruises / dark urine / tarry stools? o Any fever / night sweats / chills? Change in appetite / weight loss? Lumps and bumps in your body (for LNs)? Bony pains? Any repeated infections? ─ Parasites: o Have you ever consumed raw fish (chronic intestinal infestation by the fish tapeworm: Diphyllobothrium)? 5.History taking – Medicine 4.PMH 6.doc Page 22 of 255 .SH OSCE-guide-III.FH 7.I would like also to ask more questions to find out what might be the cause: CAUSES of Vit B12 deficiency: ─ Diet intake: Are you vegetarian? For how long? Do you take supplements? ─ Gastric causes: o Did you have any surgeries in your stomach? When? o History of long standing PUD? Any heaviness / fullness after meals / indigestion? (Lack of acidity) o Were you ever yourself or any member of your family diagnosed with what is called “autoimmune disease”. by this I mean a condition called “pernicious anemia”. solids then fluids Mechanical Cancer OR stricture   Intermittent On and Off Achalasia: respiratory symptoms Fluids first Then solids Fluids and solids Solids only (Large bolus) Achalasia Scleroderma Esophageal spasm Esophageal webs and rings Scleroderma: reflux / tight skin of fingers / change hand color when exposed to cold (Reynaud’s disease) Mechanical Dysphagia: Analysis  OsCfD: gradual. to solids then fluids / PQRST / What ↑ / ↓ of CC  Associated symptoms: ─ The same system: o Nausea / vomiting / undigested food o Change in bowel movements o Change in the size of the abdomen / abd pain / blood in stools o Liver: yellowish discoloration / itching / dark urine/ pale stools ─ Near-by systems: o Any chest pain / tightness o Any cough / change in your voice / neck swelling (thyroid lump) Impact Weight loss Red flags  Constitutional symptoms: fever/ night sweats/ chills / change in appetite / loss of weight / lumps & bumps  Risk factors: ─ GERD / PUD: o Hx of heart burn o Were you ever diagnosed with a condition called GERD / PUD o Have you ever checked with a camera or a light (endoscope) inserted into your stomach ─ Smoking / Alcohol ─ Family history: esophageal cancer ─ Radiation to chest ─ Have you ever swallowed any chemical? Barium swallow: string sign /or/ apple core sign / graded narrowing of intra-esophageal diameter extending from T5-T8 level  most likely diagnosis: esophageal cancer Investigations: endoscopy and biopsy / chest x-ray and CT / liver function tests / abdominal US OSCE-guide-III.doc Page 23 of 255 .History taking – Medicine Difficulty swallowing What do you mean by difficulty swallowing? ─ Do you feel difficulty initiating the swallowing? ─ Do you feel pain when you swallow? ─ Do you feel food is stuck? Can you point where it is usually stuck? Dysphagia (esophageal) Progressive All the time and ↑ Progressive. ↑ progressively. Give the information: ─ Liver enzymes: AST / ALT ≥ 2 ─ Explanation: there is increase in one of the markers used to assess / check the liver functions. but it is important to ask (start from least offensive to most offensive) Including the alcohol.History taking – Medicine Elevated liver enzymes Introduction HPI: 1.FH: suicide / depression / drinking / liver cancer 7. I would like to ask you some questions to see if you were exposed to liver disease without being aware of that. it may indicate that there is an injury to your liver. 3. some of these questions may be personal.doc Page 24 of 255 .I would like to ask you some questions to see how did this affect you: CONSEQUENCES of liver injury: ─ Acute phase: o Any yellowish discoloration / itching / dark urine/ pale stools o Recently.First let me ask you few questions about the lab test itself (this applies to any blood work. HIV testing. biopsy. jaundice.SH OSCE-guide-III. xray.PMH: ─ Were you ever diagnosed with liver disease before ─ Were you ever checked for liver disease before ─ Were you ever vaccinated for liver disease before 6. I am happy you came here today to discuss it so that we can figure this out. have you noticed any fever / flu-like symptoms / muscle/joint aches o Constitutional symptoms: sweats / chills / appetite / weight / lumps ─ Chronic manifestations: o Did you notice any increase in the size of your abdomen? Puffiness in your face? Swelling in your legs/ ankles? o Bruises in your body? o Vomited blood? Blood in stools? o WITH ALCOHOL: did you notice changes in memory and concentration? Any weakness / numbness? Balance and falls? 4. during which  Drinking assessment 5. anemia): ─ Why have you done this test? ─ Is it the first time to have it? ─ Who ordered this test for you? Why? ─ When did you have it? 2.I would like to ask you more questions to find what might be the cause: CAUSES of liver injury: Now. doc AST / ALT / GGT / Alkaline phosphatase LDH Bilirubin INR / PTT Albumin Glucose Serum ferritin / TIBC / serum ceruloplasmin Viral serology: Hep A/B/C antibodies and Hep B Ag Abdominal US Liver biopsy Page 25 of 255 . women or both? ─ What type of sexual activity do you practice? Did you practice safe sex all the time? And by that I mean using condoms! ─ Any history of sexually transmitted infections? And screening for STIs? ─ Have you ever had sex with sex worker? ─ Within the last 12 months. I would like to ask you some questions to see if you were exposed to liver disease without being aware of that.History taking – Medicine Now. have you had any other sexual partners? When do you need to take extensive sexual history? Risky behaviour!  Liver enzymes / Jaundice  Fever / Tired  LNs  Discharge  PAP results  HIV test results Liver investigations           OSCE-guide-III. did you have other partners? ─ When did you start to be sexually active? How many sexual partners did you have from that time till now? ─ What is your preference. men. some of these questions may be personal. but it is important to ask (start from least offensive to most offensive) Oral ─ ─ Any recent travel outside Canada? Did you eat any raw shell fish? Did you eat in new place that you are not used to? Surgical ─ Any history of surgeries / hospitalization? ─ Any history of blood transfusion? ─ Any history of blood donation? Social ─ ─ How do you support yourself financially? If hazardous occupation? Did you get exposed to blood products / body fluids? Risky behaviour ─ Any tattooing or piercing? ─ SAD? o Do you smoke? o Drink alcohol? How about the past?  Drinking assessment o Have you ever tried recreational drugs? Any injected drug use? When was the last time? ─ With whom do you live? For how long have you been together? ─ Before being with your current partner. History taking – Medicine Drinking / Alcohol Drinking assessment Use / abuse MOAPS Drinking hx How much? CAGE Mood Organic Anxiety Psychosis Self-care / suicide Impact Medical Social Liver Home Memory / conc.doc Page 26 of 255 . Work B12 Heart Legal Use / Abuse: ─ Do you drink alcohol? How about the past? ─ What do you drink? o For how long? o How often? ─ How much? o 2 bottles of wine a day? 12 beers a day? Have you drunk more than 6 drinks in one setting? Have you ever exceeded the amount you intended to drink? o Do you drink alone or with other people? o Did you ever drink to the extent of black out? o What do you feel if you do not drink? Any shaking / heart racing / sweating? Have you ever had seizures before? Were you hospitalized? Did you have delirium tremens? o Do you avoid going to places where you do not have access to alcohol? ─ CAGE: o Did you ever think that you need to cut down on your drinks? o Do you get annoyed by other people criticizing your drinking? o Do you feel guilty for your drinking habits? o Early morning drink? Problem drinking: 2 of CAGE list for males OR 1 for females MOAPS: ─ Mood: How is your mood? Interest? If ok  do not proceed o If not ok  MI PASS ECG Anxiety: o Are you the person who worries too much? o Do you have excessive fears or worries? Psychosis: o Do you hear voices or see things that others do not? o Do you think that someone else would like to hurt you? Self care / suicide o Any chance you might harm yourself or somebody else? o ─ ─ ─ OSCE-guide-III. we take a lot of measures to make sure it is accurate and usually if it is positive.doc Page 27 of 255 . symptoms and examination. we might need to do more tests. let us proceed with history and examination. Chances of having a mistake are very low. are you Canadian graduate. and at the end I will be able to tell you. By the way. follow Canadian guidelines. doctor?  Why do you ask?  Whether I drink or no does not matter. doctor?  I fulfill all the requirements to practice medicine in Canada!  I passed all the exams.History taking – Medicine Impact: I am going to ask some questions to check what effects does alcohol have on your life? ─ The medical is already done in the consequences of liver injury ─ Social: o With whom do you live? For how long? How is the relation? Is there any problems? Is it related to your drinking habits? o How do you support yourself financially? Where do you work? For how long?  How is the relation with your coworkers / manager?  Do you miss working days because of your drinking habits?  Do you need to drink at work? ─ Legal: o Did you have any legal issues related to your drinking? o Fights? Arguments? Were you arrested before because of drinking? o Were you charged before for DUI (driving under influence)? Alcoholic beverages: ─ Beer: o Alcohol percent around 5% o Pitcher (60 oz) = 3 pints (pint = 20 ounces) o Ounce (oz) = around 28-30 ml ─ Wine / Champaign: o Alcohol percent around 12% o Bottle: 750 ml o Glass: 150 ml ─ Hard liquor (whisky / gen): o Alcohol percent around 40% o 1 glass (shot) = 1 ½ oz (50 ml) Ethical challenges (patient asks) Doctor is there any possibility that these test results are wrong? Can we repeat it to make sure?  Whenever we do blood work. Do you drink yourself. the labs double check it before they send it.  Based on your history. we might not only repeat the test. it is better to discuss your case now. and practice under supervision! OSCE-guide-III. It is early to tell now. did you seek medical attention? What made you choose to come today? Compared to before. TB.History taking – Medicine Fever / Tired Intro CC … But first I would like to ask you. how do you feel now?  Fever  Do you have other concerns? Analysis OsCfD  Did you measure it? How often? How? What is highest?  And medications? Did it help?  Any flu / illness / sickness  Any diurnal variation? Any special pattern? Is it more every 3rd or 4th day? (malaria) Impact Are you able to function? Red flags Constitutional symptoms Differential  CNS: headache / neck pain / stiffness / nausea / vomiting / vision changes / diagnosis bothered by light / weakness / numbness  ENT: ─ Ears: pain / discharge Extensive review of ─ Nose: runny nose / sinusitis (facial pain) ─ Throat: sore throat / teeth pain / difficulty swallowing systems  Cardiac: chest pain / heart racing (pericarditis)  Lung (pneumonia.doc Page 28 of 255 . cancer): cough / blood / phlegm / wheezes / chest tightness / contact with TB pt 3  GIT (except the liver ): abd pain / distension / change in bowel movements / blood in stools  Urinary: burning / frequency / flank pain / blood in urine  Do you have any discharge? Ulcers? Blisters? Warts?  MSK: joint pain / swelling / ulcers in your body / mouth / skin rash / red eye  Autoimmune: fm hx / dx before with autoimmune dis  The LIVER: ─ Local: yellow / itching / dark urine / pale stools ─ Dx before with liver dis? Screened? Vaccinated? ─ Transition to risky behaviour PMH Cancer / Autoimmune disease FH Cancer / Autoimmune disease SH Does your partner have any fever? Discharge? Skin rash?   3 During the last 6 weeks. any special changes? The liver will be put at the end as a transition to ask about risky behaviour (see liver enzymes case) OSCE-guide-III. PE (DVT). Any floating fat droplets / difficult to flush / undigested food  Did you notice blood? When did it start? ─ Before you have your bowel move? ─ Mixed (higher source of bleeding)? ─ On the surface? AS  Pain  OCD / PQRST ─ If pain improves after bowel movement: IBS  Vomiting  Alteration with constipation  Acute dehydration: thirsty / dizziness / light headedness / LOC / weak  Constitutional symptoms – for infection / cancer  Flu like symptoms  Any body around you have the same diarrhea?  Other causes: ─ Hyperthyroidisms: heat intolerance ─ Stress? What do you do for life? Any stress? Does the diarrhea ↑ with stress? How about your mood? ─ Infectious: travel / camping / with whom do you live? Any other person at home with diarrhea? ─ HIV – if risk factors ─ Diet: Celiac disease / a lot of dairy products / lactose intolerance / lots of juice / sugars ─ Medications: antibiotics / stool softeners  Rheumatic diseases: red eyes / mouth ulcers / skin changes/ rash / nail changes / hx of psoriasis / joint pain / swelling / back pain / stiffness (especially in morning) / discharge / renal stones   PMH FH SH Bloody diarrhea DD: ─ GE (gastroenteritis) ─ IBD (inflammatory bowel disease) ─ Bleeding peptic ulcer Investigations for clostridium difficile  CBC / differential / lytes and chemistry  Stool culture for parasites  Stool assay for clostridium toxin  Endoscopy  Blood grouping and cross matching OSCE-guide-III.doc Treatment for clostridium difficile  Stop the antibiotic  Metronidazole (500 mg tid x 7 days)  If metronidazole is not effective or severe case  vancomycin (125 mg qid x 14 d) Page 29 of 255 .History taking – Medicine Diarrhea – ACUTE Analysis of CC Impact Red flags Differential diagnosis OsCfD COCA ± B/Mucous ↑↓ How many times? What bout during night? ─ Yes  organic ─ No  irritable bowel syndrome (IBS) – day only ─ How does if affect your sleep?  Consistency: watery / loose / formed / bulky. except the impact and red flags Introduction CC Analysis of CC Impact Red flags Differential diagnosis OsCfD COCA ± B/Mucous ↑↓ How many times? What bout during night? ─ Yes  organic ─ No  irritable bowel syndrome (IBS) – day only ─ How does if affect your sleep?  Consistency: watery / loose / formed / bulky. Any floating fat droplets / difficult to flush / undigested food  Did you notice blood? When did it start? ─ Before you have your bowel move? ─ Mixed (higher source of bleeding)? ─ On the surface?  Pain  OCD / PQRST AS ─ If pain improves after bowel movement: IBS  Vomiting  Alteration with constipation  Acute dehydration: thirsty / dizziness / light headedness / LOC / weak  Chronic  weight loss  Constitutional symptoms – for infection / cancer  For cancer: Age / family hx of Ca colon / change in the calibre of stools / what kind of diet  Rheumatic diseases: red eyes / mouth ulcers / skin changes/ rash / nail changes / hx of psoriasis / joint pain / swelling / back pain / stiffness (especially in morning) / discharge / renal stones  Other causes: ─ Hyperthyroidisms: heat intolerance ─ Stress? What do you do for life? Any stress? Does the diarrhea ↑ with stress? How about your mood? ─ Infectious: travel / camping / with whom do you live? Any other person at home with diarrhea? ─ HIV – if risk factors ─ Diet: Celiac disease / a lot of dairy products / lactose intolerance / lots of juice / sugars ─ Medications: antibiotics / stool softeners   PMH FH SH Rheumatic diseases: IBS / ankylosing spondylitis / psoriasis / reactive arthritis OSCE-guide-III.doc Page 30 of 255 .History taking – Medicine Diarrhea – CHRONIC The same as acute diarrhea. a lot of people have it.75 mg orally every 12 hours (do not exceed 1. how do you think about that? o Is it serious condition doctor?  It is not serious.125 to 0. but it is treatable.g. and do some blood works and stool analysis to rule out other causes.Interstitial cystitis OSCE-guide-III.doc Page 31 of 255 . the most likely explanation for your diarrhea is the medical condition known as “Irritable Bowel Syndrome”. it is like “unhappy colon” o What do you know about IBS? o We do not know the exact mechanism behind this disease. as it does not affect life expectancy. it might be a mood problem o Life style modifications:  Stress management and relief ─ Relaxation techniques such as meditation ─ Physical activities such as yoga or tai chi ─ Regular exercise such as swimming. this is a long term disease.375 to 0. peppermint oil: ─ Offer more information: o I will give you some brochures and web sites in case you want to read more about that Associated diseases: .Chronic fatigue syndrome . walking or running  Diet modification: lactose-free diet or a diet restricting fructose is sometimes recommended  If drinks too much alcohol  advise to decrease alcohol o Medications  Abdominal pain: ─ Hyoscyamine (antispasmodic): 0.Fibromyalgia .25 mg PO or SL q4h or PRN /OR/ extended-release tablets: 0.IBS . and around 80% of patients improve over time ─ Management: o Psychotherapy:  Establish good relationship with the patient  CBT (cognitive behavioural therapy)  If mood is low  depression counselling. and it is a common condition.5mg in 24 hours) ─ Amitriptyline (10 mg qhs)  Diarrhea: ─ Imodium up to 8 tab / day ─ Lomotil  Constipation: ─ ↑ fibre content in diet ─ Metamucil (psyllium): bulk-producing laxative and fibre supplement  SSRIs o Alternative medicine: ─ Probiotics ─ Herbal remedies.History taking – Medicine Counselling: ─ Explanation: o From what you have told me. e. o What I need to do is to do physical exam. did you notice a need to ↑ the doses?  Any attacks during the night?  Do you use peak flow meter?  Did you have PFTs (pulmonary function tests) done?  How many times did you have to go to ER?  Recent chest infection? Flu-like symptoms? Fever / chills?  How do you use puffers? Stored properly? Not expired?  Did you start new medication? β-blockers? Aspirin? Any recent ↑ in dose of these medications?  Exercise  Cold air  Pollens (is it seasonal?)  Dust: construction / smug (smoke/ fog/ exhaust)  Do you smoke? Anybody around you?  Do you have pets? People around you?  Fabrics related: carpets floor? Any change in linen? Pillows? Blankets? Mattress? Curtains?  Relation to any type of food?  Perfumes  Do you live in a house (basement  mold)?  Any construction renovation? Exposure to chemicals?  Any new stressful situations?  Skin allergies  Other allergies Asthma Management 1. repeat PFTs after 20 min. if ↑ > 12%  Asthma OSCE-guide-III.doc Page 32 of 255 . Introduction EVENT Asthma history Triggers Infection Medications Outdoor Indoor Stress PMH and FH How do you feel now?  O S Cf D  Which medication was used? How many times did you need to puff?  Symptoms: SOB / Tightness / Wheezes / Sweating / heart racing / LOC / did you turn blue? Were you able to talk?  Did you call 911 or someone called for you? Did they give you meds? What were these meds?  Were you admitted to hospital? ER? Did they need to put a tube? What were the discharge meds?  When were you diagnosed? How? Type of buffers?  Were you controlled? How many times do you puff (excluding exercise)? Are you using spacer?  Recently. he was treated and discharged with advice to see his family physician. he had asthmatic attack three days ago.Confirm diagnosis: ─ Symptoms: o Cough (dry / more at night / more with exercise / induced by allergens) o Wheezes (noisy breathing) o Chest tightness ─ Examination: wheezes ─ Diagnosis: o Chest x-ray: R/O pneumonia / infection / cancer o Pulmonary Function Tests (PFTs):  FEV1/FVC < 80% of expected  obstructive lung disease  Give bronchodilators. He went to ER.History taking – Medicine ASTHMA Mr … comes to your office as post-ER visit follow-up. aim for SaO2 > 94% ─ Bronchodilators: o Beta 2 agonists: salbutamol 100 mcg 4 puffs q 15-20 min x 3 AND o Ipratropium bromide 4 puffs q 15-20 min x 3 ─ Corticosteroids: hydrocortisone 200 mg IV or prednisone 40-60 mg PO ─ Rehydration: aggressive IV fluids to liquefy bronchial secretions OSCE-guide-III.g.doc Page 33 of 255 .Management: ─ Environment control: avoidance of irritant and allergic triggers (e.Emergency treatment of Asthma: ─ Oxygen.History taking – Medicine 2. avoid smoking / change β-blocker for treatment of HTN) ─ Patient education: the allergic nature of the disease and triggering factors ─ Written action plan: see the diagram below (next page) 3. anticholinergic bronchodilator Leukotriene receptor antagonist (LTRA) Also available as diskus For Asthma and COPD For COPD 4. 1 puff = 100 mcg LABA : Long-acting beta2-adrenoceptor agonist LTRA : Leukotriene receptor antagonist Puffers and LTRA tablets: Medication Ventolin Serevent Flovent Color Blue Blue Orange Active ingredient Salbutamol Salmeterol Fluticasone propionate Advair Purple Pulmicort Brown Fluticasone Salmeterol Budesonide Symbicort Red Atrovent Spiriva Singulair Budesonide Formoterol Green Ipratropium bromide Spiriva Tiotropium bromide Handihaler Tablets Montelukast / puff 100 mcg 50 mcg 125 mcg 250 mcg 500 mcg 250 mcg 50 mcg 100 mcg 200 mcg 400 mcg 400 mcg 12 mcg 20 mcg 18 mcg 4 mg 10 mg Class Short acting β2-agonist LABA ICS Notes ICS LABA ICS Also available as diskus ICS LABA Anti-cholinergic Long-acting. 24 hrs.Medications: Type Mild intermittent Symptoms < 2 times / week Mild persistent > 2 times / week but < 1 time / day Moderate Daily Severe Continuous / Uncontrolled Treatment Short acting β2-agonist: 1-2 puffs (PRN and before exercise) Short acting β2-agonist (Ventolin 100 mcg – 12 puffs qid) LABA (Serevent 50 mcg – 1 puff bid) Add LABA or LTRA (Singulair 10 mg PO qhs) Notes Does not need daily medication Low dose ICS (Flovent 125 mcg – 1 puff bid) Moderate dose ICS (Flovent 250 mcg – 1 puff bid) High dose ICS (Flovent 250 mcg – 2 puffs bid) Oral prednisone LTRAs are second-line monotherapy for mild asthma 6-11 yrs: ICS should be ↑ to moderate dose > 12 yrs: LABA should be considered first Omalizumab (anti IgE) may be considered in patients > 12 yrs ICS : Inhaled Corticosteroids. History taking – Medicine Canadian Thoracic Society – Asthma Management Continuum – 2010 Asthma action plan: OSCE-guide-III.doc Page 34 of 255 . increased sputum amount. bacteria. or < 60 mmHg with cor pulmonale or polycythemia) Ventolin (q6h PRN) + Atrovent (1-2 puffs q6h)  LABA (Serevent 50 mcg/dose) ± Atrovent /or/  LACA (Spiriva): 18 mcg qAM + must stop Atrovent    Bronchodilators Short acting Long acting Respiratory rehabilitation Corticosteroids Inhaled Combination of ICS + LABA Advair: Fluticasone / Salmeterol (250mcg/50mcg) od or bid  Symbicort: Budesonide / Formoterol (400mcg/12mcg) 2 puffs bid Short course of oral corticosteroids: 50mg/d x 5 days  Oral COPDE (COPD exacerbation):  Definition: episode of increased dyspnea. 3. PE. increase in sputum volume or purulence  Etiology: viral URTI. Step-down to oral therapy when tolerated ─ Susceptible for pseudomonas / recent use (within 3 months) of antibiotics or cortisone: piptazo (piperacillin / tazobactam). or increased sputum purulence (change in colour. consider assisted ventilation if decreasing LOC or poor ABGs ─ Supplemental O2 (controlled FiO2: target 88-92% SaO2 for CO2 retainers ─ Bronchodilators by nebulizer o Short acting beta2-agonists used concurrently with anti-cholinergics o Salbutamol and Ipratropium bromide via nebulizers x 3 back-to-back ─ Systemic corticosteroids: IV solumedrol or oral prednisone (50mg/d x 5 days) ─ Antibiotics: often used to treat precipitating infection: o Indications (2 out of 3): increased SOB.g. greenish) o Type of antibiotics: see below ─ Post exacerbation: rehab with general conditioning to improve exercise tolerance  ICU admission ─ For life threatening exacerbations ─ Ventilator support o Non-invasive: NIPPY. CHF. BiPAP o Conventional mechanical ventilation Antibiotics: ─ Outpatient: resp fluoroquinolones: levofloxacin 750 mg PO q24h x 5 days OR beta-lactam + macrolide (amoxicillin 1000 mg PO tid + clarithromycin 500 mg PO bid) ─ Risk factors (group home / hospital infection / immunocompromised): ceftriaxone (1 g IV q24h) + azithromycin (500 mg IV q24h x 5 days). Ml must be considered  Management: ─ Assess ABC. air pollution. coughing. e.375 gm IV q6h) ─ MRSA: Vancomycin 1 gm IV q24h OSCE-guide-III.doc Page 35 of 255 .History taking – Medicine COPD management Prolong survival Smoking cessation Vaccination: influenza virus. pneumococcus (Pneumovax) Home oxygen: to prevent cor pulmonale and decrease mortality if used > 15 hrs/day (indications: PaO2 < 55 mmHg. no blood. ketones.vs. NSAIDs. bi. kidney)  Hx or Dx of DM  Any medications (penicellamine. no WBCs Diagnosis: nephritic syndrome (minimal changes) Investigations: ─ Kidney function tests / urinalysis / 24 hrs protein in urine / renal biopsy ─ Lipid profile / blood glucose studies ─ Hepatitis B serology / ANA / C3 and C4 Management: ─ Salt restriction / avoid fats ─ Diuretics / monitor fluids in and out ─ Anti-HTN: ACE inhibitors ─ Prednisolone OSCE-guide-III.g. BP 150/90. no glucose.lateral OsCfD  What ↑? Walking / standing what ↓? Raising legs  How high does it go? ↑↓ AS  Local symptoms: ─ Pain / fullness / heaviness / tightness ─ Skin changes (redness / swelling / do you feel your feet warm?) ─ Nail changes  Other swellings in your body: ─ How about swelling in your face? Eye puffiness? Do you find it difficult to open your eyes in the morning? ─ How about your belly? Did you need to ↑ the size of your belt? ─ Hands.History taking – Medicine Ankle swelling – Bilateral Introduction CC Analysis of CC Impact Red flags Differential diagnosis uni. protein in urine. did you feel it is tight to wear your ring?  How does this affect your life?  Constitutional symptoms – for infection / cancer Differential diagnosis of BILATERAL ankle swelling:  Failure Heart  Failure Liver  Failure Kidney: history of kidney disease (changes in urine / bruising / frequency / burning / frothy urine / clear or no)  Hypoalbuminemia  Thyroid diseases Specific cause within this system (e. gold.doc Page 36 of 255 . …)  Recent sore throat  Any skin infection / rash  Hx of autoimmune disease  How about diet? Is it balanced? Any diarrhea? PMH FH SH Case: patient with face swelling. vs. bi. Sexual history. sepsis.History taking – Medicine Ankle swelling – Unilateral Introduction CC Analysis of CC Impact Red flags Differential diagnosis uni. previous attacks.lateral OsCfD  What ↑ / ↓?  How high does it go? ↑↓  If pain  PQRST AS  Local symptoms: ─ Pain / fullness / heaviness / tightness ─ Skin changes (redness / swelling / do you feel your feet warm?) ─ Nail changes  Other joints? Toes? Other ankle?  How does this affect your life?  Constitutional symptoms – for infection / cancer Differential diagnosis of UNILATERAL ankle swelling:  Any trauma. any twist in your ankle?  Gout.g. pus. tenderness  Gonorrhea septic arthritis. penile discharge? Unprotected sex recently?  DVT Specific cause within this system (e. discharge.doc Page 37 of 255 . gout)  Tell me more about your diet? Too much protein?  How about alcohol?  Medications? Pain meds (aspirin) / diuretics (furosemide. fever. screen kidney  for kidney stones  Infection. cellulitis. thiazides)?  Hx of cancer / chemotherapy (cytotoxic drugs) / radiation?  Family hx of gout / kidney stones? PMH FH SH DVT: see the physical examination section OSCE-guide-III. g. indomethacin ─ How about water pills? ─ Are you under regular F/U? How often? When was the last time? Were you symptoms free at that time? Diet: ─ Do you have special diet? Salt-free diet? Do you monitor that? ─ Any new changes in diet? ─ Any chance of salty food.When do you say SOB.Do you have any hx of asthma? Lung disease? ─ Any wheezes? Chest tightness? Cough? 3.g. canned food.doc Page 38 of 255 . e. what do you mean? Cardiac or chest? ─ Is it difficult to breathe in and out?  cardiac / anemia ─ Is it difficult to breathe out?  COPD / asthma 2.Do you have any hx of heart disease? ─ No  newly dx ─ Yes  ? acute on top of CHF ─ Any racing heart? Dizziness? LOC? Any hx of HTN?  Is it first time? Or you had it before? When and how were you OSCfD PQRST diagnosed? How about treatment? ↑↓  Is it related to activity? How many blocks were you able to walk? And now?  How about at rest? And at night?  Left ventricle: ─ SOB? How many pillows do you use? ─ Do you wake up at night gasping for air? ─ Cough / crackles?  Right ventricle: ─ Any swelling in your LL? How high does it go? Related to position / standing? Weight gain? ─ Eye puffiness? Swollen face? Pain on the liver?  Other cardiac symptoms: ─ Chest pain? Nausea/vomiting? Sweating? ─ Heart racing / dizziness / LOC? Do you feel tired?  Constitutional symptoms – for infection / cancer  Risk factors for ischemic heart diseases – IHD Causes (that precipitated acute on top of CHF):  Compliance  Diet  Medical DM / Kidney / Liver diseases HTN / heart attacks SAD DD (Causes that precipitated acute on top of CHF):  Compliance: Are you receiving treatment? Which medications do you take? How much? For how long? Any change in medications? Change in dose? ─ Do you take it on regular basis? Any chance that you may skip one or more doses? ─ Do you take it by yourself or do you need help? ─ Did you get your Digoxin level measured before / recently? ─ Did you start new medication? Rx or (OTC) over the counter? e. pickles. dried meet and fish ─  OSCE-guide-III.History taking – Medicine Congestive heart failure – CHF 68 years old man comes to ER with 4 weeks of SOB Analysis of CC Impact Red flags DD PMH FH SH Clarification 1. dizziness. cough. (3) Kerley B lines (thin linear pulmonary opacities caused by fluid or cellular infiltration into the interstitium of the lungs). arrhythmias) o L Lasix (diuretics)  ↓ pre-load (furosemide: 40-500 mg IV) o M Morphine. nausea. ACEIs) • Spironolactone for class Ill-b and IV CHF already on ACEI and loop diuretic • If still uncompensated: Implantable Cardioverter Defibrillator (ICD) o Anti-arrhythmic drugs: for use in CHF with arrhythmia can use amiodarone.doc Page 39 of 255 . furosemide 80 mg OD (furosemide opposes the hyperkalemia induced by beta-blockers. heart block ─ Vision: yellow hallos around objects OSCE-guide-III.History taking – Medicine  Medical: Do you take medications on regular basis? Any new medication? Advil? Any hx of thyroid dx. or digoxin o Anticoagulants: warfarin for prevention of thromboembolic events Digoxin overdose: ─ Anorexia. betablocker. any sweating / diarrhea? Any hx of heart disease / HTN ( A Fib) / heart attack / CAD (ischemia) / did you feel your heart bouncing (arrhythmias)? Any congenital or valvular disease / Chest pain / tightness / dizziness / light headedness / LOC? ─ Any chest / lung disease (wheezes. 2-4 mg IV – decreases anxiety and preload (venodilation) o N Nitrates (venous and arterial dilator  ↑ kidney perfusion) o O Oxygen o P Positive airway pressure (CPAP/BiPAP) – decreases preload and need for ventilation / Position (sit patient up with legs hanging down unless hypotensive) o In ICU or failure of LMNOP: sympathomimetics (dopamine or dobutamine) ─ Chronic heart failure (long term management): o ACEI (slow progression and improve survival) or ARBs (if ACEI not tolerated) o Beta blockers: slow progression and improve survival  Should be used cautiously. management of fluid overload. less than ordinary physical activity  symptoms  Class IV: inability to carry out any physical activity without discomfort. ischemia. (4) Bilateral interstitial infiltrates. symptoms may be present at rest Investigations:  Labs: CBC / lytes / ABG (arterial blood gases) / glucose / INR / PTT / serial cardiac enzymes (q8h x 3) / ECG / fluid balance  Chest x-ray findings of CHF: (1) Enlarged heart. chest tightness) ─ Any kidney disease? Renal failure? ─ Any bleeding? Anemia? ─ ─ ─ New York Heart Association functional Classification of heart failure:  Class I: ordinary physical activity does not cause symptoms of HF  Class II: comfortable at rest. (2) Upper lobe vascular redistribution. ordinary physical activity results in symptoms  Class III: marked limitation of ordinary activity. LOC ─ ECG: PVC. titrate slowly because may initially worsen CHF  Side effects: fatigue / bradycardia  If pt on β-blockers  exacerbation  stop the β-blockers for 2 days o Digoxin (if A Fib OR symptomatic on ACEI) o Diuretics: symptom control.g. vomiting ─ Bradycardia. (5) Bilateral small effusions Treatment: ─ Acute heart failure: o Treat acute precipitating factors (e. what do you mean? ─ Do you feel your heart is going fast CC ─ Or is skipping beats ─ Can you tap it for me please? … ─ It sounds irregular for me!  Is it first time? Or you had it before? When and how were you OSCfD PQRST diagnosed? How about treatment?  Is it related to activity? How many blocks were you able to ↑↓ walk? And now?  How about at rest? And at night?  When was the last attack? And what is the duration of the longest attack?  Is it related to caffeine.History taking – Medicine Heart racing For few weeks Introduction … But first I would like to ask you. any other type of food?  SAD (cocaine or any other stimulant) Impact  CVA (any weakness / numbness / difficulty finding words / visual problems)  Heart failure (SOB / limitation of activity / swelling in your legs / how many pillows do you use??  Other cardiac symptoms: ─ Chest pain? Nausea/vomiting? Sweating? ─ Heart racing / dizziness / LOC? Do you feel tired? Red flags  Constitutional symptoms – for infection / cancer  Risk factors for ischemic heart diseases – IHD DD  Do you take medications on regular basis? Any new medication?  Any hx of thyroid dx. how do you feel? CC Analysis of Clarification When do you say your heart is racing. chocolate. chest tightness)  Any kidney disease? Renal failure?  Any bleeding? Anemia? PMH DM / Kidney / Liver diseases FH  Family history of sudden death at a young age? (cardiomyopathy)  HTN / heart attacks SH SAD Physical  Vitals examination  Cardiac exam (looking for mid-diastolic. any sweating / diarrhea?  Any hx of heart disease / HTN ( A Fib) / heart attack / CAD (ischemia) / did you feel your heart bouncing (arrhythmias)? Any congenital or valvular disease / Chest pain / tightness / dizziness / light headedness / LOC?  Any chest / lung disease (wheezes. coke. at the moment.doc Page 40 of 255 . cough. mitral stenosis. rumbling character)  Thyroid exam  Neurological exam: brief / gross motor and reflexes OSCE-guide-III. Cocaine 15.Medications (e.Smoking 9. this is not uncommon condition. This is a reasonable concern?  AF may lead to embolic event (CVA)  AF may lead to heart failure  AF may lead to V. Fib However. verapamil) ─ Digoxin (in patients with heart failure)  Anti-coagulation (3 weeks prior and 4 weeks after cardioversion) Unstable     SOB BP < 90/60 Chest pain Confusion Cardioversion:  Electrical: 150 joules for A Fib (50 joules for A Flutter)  Pharmacological: procainamide.Caffeine / stimulants 4.Electrolytes imbalance 6.Cardiac surgery 14. 2.doc Page 41 of 255 . some antiarrhythmic meds – class I) 5.History taking – Medicine The patient daughter has a concern: my mother was diagnosed with AF. Stroke / TIA 2 mitral stenosis.Hypertension / CHF 3.Any condition that lead to tachycardia in a susceptible person Causes for TACHYCARDIA 1. digoxin.Valvular heart diseases 9. Diabetes 1 TIA or embolism.Exercise 2. and it is treatable with medications Causes for AF: 1.Pregnancy 3. 1 g / 1 hr infusion Anti-coagulation: Assess stroke risk: determine CHADS2 score in patients with non-valvular AF Risk factor Points CHADS2 score Anti-coagulation CHF 1 0-1 Aspirin 81-325 mg daily Hypertension 1 ≥ 2 moderate risk Warfarin factors or any high risk Age > 75 yrs 1 factor (prior stroke.Congenital heart diseases 10.Fever 7.Loan AF 11.Stress 8. Should I worry about this? 1.Myocarditis 12.Hyper-thyroidism 4.Cardiomyopathy 8.COPD / pneumonia 13. prosthetic valve) OSCE-guide-III.Pheochromocytoma Atrial fibrillation Stable < 48 hours If in doubt  TEE > 48 hours  Rate control: ─ β-blockers ─ Ca ch blockers (diltiazem.IHD 2.Hypovolemia 6.Hyper-thyroidism 10.Too much alcohol (holiday heart) 7.g.Anemia 5. any chest pain. when started? ─ Any OTC? Aspirin? Who prescribed it to you?  Are you getting enough fluids      Page 42 of 255 .  Were you shaking? Certain part of your body or whole?  Were you breathing? Did you turn blue?  Did you bite your tongue? Roll your eyes? Wet yourself?  Were you able to take few steps or did you fall immediately (orthostatic hypotension)?  Before you lose consciousness. or did it happen before? Was it related to Emotions? Coughing? Urination? Did you lose conscious? Did you hit your head? Were you alone or with someone? Did your wife describe it to you? Is she with you? If it is ok with you. CVA. strange smells (epilepsy) ─ Sweaty.History taking – Medicine Fall Orthostatic hypotension 76 years old male patient came to clinic because he fell few days ago. after we finish.doc Did you hurt yourself? How do you feel? Was this the first time. numbness. He was getting out of bed. vomiting. vision changes (stroke) ─ Any flashing light. nausea. LOC or seizure During Before After Impact Red flags DD OSCE-guide-III. lightheaded. heart racing (cardiac) ─ Things are spinning around you (vertigo) ─ Weakness. were you able to recognize the surroundings? Able to talk? Able to move?  Did you feel any weakness. shaky. seizure. did you feel: ─ Dizzy. when he fell to the ground Introduction HPI: analysis of CC Associated Symptoms For any Fall. hungry (hypoglycemia)  How long did it last?  How did you regain consciousness? By yourself or did you need intervention?  After you regain consciousness. I would like to speak with her to get some info. hypoglycemia (already analyzed – before the event)  Environment: is your room well lit? Any chance you tripped?  Do you take any medications? Do you have a list? ─ Go through it one by one ─ Which one was added / changed recently? ─ Each medication: ask about the disease. numbness?  Did you hurt yourself? How do you feel now?  Constitutional symptoms  Risk factor for IHD Any geriatric  Balance patient. ASK about:  Vision  Hearing  Urination  Diseases: arrhythmia / CAD. this leads to pooling of blood in lower extremities  ↓ amount of blood reaching to heart  ↓ blood reaching the brain  they end-up losing their consciousness temporarily. sit for a couple of minutes on the bed before standing up. blood tends to pool in the lower extremities. It means drop in the blood pressure with change of posture. ─ Explain the pathophysiology: o When we change position from lying or sitting to standing. and before you stand up. from lying down. if you are changing positions. o In patients having orthostatic hypotension. e. push your feet against the ground for few seconds. and this leads to drop in blood pressure.g. any chance you are depressed OSCE-guide-III. do this slowly. Notes:  The patient will have a list of medications: ─ Lipitor ─ Hydrochlorothiazide  ask about fluids intake ─ β-blocker ─ Aspirin  ask about bleeding ─ Lorazepam ─ Oxazepam  I can see that you are taking 2 sleeping pills. to discuss with him the possibility of decreasing the dose or changing medications. who prescribed them to you? The same doctor or no? ─ Metformin ─ B12 / B complex  If the patient looks sad / depressed  you look down for me. blood vessels in our body react by narrowing in order to prevent this and to maintain normal blood pressure. Normally. their blood vessels fail to react fast enough.  I will give you brochures and web sites in case you need to read more. and this could be due to age / medications / DM or combination.History taking – Medicine Counselling:  Inform the patient ─ The most likely explanation to what happened is a condition called “postural orthostatic hypotension”.doc Page 43 of 255 . ─ Consequences: this might happen again ─ Investigations: o Blood works / CBC / differential / lytes / kidney and liver function tests o ECG  Preventive measure: ─ Contact the psychiatrist to check the poly-pharmacy. on steps. ─ Meanwhile. the 6 Ps of ischemia: Pallor / Pain / Parathesia / Paralysis / Pulseless / Polar (cold) OSCE-guide-III.doc Page 44 of 255 .B.History taking – Medicine Peripheral vascular disease: Calf pain / swelling Introduction Analysis of CC Impact Red flags DD PMH FH SH OsCfD PQRST ─ P: unilateral or bilateral ─ R: what about other joints. knees? Thighs? Feet?  What ↑ or ↓: did you notice that your pain ↑ while walking up or down hill? ─ ↑ while walking uphill: peripheral arterial disease ─ ↑ while walking downhill: spinal stenosis  Is it first time? Or you had it before? When and how were you diagnosed? How about treatment?  Is it related to activity? How many blocks were you able to walk? And now?  How about at rest? And at night?  When was the last attack? And what is the duration of the longest attack?  History of strokes / TIAs / neurological symptoms  Chest pain / SOB / heart racing  Pain after eating (intestinal ischemia)  Effect of pain on daily activities / work?  Leriche syndrome (aorto-iliac occlusive disease): numbness in buttocks & thighs / absent or decreased femoral pulses / impotence  Constitutional symptoms – for infection / cancer  Risk factors for ischemic heart diseases – IHD ─ Smoking? How much and for how long? ─ High blood pressure? For how long? Controlled or not? ─ Diabetes mellitus ─ Cholesterol measured? When? What was it? Peripheral Arterial Disease versus Spinal Canal Stenosis Vascular symptoms Neuro symptoms  Cold feet / ulcers  Weakness / numbness / tingling  Swelling / redness  Back trauma / back pain  Delayed wound healing  Sexual dysfunction / difficulty with  Nail changes / hair loss erection Past history of heart disease / stroke / symptoms of stroke / DM / Kidney / Liver diseases Family history of heart disease / HTN / heart attacks SAD   N. do you feel that you emptied your bladder completely or do you need to go again? Irritative (frequency  UB disease): ─ How many times do you go to the washroom? o How about before? Any change? o How about during night time? How does this affect your sleep? How about your concentration and mood? ─ Do you need to rush to washroom? Are you able to make it all the time? ─ Have you ever lost control or wet yourself? ─ Any burning sensation? Any flank pain? ─ Fever / night sweats / other constitutional symptoms Urine analysis (changes): ─ COCA + B (content: frothy / cloudy / not clear) ─   Summary of irritative symptoms: FUND Frequency / Urgency / Night time / Discomfort OSCE-guide-III.doc Page 45 of 255 .History taking – Medicine Urinary symptoms:  Obstructive (prostatic disease in ♂  anuria): Difficulty to initiate urine? Do you need to strain? ─ Any changes in the stream? ─ Any dripping? ─ After you pass urine. e. terazosin) / 5-α-reductase inhibitors (finasteride) ─ Surgery: open surgery / TURP / minimally invasive (stent / laser ablation / cryosurgery) OSCE-guide-III.doc Page 46 of 255 . like those used for incontinence.History taking – Medicine Anuria Introduction Analysis of CC Impact Red flags DD PMH FH SH Empathy – how do you feel right now?  OsCfD  PQRST  What ↑ or ↓  Is it first time? Or did it happen before? When and how were you diagnosed? How about treatment? Associated symptoms:  Obstructive symptoms  Irritative symptoms  Urine analysis (changes): COCA ± Blood Local symptoms:  Any problems with passing stools? What? When?  Any masses in the groin / pelvic mass / pain?  Abdominal pain? Distension? Metastasis  Back: pain / weakness / numbness  Liver: yellow / itchy / urine / stools  Lungs: cough / phlegm / hemoptysis  Brain: headache / nausea / vomiting Renal failure Generalized swelling / face puffiness / itching Sexual Sexual dysfunction  Constitutional symptoms – for infection / cancer  Risk factors for cancer prostate / bladder ─ Were you ever diagnosed with prostate disease? Screened for prostate diseases? (DRE or PSA) ─ Family history of prostate disease / cancer? ─ Ca bladder (radiation / exposed to chemicals / aniline dye) ─ Smoking? Alcohol?  Renal stones: Have you ever had a renal stone? Any history of colicky pain in flanks? Have you ever passed a small crystals or stone during voiding? Hx or repeated UTIs?  Medications: glaucoma / anti-psychotic meds / anti-cholinergic drugs. Ditropan (Oxybutynin).g. Detrol (Tolterodine)  2 Neuro: ─ Back problem: trauma – metastasis – cauda equine (spoiled himself with stools / buttocks numbness) ─ Stroke (diagnosed / weakness / numbness / difficulty)  2 Cancer: ─ Cancer prostate ─ Ca bladder (hematuria) AMPLE DM / anemia / polycystic kidney disease / renal stones SAD Most likely diagnosis: BPH Other possible diagnoses: UTI / prostatitis / Ca prostate Investigations: urea & creatinine / urinalysis / renal US / DRE & PSA / TRUS If cancer is suspected: bone scan / CT Treatment: ─ Watchful waiting: may resolve spontaneously ─ Medical treatment: α-adrenergic antagonists (doxazosin. went to walk in clinic for sore throat and was prescribed Biaxin. Diagnosis: coagulopathy. developed hematuria. OSCE-guide-III. fib for 2 yrs.History taking – Medicine Hematuria Introduction Analysis of CC Impact Red flags DD PMH FH SH Investigations: Empathy – how do you feel right now?  OsCfD  Timing: ─ Initial versus terminal or total ─ Diurnal variation  What ↑ or ↓  Painful or Painless  Is it first time? Or did it happen before? When and how were you diagnosed? How about treatment? Associated symptoms:  Obstructive symptoms  prostate disease  Irritative symptoms  UB disease  Urine analysis (changes): COCA ± Blood Local symptoms:  Any problems with passing stools? What? When?  Any masses in the groin / pelvic mass / pain?  Abdominal pain? Distension? Metastasis Renal failure Generalized swelling / face puffiness / itching Sexual Sexual dysfunction  Constitutional symptoms – for infection / cancer  Risk factors for cancer prostate / bladder / RENAL ─ Were you ever diagnosed with prostate disease? Screened for prostate diseases? (DRE or PSA) ─ Family history of prostate disease / cancer? ─ Family history of cancer bladder or kidney? ─ Ca bladder (radiation / exposed to chemicals / aniline dye) ─ Smoking? Alcohol?  Renal stones: Have you ever had a renal stone? Any history of colicky pain in flanks? Have you ever passed a small crystals or stone during voiding? Hx or repeated UTIs?  Medications: blood thinners / aspirin / bleeding from other sites?  Pseudo-hematuria: ─ Diet: eating too much beet ─ Medications: Rifampicin ─ Other bleeding: bleeding per rectum / vaginal bleeding  AMPLE  History of hemolytic anemia / polycystic kidney DM / anemia / polycystic kidney disease / renal stones SAD (1) Kidney: urinalysis (casts / crystals / C&S / cytology) / ultrasound (abd/pelvic) / IVP / KFTs (2) Bladder: cystoscopy (3) Prostate: PSA / TRUS (4) Others: CBC / differential / INR Case: patient on warfarin for A.doc Page 47 of 255 . extravasation ─ Cystoscopy for suspected bladder stone ─ Strain all urine  stone analysis ─ If recurrent stone formers. oxalate. xanthinuria. etc. calcium ─ Dehydration (especially in summer months) ─ Sedentary lifestyle ─ Medications: thiazides ─ UTI (with urea-splitting organisms) ─ Hypercalcemia disorders: hyperparathyroidism. RBCs. Investigations ─ Screening labs o CBC: elevated WBC in presence of fever suggests infection o Electrolytes.History taking – Medicine Renal stones Risk Factors ─ Hereditary: RTA. cystinuria.g. G6PD. ureters. uric acid. PO4. purines. histoplasmosis. etc. indinavir) / 90% of stones are radiopaque o CT scan: no contrast. distinguish radiolucent stone from soft tissue filling defect o Abdominal ultrasound: may demonstrate stone (difficult for ureters) / may demonstrate hydronephrosis o IVP (not usually done): anatomy of urine collecting system. conduct metabolic studies o Serum electrolytes. sarcoidosis. BUN  to assess renal function o Urinalysis: R&M (WBCs. degree of obstruction. Ca. bladders (KUB) x-ray to differentiate opaque from non-opaque stones (e. Cr. oxaluria. creatinine and urea o PTH if hypercalcemic Approach to renal stone: OSCE-guide-III. ─ Dietary excess: Vitamin C. uric acid. crystals).doc Page 48 of 255 . C&S ─ Imaging o Kidneys. doc Page 49 of 255 .5 cm  Percutaneous nephrolithotomy.5 to 7) / shockwave lithotripsy not effective  Cystine: alkalinize urine (bicarbonate / potassium citrate) / penicellamine / captopril (forms complex with cystine) / shockwave lithotripsy not effective ─ Interventional: o Procedural / surgical: If stone is > 5 mm or presence of complication o Kidney  Extracorporeal shockwave lithotripsy (ESWL) if stone < 2.History taking – Medicine Treatment – Acute: ─ Medical: o Analgesics (Tylenol #3) o NSAIDs help lower intra-ureteral pressure o ± antibiotics for UTI o ± (antiemetic + IV fluids) for vomiting ─ Interventional: o Ureteric stent (cystoscopy) o Percutaneous nephrostomy (image-guided) ─ Admit if necessary: o Intractable pain o Intractable vomiting o Fever (? infection) o Compromised renal function o Single kidney with ureteric obstruction / bilateral obstructing stones Treatment – Elective: ─ Medical: o Conservative if stone < 5 mm and no complications o Fluids to increase urine volume to > 2 L/day (3-4 L if cystine) o Specific to stone type:  Calcium oxalate stones: thiazides / potassium citrate (alkalinization of urine)  Calcium struvite: antibiotics for 6 wks (stone must be removed to treat infection)  Uric acid: allopurinol / potassium citrate (alkalinization of urine to pH 6.5 cm + Staghorn + UPJ obstruction + Calyceal diverticulum + Cystine stones o Ureter  ESWL is the primary modality of treatment  Ureteroscopy (extraction or fragmentation) if failed ESWL / Ureteric stricture o Bladder  Transurethral cystolitholapaxy  Remove outflow obstruction (TURP or stricture dilatation} Management of UTI: ─ Investigations: o Urine for culture and sensitivity o Blood: CBC / differential o Imaging (if suspect complicated pyelonephritis or symptoms do not improve with 72 hours of treatment): Abd/pelvic U/S / IVP / Cystoscopy / CT ─ Pregnant: amoxicillin 500 mg TID x 7 days ─ Non-pregnant: o Septra (sulfamethoxazole and trimethoprim) DS (800/160): 1 tab bid x 7 days o /OR/ Ciprofloxacin 500 mg bid x 7 days ─ Pyelonephritis: o Ceftriaxone (third-generation cephalosporins): 1 g IV q24hrs x 2 days o Then continue oral ciprofloxacin x 7 days ─ Abscess: + drain OSCE-guide-III. indications: + Size > 2. History taking – Medicine Incontinence Obstructive / 62 years old female.doc Page 50 of 255 . neurologic problem. aging  Intrinsic sphincter deficiency (ISD): pelvic surgery. stone)  Stress incontinence  Urethral hypermobility: childbirth. with hx of 3 years of urinary incontinence Introduction Empathy – how do you feel right now? Analysis of  OsCfD CC  What ↑ or ↓: lifting objects / coughing / straining  Is it first time? Or did it happen before? When and how were you diagnosed? How about treatment? Impact Red flags DD Associated symptoms: If at any time there is a frequency or  Obstructive symptoms some new symptom  analyze it  Irritative symptoms first then resume!  Urine analysis (changes): COCA ± Frequency in ♀  UTI Blood Local symptoms:  Any problems with passing stools? What? When?  Any masses in the groin / pelvic mass / pain?  Any perineal skin lesions?  How does it affect your life? Daily activities?  Constitutional symptoms – for infection / cancer  Risk factors (MGOS):  Menopausal symptoms. inflammation / infection (cystitis). Pharmaceuticals / Psychological. Causes of reversible urinary incontinence (DIAPERS): Delirium. TVT / TOT4. Stool impaction 4 TVT: Tension-free Vaginal Tape. Inflammation / Infection. Atrophic vaginitis. aging and hypoestrogen state Diagnosis: ─ History ─ Urinalysis + C&S (if infection suspected) ─ Urodynamics ─ Stress test Treatment of urge incontinence Treatment of stress incontinence ─ Bladder habit training ─ Weight loss ─ Botox (botulinum toxin) injection ─ Kegel’s exercises ─ Medications: anti-cholinergics. TCAs sphincters) N. Restricted mobility. Excess urine output. pelvic surgery. ─ Bulking agents Tolterodine (Detrol). bladder neck obstruction (tumour.B. TOT: Trans Obturator Tape OSCE-guide-III. Oxybutynin ─ Surgery (slings. and HRT M  LMP  Gynaecological history G  Previous abdominal or pelvic surgeries  Obstetric: How many pregnancies? Route of delivery? O  Sexual: Repeated infections / dryness / dyspareunia S  Overflow incontinence  Urge incontinence  Detrusor overactivity: CNS lesion. artificial (Ditropan). Did you notice any vaginal discharge/ bleeding? Any pain/ blisters/ warts? Discoloration/ itchiness? HIV / Lymphoma / Leukemia / Infectious mono-nucleosis     History of cancer History of cancer / lymphadenopathy  Vital signs  Neck exam / Thyroid exam if the swelling is central  LNs / Lymphatic system / LNs in groin / pelvic exam  Liver / Spleen Notes: ─ ─ Whenever there is IV drugs  screen for liver symptoms / HIV Whenever there is risk for STIs  screen for liver symptoms and PID OSCE-guide-III. last time. cold intolerance / sweating / hand shaking / heart racing / diarrhea vs.History taking – Medicine Lump – Neck Swelling Introduction Analysis of CC: The lump Associated (local) symptoms Impact Red flags Differential Diagnosis PMH FH Physical exam Can you point to it? OSCfD / Anything special at that time? Fever? Rash? Is it painful? PQRST Can you estimate its size for me? Is it like a lent.doc Page 51 of 255 . constipation  How does this affect your life?  Do you feel tired? ? HIV  Easy bruising? Repeated infections? ? Leukemia  Constitutional symptoms  Bone pains / Tender points HEAD SSS  risky behaviour:  A: includes recent travel  SAD: how about injection drugs? Did you share needles?  Sexual hx: Detailed (safe sex. how many partners). olive. or larger? Did it change in size? How fast was the change in size?  Did you try to feel it? Does it feel soft / rubbery / hard?  Do you feel it is fixed or moving?  Any skin changes? Redness? Ulcers?  Any history of trauma?  Is it the only one? Did you notice other lumps in your body? How about other side of your neck? Arm pits? Groins?  Rule out infection: Any recent flu-like symptoms? Do you feel tired/ fatigue? History of sinusitis/ Pain in your face? Runny nose? Pain/discharge in ears? Any sore throat/ oral ulcers/ tooth pain? Difficulty swallowing? Neck stiffness/pain? Headache? Vomiting?  Thyroid (if central): heat vs. lemon. g. is it LT / RT? Upper / Lower? Outer / Inner? How about the other breast?   DO NOT POINT WITH YOUR HANDS OR FINGERS! OSCfD / Anything special at that time? Fever? Rash? Is it painful? PQRST Can you estimate its size for me? Is it like a lent. olive.doc Page 52 of 255 .History taking – Medicine Lump – Breast Introduction Analysis of CC: The lump Can you point to it? Is it one breast or both? Where did you notice it? You can ask verbally. lemon. TB. or larger? Did it change in size? How fast was the change in size?  Did you try to feel it? Does it feel soft / rubbery / hard?  Do you feel it is fixed or sliding (moving)?  Any skin changes? Redness? Ulcers?  Any history of trauma?  Is it the only one? Did you notice other lumps in your body? How about your neck? Arm pits? Groins?  Is it related to your period? Does it change with the period?  Any nipple changes? Discharge? Bleeding? Itching? Associated (local)  Rule out infection: Any recent flu-like symptoms? Do you feel tired/ fatigue? symptoms Impact  Headache/ vomiting? (consequences of  Back pain/ weakness/ numbness/ tingling in arms or legs? cancer:  Chest pain/ cough/ phlegm/ wheezes/ heart racing? metastasis)  Liver: yellow discoloration/ itching/ urine/ stools? Red flags  Constitutional symptoms  Bone pains / Tender points Risk factors of cancer: MGO  Menstrual history: first period / last period / regular?  G: OCPs?  Obstetric: History of pregnancies? Number of pregnancies? First pregnancy at what age?  Breast feeding?  Diet rich in fat  PMH or FH of cancer breast / ovarian carcinoma Differential  Benign disease Diagnosis  Trauma  fat necrosis PMH History of cancer breast / ovarian carcinoma FH History of cancer breast / ovarian carcinoma DD for Breast Mass: ─ Breast Cancer ─ Sclerosing adenosis ─ Fibrocystic changes ─ Lipoma ─ Fibroadenoma ─ Neurofibroma ─ Fat necrosis ─ Granulomatous mastitis (e. ─ Papilloma / papillomatosis sarcoidosis) ─ Galactocele ─ Abscess ─ Duct ectasia ─ Silicon implant ─ Ductal / lobular hyperplasia    OSCE-guide-III. reduction)  Firm. local pain  Risk of secondary infection (abscess. and fibrosis  May present with nipple discharge. No ↑ risk of breast cancer / Age 30 to menopause / ↑ pre-menstrual. periductal / subareolar  Unilateral localized pain. ill-defined mass with skin or nipple retraction. nipple inversion  Rule out inflammatory carcinoma. focal areas of nodularity or cysts often in upper outer quadrant  Treatment: Evaluation of breast mass and reassurance / Analgesia (ibuprofen. nodule on U/S  Treatment: excision of involved duct to ensure no atypia ─ Other lesions: o Fat Necrosis  Uncommon. early childbirth OSCE-guide-III. nontender. mobile. nulliparity.History taking – Medicine    Galactorrhea (prolactinoma): normal prolactin level: 10-20 ng/ml (non-pregnant)  If < 40 ng/ml: follow-up  If > 40 ng/ml: CT and bromocriptine  If progressive ↑ / headache / affecting vision  surgery Benign Breast Lesions: ─ Non-proliferative lesions: o Aka fibrocystic changes. after breast surgery (i. hormone-dependent. breast mass. nipple discharge. first pregnancy >30 years old. rapidly growing on serial US.doc Page 53 of 255 . rubbery. >5 years HRT  Decreased risk with lactation. mastitis)  Resolves spontaneously o Abscess  Lactational vs. result of trauma (may be minor – commonly a tight bra. bluish mass under nipple. inflammation. well-circumscribed.e. mammary dysplasia  Benign condition characterized by fibrous and cystic changes in the breast. ASA) / If > 40 years old: mammography every 3 years or biopsy ─ Proliferative lesions: o Fibroadenoma:  Most common benign breast tumour in women under age 30  Risk of subsequent breast cancer is increased only if fibroadenoma is complex. U/S to assess for presence of abscess Breast Cancer: ─ 1/9 women in Canada will be diagnosed with breast cancer in their lifetime ─ Risk factors:  Prior history of breast cancer  1st degree relative with breast cancer (greater risk if relative was premenopausal)  Increased risk with high breast density.  Breast pain. Consider excision if size 2-3 cm. chronic cystic mastitis. unilateral bloody nipple discharge). discrete. but complete imaging ± biopsy to R/O cancer o Mammary Duct Ectasia  Obstruction of a subareolar duct  duct dilation. early menopause. there is adjacent atypia or a strong family history of breast cancer  Clinical features: nodules: smooth. if persistent total duct excision (definitive)  If mass does not resolve: fine needle aspiration (FNA) to exclude cancer. erythema. as indicated  Treatment: initially broad-spectrum antibiotics and I&D (incision and drainage). if symptomatic or pt preference o Intra-ductal Papilloma  Solitary intra-ductal benign polyp  Present as nipple discharge (most common cause of spontaneous. late menopause (> 55 yrs). ± tenderness  Regress spontaneously. early menarche (< 12 yrs). subareolar mass. positive history in only 50%). Needle aspiration yields no fluid  Investigations: Core or excisional biopsy required  Treatment: Generally conservative. serial observation. tenderness. edema. often multifocal / 80% non-palpable. tender breast ± lump ─ Peau d'orange indicates advanced disease (III-b – IV) Treatment of breast cancer: Stage Primary treatment options Adjuvant systemic therapy 0 (in situ) BCS + radiotherapy None I BCS (or mastectomy) + axillary node dissection + May not be needed radiotherapy II Chemotherapy and / or hormone therapy III mastectomy + axillary node dissection + radiotherapy Inflammatory IV Surgery as appropriate for local control BCS = breast-conserving surgery OSCE-guide-III. detected by screening mammogram. most aggressive form of breast cancer. ─ Clinical features: erythema. eczematoid lesion  Inflammatory carcinoma (1-4%): ductal carcinoma that invades dermal lymphatics. pain.g. Ashkenazi Jewish) o Family history of male breast cancer o Young patient ( <35 years old) Pathology o Non-invasive: ductal carcinoma in situ (DCIS): completely contained within breast ducts. or high grade o Invasive:  Invasive ductal carcinoma (most common 80%): hard. low specificity  Galactogram / ductogram (for nipple discharge): identifies lesions in ducts  Metastatic workup as indicated (usually after surgery or if clinical suspicion of metastatic disease) – bone scan. harder to detect mammographically (may benefit from MRI)  Paget's disease (1-3%): ductal carcinoma that invades nipple with scaling. abd U/S. need experienced practitioner for adequate sampling o U/S or mammography guided core needle biopsy (most common) o Excisional biopsy: only performed as second choice to core needle biopsy.History taking – Medicine ─ ─ ─ ─ ─ Investigations o Mammography  Screening: every 1-2 years for women age 50-69 / If positive family history in 1st degree relative: every 1-2 years starting 10 years before the youngest age of presentation  Diagnostic: investigation of patient complaints (discharge. lump)  Follow-up after breast cancer surgery  Findings indicative of malignancy: mass that is poorly defined. send fluid for cytology if blood or cyst does not completely resolve o Fine needle aspiration (FNA): for palpable solid masses. Does not form micro calcifications. architectural distortion. CXR.doc Page 54 of 255 . should not be done for diagnosis if possible Genetic Screening: consider testing for BRCA 1/2 if: o Patient diagnosed with breast AND ovarian cancer o Strong family history of breast / ovarian cancer (e.  Treatment: lumpectomy with wide excision margins + radiation OR mastectomy if large area of disease. head CT Diagnostic Procedures o Needle aspiration: for palpable cystic lesions. normal mammogram does not rule out suspicion of cancer based on clinical findings o Other radiographic studies:  Ultrasound – differentiates between cystic and solid  MRI – high sensitivity. spiculated border. micro-calcifications. infiltrating tentacles  Invasive lobular carcinoma (8-15%): 20% bilateral. swollen. warm. History taking – Medicine Dizziness Causes Vertigo Symptoms ─ Imbalance ─ N&V ─ Auditory ─ Neurological ─ Nystagmus Non-vertigo Peripheral ─ Benign paroxysmal positional vertigo ─ Ménière's disease ─ Vestibular neuritis ─ Labyrinthitis ─ Acoustic neuroma EAR ─ Mild-moderate ─ Severe ─ Common Central ─ Stroke ─ TIAs ─ Brain tumour ─ MS ─ Cerebellar lesion ─ ─ Unidirectional ─ Common ─ Bidirectional (horizontal or rotatory) Clarification Analysis of CC Impact Red flags DD Syncope Vertigo Cerebellar lesion Cardiac ─ Arrhythmias ─ CAD / MI ─ CHF ─ Aortic stenosis ─ Postural hypotension Non-cardiac ─ Vaso-vagal episode ─ Panic attack ─ Somatization Brain / Neuro ─ Severe ─ Variable ─ (horizontal or vertical) Do you feel You have blackout (syncope) OR the room is spinning around you (vertigo) OCD Timing: when / frequency What ↑ or ↓: certain position Did you lose consciousness? Did you fall to ground? Did you hit your head? Constitutional symptoms  Vasovagal attack: LOC / while straining or urinating / nausea / do you feel warning signs before the dizziness?  Cardiac (tight AS / arrhythmia): heart racing / chest pain / immediate (no warning signs)  Hypotension: antihypertensive meds. change of dose or new medication  Postural hypotension: diabetes / dehydration / parkinsonism  Neuro (stroke / TIAs): vision changes / loss. weakness        Condition Benign Paroxysmal Positional Vertigo Duration Seconds to minutes Hearing loss – Tinnitus – Ménière's disease Vestibular neuritis Labyrinthitis Acoustic neuroma Minutes to hours Hours to days Days Chronic Fluctuating Unilateral Unilateral Progressive + – Whistling +     Other features ─ Certain positions ─ Nystagmus Ear fullness Recent AOM Ataxia CN VII palsy ─ ─ Risk factors for CAD: HTN / DM / cholesterol / smoking Unbalanced gait Aspirin / blood thinners Alcohol: neuropathy / cerebellar degeneration PMH / FH / SH OSCE-guide-III. speech impairment.doc Page 55 of 255 . turned to one side at 45° holding the position for 20 seconds. results in complete SNHL. diazepam Convalescent phase:  Progressive ambulation especially in the elderly  Vestibular exercises: involve eye and head movements. and vertigo lasting minutes to hours ─ Acute onset of disabling vertigo often accompanied by nausea. or purulent (bacterial) ─ Occurs as complication of acute and chronic otitis media.History taking – Medicine Condition Benign Paroxysmal Positional Vertigo (BPPV) Ménière's disease Vestibular neuritis Management Acute attacks of transient vertigo lasting seconds to minutes initiated by certain head positions. accompanied by torsional (rotatory) nystagmus Diagnosis: ─ History ─ Positive Dix-Hallpike manoeuvre ─ Reassure patient that process resolves spontaneously ─ Particle repositioning manoeuvres: Epley’s manoeuvre Episodic attacks of tinnitus. blood cultures Treatment: ─ IV antibiotics ─ Drainage of middle ear ─ ± mastoidectomy Investigations: ─ MRI with gadolinium contrast is the gold standard ─ Audiogram – SNHL (sensori-neural hearing loss) ─ Vestibular tests: normal or asymmetric caloric weakness (an early sign) Treatment ─ Expectant management if tumour is very small or in elderly ─ Definitive management is surgical excision ─ Other options: gamma knife. standing. vestibular sedatives (Gravol). anti-emetics. vomiting and imbalance without hearing loss that resolves over days leaving a residual imbalance that lasts days to weeks Labyrinthitis ─ Acute infection of the inner ear Acoustic neuroma resulting in vertigo (days) and hearing loss ─ May be serous (viral). (3) Betahistine (Serc) prophylactically to decrease intensity of attacks  Surgical: selective vestibular neurectomy or transtympanic labyrinthectomy Must monitor opposite ear (bilaterality in 35% of cases) Acute phase:  Bed rest. (2) Local application of gentamicin to destroy vestibular end-organ. bacterial meningitis and cholesteatoma Schwannoma of the vestibular portion of CN VIII OSCE-guide-III. aural fullness (pressure / warmth).doc ─ ─ ─ ─ ─ ─ ─ (performed by MD) OR Brandt-Daroff exercises (performed by patient) Surgery for refractory cases Anti-emetics for nausea/vomiting Drugs to suppress vestibular system delay eventual recovery and are therefore not used Acute management may consist of bed rest. and walking Investigations: ─ CT head ─ If meningitis is suspected: lumbar puncture. sitting. hearing loss. diuretics (hydrochlorothiazide). anti-vertiginous drugs (betahistine) Long term management may include:  Medical: (1) Low salt diet. Onset of vertigo is noted and the eyes are observed for nystagmus Page 56 of 255 . radiation Dix-Hallpike Positional Testing: the patient is rapidly moved from a sitting position to a supine position with the head hanging over the end of the table. History taking – Medicine INR – Counselling Analysis Impact Red flags History / Give the information DVT relapse / Bleeding Female: OCP / pregnancy / LMP / vaginal bleeding DVT Warfarin / Blood thinners Decision Causes / Complications Conclusion Offer brochures, support. If you have time: SAD / PMH  Analysis: History: ─ o o o o o o ─  Why are you doing this INR? When were you diagnosed? How? Were you admitted through the ER or outpatient? Was there any involvement of your lungs? Which medications were you taking? Do you measure your INR regularly? When was the last time? What was the result? What is your target INR? Give the information: Your measurement today shows INR of 1, any idea why? o Compliance: Are you still taking your warfarin? On regular basis? Did you stop your medication? Why? o Forget: Do you take your medications on your own, or does someone else help you? Any chance that you missed a dose? o New medications: Did you start a new medication? What? Why? When? o Diet: Do you eat a lot of spinach? Or dark green vegetables? (rich in vit K) Impact: Now, I would like to ask you some questions to check if you have relapse of your DVT or bleeding, then we will go from there ─ DVT relapse: Because you stopped your medication, I would like to make sure that there is no relapse o DVT: Have you had any pain / swelling / redness in your calf muscles? o PE: Have you had any SOB, chest pain, heart racing? o Stroke: Any confusion? Vision changes? Difficulty finding words? Weakness? ─ Bleeding: o Did you notice any bleeding? o Did you notice bleeding from your gums / nose / coughing or vomiting blood / bruises in your body / dark urine / urine in stools? o Any weakness / numbness / difficulty finding words / vision difficulty? o Did any one tell you that you look pale? Do you feel fatigued? Based on what you have told me, there are no obvious serious consequences, if it is ok with you, we can discuss your situation now!  Red flags: for FEMALES Are you taking any OCPs? OCPs might increase the risk of developing DVT. Are you pregnant? Warfarin is not to be used during pregnancy; we will use heparin instead of it. Have you had any vaginal bleeding? ─ ─ ─ OSCE-guide-III.doc Page 57 of 255 History taking – Medicine  What is your understanding about DVT? Causes: o It might happen after prolonged sitting without movements (like very long flights) o Or due to certain medical condition, The blood tends to form clots in the deep veins of the lower extremities ─ Complication: o Relapse 8%: without treatment, and that is concerning!  Whenever we treat the patient, our target is to decrease the relapse rate to 0.8% which is 1/10 of the risk without treatment o These clots are not fixed, and sometimes they get dislodged from your leg and travel along your blood vessels, all the way to the lungs (chances are 3%):  If large enough  might cause sudden death  If showers of small clots  you may not feel anything right now, but it later will cause what we call “pulmonary HTN”, which is a debilitating disease, with serious consequences and we do not have treatment for it right now o Always in medicine, we try to balance the benefits and the side effects, and in this condition, the benefits largely outweigh the risks. ─  Now, what do you know about blood thinners? o It is a medication used to make our blood thin, preventing our body from forming clots by competing with vitamin K, which is needed for the formation of the elements of blood clots. o We take warfarin seriously, and that is why we monitor it closely and regularly, by assessing the INR which is an indicator of the effect of warfarin. Therefore, as long as your INR is within your target, the risk of bleeding is less than 1%, and almost near 0% to have intra-cranial (brain) hemorrhage without having external bleeding first. That is why you need to keep monitoring yourself, and seek medical attention if you notice any signs of bleeding.  Decision: If the patient decides that he will restart the treatment: ─ We will do it the same way as we did the first time: o We will start heparin and warfarin together then stop heparin after 3 days ─ We will need to measure the INR daily (till we reach our target) then twice a week, then weekly, then every 2-4 weeks NOTES: ─ Numbers to remember: o Relapse (recurrence) of the DVT: 8% without treatment and 0.8% with treatment. o Possibility of DVT  clots and PE: 3% o Chances of having bleeding with warfarin: 1%, and almost near 0% chance of having intra-cranial bleeding without having an extra-cranial bleeding. ─ The initial DVT counselling should have been done in the first time, when the patient was diagnosed; which includes: o General knowledge about DVT o Causes and risk factors o INR follow up ─ My best friend was taking warfarin, and he had brain hemorrhage! o I am sorry to hear that, this must be stressing / worrying, especially that you are taking the same medication and he is a close friend to you. o We prescribe warfarin for many reasons, the issue here is that your friend was not my patient, and I do not know about his condition, so I am not in a position to comment on this situation. o I am glad you came here today, so that we can discuss this together. OSCE-guide-III.doc Page 58 of 255 History taking – Medicine Patient is receiving blood – counsel for adverse reactions You were called to assess a patient who is receiving blood, and the nurse has concerns. Adverse effects of blood transfusion:  Febrile reaction: most common / not serious  Anaphylactic reaction: not common / serious  Haemolytic reaction: not common / serious Introduction to nurse / what is your concerns / ethical challenge Introduction to patient / ethical challenge  ABCD  History  Brief physical exam Adverse reactions of blood transfusion Plan  Introduction to nurse / what is your concerns / ethical challenge Good afternoon, I am Dr … May I get your name please? How can I help you? OR What are your concerns? o There is a mistake! ─ What do you mean by mistake? How is the patient doing? o This was wrong blood! ─ Did you stop the transfusion? o Yes ─ That is great, this is the first step in the right direction ─ Now, what do you mean by “wrong blood”? Is it the same bld group or no? Cross matched or not? Do we have the patient name on the units? o It is the same group but with other patient name o Doctor, please do not tell the patient! ─ Why? Do you have any concerns? o I might be fired! ─ I see you have concerns here, but we need to stabilize the patient first. Then we will speak about that. However, we need to investigate before making decisions. ─ Can you tell me when did this happen? How much did he receive? ─ ─  Introduction to patient / ethical challenge Good afternoon Mr …, I am Dr … The nurse was updating me about your condition. It looks like there was an unintentional medical error took place, and I need to make sure you are ok. o Is it serious doctor? ─ Could be! There are different possibilities; I need first to check you. o Whose mistake is this? Is it the nurse mistake? ─ Usually in the blood transfusion process, there are many steps; any one of those might go wrong. It is early now to judge. I need first to make sure you are ok and stable, and then I will file an incidence report. Investigations will be done, and you will be informed with the results. ─  ABCD Let us make sure you are safe and stable first. AB: ─ Can you please open your mouth? Mouth is clear with no swelling. Do you have any itchiness or swelling in your mouth? ─ Trachea is central, no engorged jugular veins. Can I listen to your heart please! Normal heart sounds. OSCE-guide-III.doc Page 59 of 255 History taking – Medicine C: ─ ─ ─ Can I know the vitals please? Normal / stable. Can you remove the blood unit please, and send it to the blood bank. We need to re-cross this patient blood with this unit. Can you put another IV line please! We need to take samples for: CBC / differential / lytes / blood grouping and re-crossing / haptoglobin / bilirubin level D:   ─ ─ ─ ─ I am going to shine light in your eyes! Can you hold my fingers please? Do not let go. Do you feel me touching you? Patient is grossly neurologically free. ─ ─ If fever: give 2 tablets Tylenol 325 mg Can you please prepare: o Allergic reaction: Benadryl o Anaphylactic reaction: Benadryl / Epinephrine / Steroids o Haemolytic reaction: Diuretics and fluids History Now, I would like to ask you some questions: ─ Why are you taking blood? They have found that I have anemia ─ Did you take blood before? Or is this the first time? ─ Do you feel warm? Shivering? Chills? ─ Do you feel any itching or swelling in your lips / mouth? ─ Any heart racing? SOB? Wheezes? Dizziness? ─ Any flank pain? Back pain? Weakness? Brief physical exam No IV line oozing / No hives on skin / No mouth swelling Listen to heart / lungs  clear Press on flanks / spine  no tenderness ─ ─ ─  Adverse reactions of blood transfusion Blood transfusion is a commonly used procedure, and it is life saving. A lot of precautions are taken to make sure it is completed safely. However, like any other medical intervention, it has some side effects. ─ The most common reaction that might happen is called “febrile reaction”. This is not serious reaction and it is self limited. It might happen again, so if it happens, next time we give you Tylenol before the transfusion. ─ Another adverse effect, which is less common but more serious, it is called “anaphylactic reaction”. This is a form of severe and serious sensitivity reaction, in which the blood pressure drops suddenly, and there is a swelling of the tongue, lips, and mouth, with difficult breathing. We do not have a method to predict it. However, based on your symptoms, your physical exam and vital signs it is less likely you have that. ─ The third adverse effect is called “hemolytic reaction”, and it happens if the patient receives blood that belongs to another blood group. It causes damage to blood cells which leads to back pain and flank pain, and could have serious consequences. Again, based on your symptoms, physical exam and vital signs, your condition does not cope with this reaction too. And the fact that you received blood from the same blood group makes it less likely you will have hemolytic reaction. ─ We prepared medications to deal with any reaction and we will keep you for a while to monitor you, to make sure that will not happen. ─  Plan Call the blood bank to withhold the other units (previously cross-matched) File an incident report ─ ─ OSCE-guide-III.doc Page 60 of 255 History taking – Medicine Counselling – Ventilator Mr Johnson is 75 years old gentleman, his life-long wife for 50 years has a terminal COPD, with severe pneumonia, and she is on ventilator for the last 3 weeks, and it is not possible to wean her from ventilator, you called him to inform him about the condition.    What do you know about your wife’s condition? ─ Listen carefully ─ Show understanding and empathy Give information about her condition, ─ Give clear simple information ─ Stress on the progressive, irreversible nature of the disease Give alternatives: Remain on ventilator, with no evidence that she will be able to breath by own, and with the possibilities of fatal complications like infections, bed sores, … Some people does not like to have this quality of life ─ Stop the ventilator and she will pass away in peace As regarding her condition now, have you ever discussed this with her? Has she ever expressed her wishes about what would she like to be done to her if she needs to be resuscitated or put on ventilator? Does she have any advance directives or living will? What do you think about this now? ─     Offer time if he needs to discuss it with other close family members, or if he needs to arrange any thing (e.g. I am just giving you information, and we can arrange a meeting with the family within 2 days so that I can explain to them). What if she does not want to be on ventilator but he would like to leave her on the ventilator? Mr Johnson, I am sorry to tell you that, actually it is not our decision or your decision, it is her choice. And she expressed her wishes before; she decided that she does not want to have this poor quality of life. We have to respect her wishes. ─ OSCE-guide-III.doc Page 61 of 255 o What if this patient broke his leg. and still insists to leave the hospital! o I would like to make sure he is competent. I would like first to know who has the legal custody (guardian) of this child. but he developed delirium post-operative and now he wants to discontinue his medications  NO. and to rule out suicidal ideation o I would explain to the patient: diagnosis / treatment / side effects of treatment / complications of not receiving treatment / alternatives o I will document this. incompetent to change decisions.doc Page 62 of 255 . o You have a case of patient. and he/she has the right to take all his/her medical data and file If you want to terminate a patient from not seeing you as family physician: o Give proper notice period o See him/her for emergency Confidentiality. do you want to operate him without consent? This is a new condition. a social worker (case manager) … Any unconscious patient  ask for DNR or advanced directives MMS exam score < 24  patient is incompetent. who is adopted by another family! o In order to determine whether I should release any information or no. who had surgery. he is delirious. e. and I will let him go Biological parent wants to know the medical details of his/her son. is taking medications. patient has just had a heart attack. he already consented to take the medications before he entered this delirium. not under influence of alcohol or any substance. when to break confidentiality? To report for the ministry of transportation for example: o Dementia / delirium o Vision problems o Seizure disorders o Schizophrenia (case-based) o Heart attack  1 month not allowed to drive o Alcoholic with liver failure (based on Child’s criteria: albumin / ascites / INR / bilirubin) Report for child safety  CAS (Children Aid Society) o Even if POTENTIAL or SUSPECTED o Child neglect / abuse Patient wants to leave hospital against medical advice. It might be the adopting father. and I will ask the patient to sign a LAMA (leaving against medical advice).g. we do not know what would be his competent wishes  look for SDM (substitute decision maker).History taking – Medicine Ethical questions          Patient has the right to access his/her medical file. we can not withhold it Patient wants to leave you as family physician  it is his right. OSCE-guide-III. will not die tomorrow or so. sure about the result. Be sensitive. ─ What have you discussed last time? ─ Why did you ask for the test last time? ─ Did you feel sick in any way? ─ Was there anything made you worried about your own health?  Give the test result: It is positive.History taking – Medicine HIV post-test counselling ─ ─ Usually you are covering for other physician to give the test result — which means this is a new pt to you. I just need to understand the situation here. Check with patient’s reaction: normalize patient’s feeling. years ─ ─  Consequences of HIV: Repeated infections / LNs Tired / fatigue Memory – dementia Depression ─ ─ ─ ─  Causes of HIV: SAD – shared needles Sexual: o Risky behaviour o Confidentiality – how to inform the partner?  Get the background info: duration of the relationship.  Partner has to know: Risk of infection / Needs to be tested  Will know anyway. offer help to tell. inquire about support system. ─ ─ Case: HIV patient with diplopia  cranial nerve examination ─ CT brain: enhanced multiple rings  toxoplasmosis with HIV ─ DD: TB / toxoplasmosis / CMV / CNS lymphoma ─ Management: refer to infectious disease specialist OSCE-guide-III. I am covering for him/her.doc Page 63 of 255 . is away. because they do two tests before giving the positive result. ─ Education: emphasize the importance of safe sex: advice use barrier contraceptive methods all the time with all partners in the future to prevent the transmission. all the feelings you are having now are very normal. either from public health or him. no suicidal or homicidal ideation. Prefer him to tell. empathetic. ─ ─  Before discharge the patient: Arrange follow-up visit in the next couple days after patient digests the info Make sure that patient is safe to go home. ─ What kind of thoughts are going through in your mind? What concerns you the most right now? ─ AIDS: don’t have AIDS. how close to each other. Prognosis is variable. but many people carry it without feeling it for quite long time. and I have your file with me. and flexible  Introduction: Your Dr. safe to drive back. re-biopsy or resect will depend on the level of suspicion o PET scan not yet routine but can help distinguish benign from malignant nodules • Watchful waiting: repeat CXR and/or CT scan at 3. or Lung cancer ─ ─ ─ ─ ─ ─ ─ ─  Management: Investigations o CXR: always compare with previous CXR o CT densitometry and contrast enhanced CT of the thorax • Sputum cytology / stains • TB skin test o Biopsy: bronchoscopic or percutaneous(CT-guided) or excision (thoracoscopy or thoracotomy): if clinical and radiographic features do not help distinguish between benign or malignant lesion  If at risk for lung cancer. phlegm. SOB.B.B.B. Definition: a round or oval.: Contact with sick person (T.doc Page 64 of 255 . chills. size up to 3-4 cm. 12 months ─ Algorithm: Evaluation of a Solitary Pulmonary Nodule. which may or may not be calcified. weight loss. skin rash History of lung disease History of cancer HIV status Family History of T. Can be benign or malignant ─ Any ideas about what could be causing this nodule ─ ─   Consequences: Local symptoms: cough. biopsy may be performed regardless of radiographic features  If a biopsy is non-diagnostic. night sweat / change of appetite. and is surrounded by normal lung. fatigue / pumps or lumps in the neck or elsewhere in the body ─ Impact / screen for metastasis: o Brain: headache/ vomiting? o Back: back pain/ weakness/ numbness/ tingling in arms or legs? o Lungs: chest pain/ cough/ phlegm/ wheezes/ heart racing? o Liver: yellow discoloration/ itching/ urine/ stools? ─ ─  Causes: Smoking Exposure to chemicals / smokes at work T.B. skin test Sarcoidosis: associated symptoms. sharply circumscribed radiographic lesion. check previous CXR o Looks benign or unchanged  repeat CXR q 3-6 months for 2 years o Significant risk factor on history or looks malignant or changed  CT chest  Cause (infection or cancer)  stage and treat  Calcification  observe  No diagnosis  trans-thoracic needle biopsy ─ Inflammatory  treat the cause ─ Cancer  stage and treat ─ Still NO diagnosis  resect for diagnosis ─ OSCE-guide-III.) / Recent travel / T. 6. whether to observe. wheezing Constitutional symptoms: fever.History taking – Medicine Lung Nodule  Introduction: Why X-ray was taken? When? When was last normal X-ray? Do we have it? Give the test result: ─ Solitary Lung Nodule. joint pain. haemoptysis. B.doc Page 65 of 255 . or Lung cancer Social History Evaluation of a Solitary Pulmonary Nodule: OSCE-guide-III.History taking – Medicine    Past Medical History Family history: of T. B with exertion ─ Swelling (ankle.History taking – Medicine High Creatinine  Introduction: Why the test was done? ─ When was the last normal test? ─ Any idea about the meaning of the test ─  Give the test result: ─  Consequences: manifestations of renal failure Nausea and vomiting / stomach pain ─ Itching ─ Pallor ─ Fatigue ─ S. around eye) ─ Bone pain ─  Causes: Renal: ─ Hypertension Diabetes o Repeated kidneys infection o Poly-cystic kidneys o Medications: NSAIDs / gold / penicellamine / ACEIs Post-renal: o Kidney stones o Bladder cancer o Prostate problem o o ─    Past Medical History: o Kidney disease o Previous hospitalization o Nephrectomy o Allergies Family history: of renal problems Social History o Smoking o Alcohol o Drugs o Work o Home o Support OSCE-guide-III.doc Page 66 of 255 .O. many organic causes are irreversible. encompassing organic & psychogenic causes. o Low testosterone: changes in secondary sex characteristics. autonomic pathways and/or blood flow. with a broad DD. … ─    Counselling: ─ Normalize patient feelings ─ ED can often be improved with: o Life style modifications: exercise / weight loss / improved diet / DM control / smoking cessation / ↓ alcohol / stress management / ↓ anxiety / sleep hygiene o Improvement of patient relationship with partner: marital counselling / address sexual boredom / refer to specialist in sexual education and therapy ─ Unfortunately.g. numbness / history of MS. e. hormonal treatment. does function vary depending on the setting? Partner / Place / Time? Consequences: How does this affect your life? Your relationship? Causes: ─ Many endocrine disorders and systemic diseases cause ED by influencing libido. socially) / anxiety attacks? Any stress? Past lifebackground. Secondary o Chronology (frequency. HTN. vascular and emotional factors. opioids. ─ Organic causes: o Medical causes: history of DM. intermittent claudication o Neuro: back trauma / constitutional symptoms (back metastasis) / back pain / weakness. but we have treatment options: o Testosterone preparations (if low testosterone) o Viagra or Cialis o Penile self-injection o Vacuum – rubber ring device o Penile prosthesis ─ Follow-up appointment for BOTH partners OSCE-guide-III. hormonal.doc Page 67 of 255 . work. course)  Onset: acute (more likely psychogenic) or gradual (organic)?  Course: intermittent (more likely psychogenic)? Libido affected? o Severity or amount? All the time? o Aggravating / precipitating and alleviating factors ─ Organic vs. Confidentiality. duration.History taking – Medicine Impotence / Erectile Dysfunction  Introduction: ED is a common problem in men. hair pattern changes / history of gynecomastia / galactorrhea / history of thyroid disease / pituitary disease ( visual defect. peripheral vascular disease. anti-depressants. This is often a difficult topic for men to discuss with their doctor. onset. upbringing. e. Psychogenic o Do you have early morning erection? o Do you have night time emissions? o Do you have desire? o Are you able to masturbate to an erection or climax? o Situational dysfunction. headache) o Medications. hyperlipidemia. Analysis of the CC: ─ Primary vs. MAO inhibitors o SAD: smoking / alcohol / recreational drugs ─ Psychogenic causes: o Any problems with their partner(s) o History / screening of depression o Any recent changes in life (home.g. ─ Penile erection is a multi-factorial process dependent on integration of neurologic. fever. hypothyroidism) ─ Septic arthritis: usually mono-arthritis. better with use. hypomagnesemia. inflammatory bowel disease. oligo-arthritis (4 or less). pleuritis. erythema nodosum. conduction defects o GIT: GERD. hot). sclerodactyly. pulmonary nodules o Cardiac: pericarditis. onycholysis. chronically can be poly-arthritis: gout (tophi. psoriatic arthritis. with tenosynovitis and skin pustules ─  Disability and adaptation How does it affect your life? Ask about daily activity! ─ Effects on ability to work! Support! ─ OSCE-guide-III.doc Page 68 of 255 . scleritis. enteropathic arthropathy. conjunctivitis. scleroderma. telangiectasias. malabsorption. Gonococcal arthritis can be migratory. nodules. can have some stiffness but usually not prolonged ─ Joint swelling / redness ─ Other Joints / Pattern of joint involvement: o Mono-arthritis. SI joints) o Small joints (hands / feet) versus large joints (hips / shoulders) o Additive joints vs. pulmonary fibrosis. inflammatory. RA. hot. poly-arthritis (5 or more) o Symmetric vs. drugs) o CPPD (hyperparathyroidism. Sjogren’s. migratory joints o Tendon involvement ─ Constitutional symptoms ─ Extra-articular features: Seropositive (e. alcohol history. SLE. worse at end of day. pyoderma gangrenosum ─ Crystal arthropathies o Mono-arthritis (red. hemochromatosis. red. constitutional symptoms o Non-inflammatory: worse with use. alopecia. Wilson’s disease. reactive arthritis) o Eyes: iritis. asymmetric o Peripheral joints versus axial involvement (spine. renal failure.g. dry eyes o Oral ulcers o Respiratory: pleural effusion. psoriasis. nail pitting. discoid. bloody diarrhea o Dermatology: malar rash. calcinosis. periungal erythema.History taking – Medicine Rheumatology – History Taking   Chief complaint: pain / stiffness / weakness / deformity / limitation of movement / joint clicking ─ OCD / Acute (< 6 weeks) versus chronic (> 6 weeks) ─ PQRST / ↑↓ Associated Symptoms: Morning stiffness o Inflammation: morning stiffness (>30 min). pericardial effusion. myositis) Seronegative (Ankylosing spondylitis. loss of balance. sensory deficits.History taking – Medicine Multiple Sclerosis Middle aged man (or woman) with episodes of numbness in one leg. diplopia. ─ History of headache ─ Sarcoidosis ─ HIV status ─ Systemic inflammatory disease ─  Family History: MS. weakness NYD. arthralgia. ─ History of weakness: MS can mimic ANY neurological disease process. dry mouth ─ Numbness in upper or lower limbs ─ History of previous vertigo. SLE. urgency. symmetric vs. dysarthria ─ Dry eyes. depression. behavioural changes ─ Constitutional symptoms ─ Impact on daily functioning ─ ─ ─  Lhermitte's sign: electrical sensation down back on neck flexion Uhthoff's phenomenon: worsening of symptoms in heat (hot bath. o Determine which groups of muscles are involved: proximal vs. ─ History: Review of systems ─ Diagnosis: MS ─ Investigations: MRI / CSF  History of present illness (neurological screening) Ethnic background. Asymmetric o Pattern of weakness: stepwise decline. relapse and remitting ─ Dysphagia.doc Page 69 of 255 . incontinence) ─ Fatigue. falls ─ Any associated sensory symptoms ─ Bowel or bladder dysfunction (difficulty fully emptying bladder. summer) Past Medical History Previous transient focal neurological deficits: vertigo. loss of vision (optic neuritis). etc. Distal. visual deficits NYD OSCE-guide-III. ─ Clumsiness. how much. o Caloric intake should be calculated /+/ does not exceed 1800 Cal/d o I will give you tables and graphs to show you the ideal meal composition. Educational information: group program / booklet and brochures Follow up on regular basis OSCE-guide-III. disabilities. what caused failure o Assistance: dietition. bedtime meals. social activities involvement o Psychological assessment: embarrassment. anxiety ─ Diet: detailed history. it is important for your general health. vegetables and fruits. but generally. Methods: ─ Set up a goal first. functional impairment. lunch and supper must be formed of: 50% vegetables and fruits /+/ 25% protein /+/ 25% carbohydrates ─ Exercises: o Program: 3-5 times per week /+/ 30-50 min each time o Set up personal instructor to guide ─ Medications: locally to absorb fats or centrally working on the satiety centre. ─ Exercises: how often.doc Page 70 of 255 . low carbohydrate. medically vs. do not like to start with ─ Surgical procedures. ─ ─ ─ Avoid: smoking / alcohol /+/ Healthful life style. low self esteem. IHD o DM. start slowly ─ Diet: can refer you to a dietition o Type of food: high fibre. cholesterol o Sleep apnea. Social history: SAD. surgically o What weight achieved. we can discuss it later. it is very difficult process. asthma o OA. type    Past medical history: AMPLE. depression. very common multiple tries. social activities. less fat/cholesterol. home and work environments Family history: obesity in the family Counselling:   Encouragement: admire patient. requires a lot of effort.History taking – Medicine Obesity  Analysis of chief complaint: ─ Weight analysis: now and how about one year ago? Maximum and minimum weights? ─ Weight loss history: o Attempts. in very advanced cases and there is medical impairment. how many meals. exercises programs o Explore why patient want to do it again o Admire and encourage the pt ─ General health now: o HTN. Psychiatric (conversion. o Lumbar puncture for CSF examination has a role in the patient with obtundation or in patients in whom meningitis or encephalitis is suspected. Metabolic (hypoglycemia). Vascular (TIAs). They are neuro-imaging evaluation (MRI or CT) and electroencephalography (EEG).g. Regarding morbidity.doc Page 71 of 255 . Some people lose consciousness. if therapeutic level but side effects or poor seizure control  add another drug (carbamazepine / valproic acid) OSCE-guide-III. malingering) ─ Prognosis: The patient's prognosis for disability and for a recurrence of epileptic seizures depends on the type of epileptic seizure and the epileptic syndrome in question. Most deaths are accidental due to impaired consciousness ─  Plan: Diagnostic workup ─ Patient education ─ Treatment ─ Pregnancy ─ ─ Diagnostic workup: o Two imaging studies must be performed after a seizure. Usually. phenytoin). panic attacks. but it tends to run in the family (idiopathic) or structural brain damage (post-meningitis) ─ Other conditions that should be considered include: Syncope (arrhythmias). causing abnormal body movements. Regarding mortality. seizures cause death in a small proportion of individuals. it does not cause learning disability or brain damage ─ The diagnosis of epilepsy requires the occurrence of at least 2 unprovoked seizures 24 hours apart ─ Most of patients have no clear reason to explain why they are having this. trauma is not uncommon. and some do not. o Metabolic screen o Serum studies of anticonvulsant agents (e.History taking – Medicine Epilepsy Counselling  Young 16 yr old male for driving license counsel Introduction Analyze epilepsy history Triggers MOAPS HEAD SSS  Why does the patient want a note from doctor for a driver’s license? Usually Dr does not give such note unless there is underlying condition!  Age of onset? / When was the diagnosis? / What was the diagnosis?  How frequently do the attacks occur?  How long does each attack last? ± LOC  Aura prior to attack?  When was last attack? Similar to previous ones?  What happens during an attack? Does the patient shake / all over / partly / roll up eyes/ bite tongue?  How do you regain consciousness / how do you feel after the attack  Which medication does the patient take? Compliance? When was the drug level checked?  Any other medications that might interact with epileptic drugs?  Sleep deprivation / Long screen time before sleep?  Alcohol? Stimulants?  Are you under stress Scan the mood and anxiety Home / Education / SAD (do you take stimulants)  Counselling: What is your understanding of seizures? It is a common condition due to increased electrical activity in the brain. History taking – Medicine ─ ─ ─ Patient education: o Dangerous activities: to prevent injury. must be seizures-free for more than 1 year  Diving. so for the time being you need to continue to use your pills and add another method (mechanical) till you contact your gynecologist OSCE-guide-III. Can you postpone the pregnancy for a while? It is better to have good control of seizures for a while. About 75% of relapses after discontinuation occur in the first year. dysarthria). to get any seizure during pregnancy will pose great risk for both of you and baby. mountain climbing  Taking unsupervised baths. sleep deprivation). educate patients about seizure precautions. females to take proper contraceptive measures  Patient might choose to wear a bracelet indicating he has epilepsy  If a seizure will happen: go to the ER  Regular follow-up visits and monitoring of anti-convulsion level in blood Treatment: o The mainstay of therapy for people with recurrent unprovoked seizures is an anticonvulsant.  Forgetting to take medication on time  Taking other medications that interact with the treatment o Life style:  You have to take the treatment almost for your whole life  Talk with your physician about any new medication you want to take  Medications are teratogenic. will start with one medication. we may increase the dose and/or add another drug o Side effects of medications: movement disorders (ataxia. patients to observe strict seizure precautions (including not driving) during tapering and for at least 3 months after discontinuation. liver. swimming. administration of an anticonvulsant is recommended. o Avoid the triggers for seizure attack:  Alcohol will exacerbate (chronic alcohol: ↓ blood level of anti-epileptics due to ↑ metabolism / excess alcohol: ↓ seizure threshold)  Stress. and at least 50% of patients who have another seizure do so in the first 3 months. with open door  Working at significant heights. no anticonvulsants are recommended unless the patient has risk factors for recurrence o Medications will be taken for long term. operating machines and the use of fire and power tools. And the medications can cause serious malformation to the baby o Yes is it OCPs? Yes! There might be drug interaction. If a patient has had more than 1 seizure. teratogenic. However. physicians consider discontinuing the medication. standard of care for a single. better take shower not bath. there are many options. hiking. poor concentration o Discontinuation: After a person has been seizure free for typically 2-5 years. Most accidents occur when patients have impaired consciousness. alcohol. if no full control. if the patient is having stress / anxiety / alcohol issues: counsel and offer social support  Sleep deprivation / long screen time before sleep  Head trauma. Therefore. Authors recommend that anticonvulsants be gradually discontinued over 10 weeks Pregnancy:  Are you sexually active?  Do you take use contraception? o No  Are you planning to get pregnant? Yes! Let us talk about pregnancy and the meds you will start.g. unprovoked seizure is avoidance of typical precipitants (e.doc Page 72 of 255 . Restrictions apply on:  Driving (report to ministry of transportation). kidney. drowsiness. Mention you believe the professor would be reasonable. OSCE-guide-III.doc Page 73 of 255 . Analyze the case  headache case  full history 3. and apologize that you can’t lie or write a note for the pt you didn’t see at that time. reason for the note? 2. What type of doctor’s note pt wants. but can give copy of today’s visit and notes about what happened.  Mention you are willing to do any thing to help him out. Deal with the patient request:  Mention that patient is probably suffering from migraine or tension headache.  Encourage the patient to talk to the tutor (or supervisor) frankly about what happened. ask about any possibility to make up the class he missed.History taking – Medicine Medical note   For past date! For headache 1.  Mention that you can’t write note for previous visit.  Mention you are happy to see this patient again to see how things going after he talks to supervisor and for follow-up regarding his headache. kidney.5 or TC/HDL < 4 BP < 130/80 OSCE-guide-III. It also shows increased risk of you having complications in the large blood vessels causing heart diseases. avoid saturated fats and simple sugars. lack of exercises.History taking – Medicine Pre-diabetes – Counselling  What is DM?  Fasting blood sugar (FBS) > 7 mmol/L Random blood sugar (RBS) > 10 mmol/L + symptoms Glucose tolerance test (GTT) > 11. ECG). strokes and peripheral vessel diseases Diabetes: ─ Increase of blood sugar in our blood due to deficient or ineffective insulin. alcohol. exercising (30 -45 min of moderate exercise for 4-5 days/wk) and life style changes (limit Na. seizures Complications of  Micro-vascular: nephropathy / neuropathy / retinopathy high blood sugar  Macro-vascular: CAD / peripheral arterial dis / impotence  Lifestyle: too much simple sugars.  I strongly recommend you to work on lowering your chance of having diabetes by half by: watching your diet (healthy balanced diet. fatigue. hospitalization FH DM in first degree relatives SH  Sexual function: any concerns  Smoking  From the conversation we had. drink more. but it shows that you have an increased chance of having it. ─ Explain the role of insulin in helping cells to utilize glucose. so let me ask few questions. pee more even at night Blurred vision Tired / weight loss Yeast infections. ─ With one reading we can not say that you are prone or have DM. CVD. LOC Symptoms of Ketoacidosis Symptoms of If patient is on insulin: sweating. family history  Medications: steroids / beta blockers (β-blockers are contraindicated in DM: it causes hyperglycemia / and it masks hypoglycemia) PMH Medications: used long term steroids.1 mmol/L What is pre-diabetes? Impaired glucose tolerance o Fasting blood sugar (FBS) 6. stop smoking). about 1–5% per year. it looks like you are likely to get DM.doc Page 74 of 255 .9 mmol/L o Glucose tolerance test (GTT) 7. to see if you have the symptoms of DM!  Symptoms of hyperglycemia     Eat more.. Hb A1C – which shows your blood sugar level over the past 3 months. Triglycerides < 1. However I am going to examine you and do blood tests (FBS. shaking.  Treatment targets: Hb A1C < 7 FBS 4 – 6 Lipids: LDL < 2. palpitation. CAD. in the toes and finger webs?  Do your wounds get long time to heal? N/V. abdominal pain.8 – 11 mmol/L o o o Introduction   Impact Red flags Pre-diabetes: does not mean that you have diabetes.1 – 6. HTN. are there itching / rashes in your groins. type I and type II. dehydration.  I can refer you to diabetes educational program if you wish. phenytoin. clozapine or other antipsychotics. two types of DM. thiazides. Cholesterol. caffeine. overweight. micro albumin / Cr ratio. hypoglycaemia headache. confusion. choose low glycemic content foods). lipid profile. Emergency Medicine Emergency Medicine OSCE-guide-III.doc Page 75 of 255 . primary survey was done.V Fib .Any trauma (kicked.Advanced directive Non cardiac - Acute abdomen for 24 hrs. …) .doc Medical I A B C D OCD PQRST ↑↓ Associated symptoms Risk factors PMH Focused physical exam Management Page 76 of 255 . thrown from height. fix with gauze Non-trauma Cardiac Chest pain: . ↑HR Upper / lower GIT bleeding Severe headache for 2 hours Seizure for the last 20 minutes 16 years brought unconscious to the ER Patient receiving blood.STEMI: catheterization.DNR . do the secondary survey N. ↓BP. counsel him Management: Trauma I A B C D AMPLE Head to toe Management OSCE-guide-III.B. thrombolytics .V Tachy ACLS: Advanced Cardiac Life Support code Heart block: old patient . angiography Arrhythmias: .Manage over the phone: trauma or meningitis . car accident.Emergency Medicine Emergency Room Trauma ATLS: Advanced Trauma Life Support protocol Primary survey: .MONA ± β-blockers .No ST elevation: heparin. if knife: leave it in place.Secondary survey: patient in the ER after car accident. nurse can you please fix his head. and I will give pain killer as soon as I can. I would like to activate the ATLS protocol . or fix pt head and remove anterior part: ─ Trachea ─ Jugular veins (JV) Listen to lungs Listen to heart sounds OSCE-guide-III. normal heart sounds (HS) S1 and S2 Trachea JV Air HS Diagnosis entry shifted Engorged ↓ same normal tension away side pneumothorax shifted depleted ↓ same normal Hemosame side thorax side central engorged bilatera muffle cardiac l d temponad e Usually no cardiac temponade in the exam Page 77 of 255 . we need to put neck collar - I understand that you are here because you had a car accident How are you feeling / doing right now? o I would like to make sure that you are stable. o I can see that you are in a lot of pain. the physician in charge in the emergency room. please bear with me for few minutes. no vomitus Pt is talking to me that means airways are patent Can you give him O2 – 4 L with a nasal canula Any change in saturation? Can you plz let me know if any change in saturation happens! By inspection. no paradoxical movements of the chest. I will check with the nurse and we will start the management then I will be asking you more questions. heart Can you please open your mouth? Nurse.Because it is a trauma case. no dentures. goggles and masks - Mr … I am Dr …. chest is symmetrical.doc Mouth is clear. no FB. bilateral air entry. meanwhile my first concern is to make sure you are stable We will start the primary survey now: Airways / Breathing Mouth / O2 / inspect chest. gowns. I will look for her and I will get back to you as soon as I can. proceed o Pt is not wearing a neck collar: Mr … please do not move your head. where is my wife? How is she doing? Was she with you? I can see that you are concerned about your wife.Emergency Medicine Trauma I: introduction: . no open wounds. gloves. o Doctor. plz? Inspect the chest Open the collar window. no use of accessory muscles for breathing Trachea is central. o If the patient is wearing a neck collar.I would like also to get protection for me and my team. no bruises. what is O2 saturation. neck / listen to lungs. JV not engorged. I would like: ─ To get stat surgery consult ─ To arrange for FAST (focused abdominal sonogram for trauma) ─ To do DPL (peritoneal lavage) I am going to press on your pelvis  Press from the sides  Press open book  If positive. monitor.doc Page 78 of 255 .Emergency Medicine + If ↓↓ BP and ↑↑ HR / other signs of tension pneumothorax  nurse. ─ Check the pulses ─ Thomas splint ─ Stat orthopedics consult Log rolling I need more team members to roll the patient on his left side:  To check for external source of bleeding  To press on the spinal processes  To perform digital rectal exam OSCE-guide-III. pt is stable / unstable I would like to have two large IV lines./ tension. patient lower extremities are symmetrical. I am suspecting pelvic fracture: ─ Cut pt sheet and wrap around the pelvis to support. after the bolus fluid is done and every 5-10 minutes or if there is a change in the vitals Look for the source of bleeding Abdomen:  Inspect the abdomen  bruises I am going to look at and feel your  Palpate the abdomen  rigidity and guarding abdomen  If positive. 16 G in both anti-cubital fossae: ─ One to start fluids: bolus 2 L ringer lactate or normal saline ─ The other line is to withdraw samples for: CBC/differential/lytes /+/ blood grouping and cross matching / and prepare 6 units of blood (4 matched and 2 “O”) /+/ stat glucose /+/ INR/PTT/LFT /+/ Bun/creatinine /+/ toxic screen/alcohol level /+/ continuous cardiac monitoring/cardiac enzymes and ECG Can you please inform me with the vitals. ─ What is the amount of blood? ─ If > 1. I am suspecting intra-abdominal bleeding. ─ Is there any gush of air? ─ Check the trachea centrality and air entry ─ We need to put a chest tube in the 5th intercostal space + If ↓ BP and ↑ HR / other signs of hemothorax  nurse.5 L  stat surgery ─ Otherwise. no deviation  If positive: I am suspecting femur fracture. I need to put chest tube in the 5th intercostal space at anterior Axillary line. and check blood on penile meatus ─ Stat orthopedics consult Lower extremities By inspection. if > 200 ml/hr  surgery Circulation Vital signs / fluids / withdraw blood samples / look for source of bleeding Can I get the vital signs please Comment. patient is … hypo. no pain. no abnormal posture or deformity./ hyper. comment on HR. I need to put a large needle (16 / 14 G) in the 2nd intercostal space at MCL (upper border of the 3rd rib). No inequality in length. do not let them go Can you wiggle your toes? Do you feel me touching you here. any history of HTN. chest. stroke. symmetrical and reactive Patient is gross neurologically free Alert Verbal 4 3 Pain 2 Unresponsiveness 1 AMPLE A Do you have any allergies? M Do you take any medications on regular basis? PMH. looking for fractures. we are waiting for (surgeon.Emergency Medicine I would like to get trauma X-ray series: for neck. DM. heart attack. and here Glasgow coma scale – eyes AVPU D3: Drugs Pupils are round.doc Page 79 of 255 . LSS and pelvis D: D1: Deficits / Disability D2: Detoxification Neuro screen / I am going to shine light in your eyes? Can you please squeeze my fingers. here. any long term disease P L  Last meal  Last tetanus shot  LMP E Event:  Can you describe to me want happened?  Car accident! Were you the driver or passenger / front passenger?  Were you wearing your seat belt?  Did you hit your head? Did you lose your conscious?  Do you remember what happened. before and after the accident? Conclusion: I am suspecting an intra-abdominal bleeding. orthopedics surgeon) to intervene Summary: If you are done  go for secondary survey: Introduction to examiner  Expose the patient Hello  Examine him head to toe. Neck collar more detailed neurological examination Introduction to patient A/B / C / order x-rays / D / AMPLE OSCE-guide-III. Trauma / GIT bleeding: we always start with 2 L bolus o If the patient is stable for the beginning  do not give anything more o It the patient was not stable.5 L bolus o Give epinephrine / steroids / anti-histaminics (Benadryl) Acute abdomen (pancreatitis / DKA): o 1-2 L bolus o Followed by 1 L / hour till the urine output improves Heart attack: o KVO (keep vein open) 100 cc / hour If trauma. ↓ HR with warm extremities  neurogenic shock (spinal cord injury)  give only 2 L of fluids then give vasopressors For the exam:  Tension pneumothorax  Hemothorax  Rupture spleen  Fracture pelvis  Fracture femur OSCE-guide-III.doc Page 80 of 255 . but becomes stable after the first 2L bolus  give maintenance fluids o If patient was not stable. and continue till you find source of bleeding Stable 2 L bolus Stable Give nothing Unstable 2 L bolus Stable Give fluids – for maintenance Unstable 2 L bolus Unstable Start blood transfusion – 2 RBCs Then continue 1 (RBCs) : 3 (NS) - - - Anaphylactic shock: o 0.Emergency Medicine NOTES: FLUIDS: . ↓ BP. and remains unstable  start bld transfusion: 1 unit of packed RBCs for every 3 units of fluids. no deviation of the angle of the mouth o Both eyes are symmetrical. and shine light in it.To check for brain death: I would like to arrange for: dolls eyes. Kernig's sign. . Tap on the shoulder.Inspection: .Check the radial pulse 5.If the neck is cleared (by CT). X) 4.Inspection: o Face is symmetrical. heart to intubation  2 large IV lines.Upper extremities: .Emergency Medicine Unconscious Patient – Neuro 1. symmetrical and reactive  Mr … if you hear me can you please move your eyes up and down  NO locked-in syndrome  D3: universal antidotes: thiamine 100 mg 2.Reflexes: Biceps / Triceps .Mr … I am Dr … I am the physician in charge in the emergency room now. or retinal hemorrhage .Inspection: no abnormal posture or contractures .Lower extremities: . I would like to do nuchal rigidity. III) / Corneal reflex (V. pupillary reaction  both D pupils are round.I would like to do a fundoscopic examination.Reflexes: Knee reflex / Ankle reflex / I would like to do Babinski reflex / Clonus .Tone: check the wrist and elbow in both sides  No cog wheel.Tone: check by leg rolling  rigidity. elevate the knee rapidly  spasticity . no clasp knife spasticity .Reflexes: Pupillary reflex (II.Check dorsalis pedis pulse . no ↑ in BP and no ↓ in HR (as seen IV fluids and to in Cushing triad) withdraw samples  No fever  No abnormal breathing patterns  Mr … I will open your eyes.Cranial nerve exam: excluding the motor and the sensory: .Introduction: .Mr … Mr … if you hear me open your eyes please. listen for patent air way  Give 4L O2 via nasal A  What is the O2 please canula B  Trachea central.doc Page 81 of 255 . no nystagmus . Brudzinski's sign 6. for I would like to get the vital signs please: C  Based on vitals. caloric reflex test OSCE-guide-III. no lead pipe rigidity. to look for disc edema.Glasgow Coma Scale  if < 8  intubate (ask about DNR) 1 2 3 4 5 6 No Pain Voice Spontaneou N/A N/A Eyes response s Words Confused Normal N/A Verbal No sounds Sounds Extension Flexion Withdrawal Localizes Obeys Motor No movements (decerebrat (decorticat commands e response) e response) 3. VII) / Gag reflex: (IX. chest is moving  Monitor O2 for need  Listen to lungs. do you hear me  I would like to activate the ACLS code please / start primary survey  Check the mouth. were you measuring your glucose? EVENT  This morning.doc Page 82 of 255 . the patient will Orient her. you can give only 50 ml/hr to keep vein open (KVO) But if ↑ HR  give 2 L fluids for follow-up 6 For any female patient: β-HCG with the blood works you will order OSCE-guide-III. … your blood sugar was low. you are doing well now. chest is moving   Listen to lungs. I would like to monitor her blood glucose every 5-10 minutes + In case of DKA and physical exam: History  The same as in hypoglycemia (see below)  The causes are (5 Is): insulin missed / infection / intoxication / ischemia / infarction Physical exam  Brief neuro exam  Brief DM exam + In case of hypoglycemia: History Are you diabetic? Analysis Diabetic  When were you diagnosed? And how? history  Do you take insulin?  Have you had coma (DKA or hypoglycemia) before?  When was your last DM follow-up visit? Any reason?  At that time. do you hear me  I would like to activate ACLS code please / start primary survey A B C D  Check the mouth. heart  I would like to get the vital signs please: ↑ BP and ↑ HR  2 large IV lines. for IV fluids5 and to withdraw samples6 When you ask the nurse for stat glucose by finger prick: Give 4L O2 via nasal canula Monitor O2 for need to intubation Hypoglycemia Hyperglycemia  Stat 100 mg thiamine IV  Stat insulin 10 units IV  Stat 50 ml D50 (Dextrose 50%) IV  Stat 100 mg thiamine IV  If no IV line  glucagon IM  2 L fluids At that time.Emergency Medicine Unconscious Patient – Diabetic 16 years old female found unconscious in her class. next 10 minutes manage and counsel Introduction: . hoe did you feel? Hungry / shaky / dizzy / sweating? 5 If the HR is normal and other VS are normal. you are in the ER in hospital. did you get breakfast. your class-mates start to regain her conscious brought you here.Ms … … if you hear me.Ms … I am Dr … I am the physician in charge in the ER. listen for patent air way   What is the O2 please  Trachea central. . were you controlled? Symptoms free?  When was your last Hb A1C test? What was it?  How about last few days. can you open your eyes please? Tap on the shoulder. your insulin? Did you check your glucose? Did you exercise?  Before you lost conscious. how do you feel right now? Patient states that she is  Reassure her worried she will lose her  I can help by giving you a doctor’s note exam / or other important  This is a very serious condition. you need medical appointment! attention for some time it is not safe to leave D1: Brief neurology  Start D5 (Dextrose 5%): 250 ml / hr D3: Dextrose  Nurse. use it. it is the only source of energy to our brains.Diabetes clinic / Foot specialist / Dietician . low blood sugar is even more dangerous. they can identify the situation and provide help. or nerves injury and harm on the longer term. eye.What is your understanding about diabetes mellitus? Pathophysiology: . reach to the nearest ER Emergency measures: . Complications: . this key is the insulin. and he can refer you to “diabetes” clinic. and immediately eat a candy / chocolate / juice o So.On the other hand. the small and big ones. their body does not produce insulin.If you exercise. do you know how to identify it before you totally lose your conscious? o Whenever you feel hungry / sweating / shaky / dizzy / heart racing o You need to stop. so we need to compensate for that by giving it from external source. Prevention: . to our bowels where it is absorbed and goes to all our body. if this happens again.I will still give you some brochures and web sites in case you would like to know more. to take it in case of emergency • If you are at home. The best way to treat is to prevent this from happening. there is a special type of injections (glucagon emergency kit). so if you lose conscious and some one finds you. .It is a condition related to our blood sugar. Notes: If you are the family physician. and may give a lot of complications! It might cause kidney. and it might happen again. . what referral will you do for a diabetic patient? . Our organs (brain / muscle) use this sugar as source of energy. if your blood sugar drops suddenly. In order for muscles to use this sugar. it needs a key to enter into cells. . by: o Make sure that you always eat after your insulin dose o Monitor your blood sugar frequently o If you exercise. because it affects all our blood vessels.High blood sugar is harmful for our bodies. . adjust your insulin dose based on your blood sugar level .Emergency Medicine Impact Red flags DD PMH   If pt dx < 5 years  no worry about complications If pt dx > 5 years screen for complications (urine changes / visual changes / numbness and tingling in her feet)  Constitutional symptoms  How much insulin do you take? Do you make sure you eat after your insulin dose?  Any ↑ or ↓ in your weight? Stress?  Did you start new medication? HEAD SSS /+/ Pregnancy Counselling: .doc Page 83 of 255 .What happened to you is a very serious condition.That is why it is important that you have a bracelet that mentions you are diabetic. do you know why? Because our brain can not survive without blood sugar for more than 5-7 minutes. Follow-up: . type I and type II. you need to keep glucose tablets in your bag. for more education and assessment.If DM type I > 5 years.We have two types of DM. .You should see your family physician within few days. • If you are out. OR type II at any time: Ophthalmologist / Nephrologist / Neurologist OSCE-guide-III. keep monitoring your blood sugar. or other people can use it to inject you. The food travels through the food pipe to our stomach. including sugar. the food contains different components. Whatever we eat.Patients with DM type I.Now. HTN. In the next 10 minutes. if blood sugar is ok. did he shout? Starred at the wall? Complained of strange smell?  Was all his body seizing or part of it? For how long? Did he bite his tongue? Rolled up eyes? Did he wet himself? Was breathing?  Did he regain conscious alone or with intervention? Cause  History of epilepsy? Medications for epilepsy?  And mood stabilizers medications?  RECENTLY. no need for naloxone. I will make sure he is stable then I will ask you more questions Mr … . kidney. manage. did he complain of: Neurological / Constitutional symptoms  Any history of trauma / head injury?  Recent ear infection?  SAD: sweating / shaking  Any medications / blood thinners PMH  Long term disease. the patient starts to seize. lung. on the way he had attack of seizures. he is unconscious now. Patient is unresponsive. Introduction Very brief introduction to wife. give ativan 2 mg IV and reassess ABCD History Event  First time to seize?  Can you describe what happened? Did he fall to the ground? (wife)  Before he seized. and received 1 dose of diazepam. I will start my primary survey: can you hear me  A: can you open your mouth (open and comment) / trachea central / JV not engorged STABILIZE  B: listen to lungs and apex / normal air entry on both sides / normal heart sounds  C: can I get the vitals please! Normal! 2 large IV lines please ─ One to give IV fluids 50 ml/hr to keep vein open (KVO) ─ The other one is to withdraw samples  D1.doc Page 84 of 255 . e. drugs: universal antidotes  thiamine 100 mg / if O2 is ok.Emergency Medicine Unconscious Seizing Patient – DT / Epilepsy / Brain tumour / … Patient arrives to the ER with his wife.g. deficits: pupillary reaction  D3. no need for dextrose If at any time. DM. or heart disease  Previous hospitalization / surgeries?  FH of epilepsy FH Examination  Vitals from the examiner  Glasgow coma scale (if < 8  arrange for intubation) Neurological examination:  Cranial nerve examination  UL and LL: tone and reflexes Management  Stat neurology consult  Stat CT brain OSCE-guide-III. Brain tumour / brain hemorrhage . if O2 saturation is low even after you give O2.Emergency Medicine N.D . jump to shine light in the pupils  if pin-point-pupil  give stat naloxone 0.Cocaine overdose How to identify if this is narcotics overdose? .While doing AB.Epilepsy .0 mg IV . and respiratory rate is low.C .Then assess Glasgow coma scale.Organ failure . if not breathing  mask and bag . if fluids  suction . Causes of seizing: .B.2 – 2.Withdrawal from sleeping pills .Stroke lateralization .A.Infection / meningitis .Delirium Tremens .doc Page 85 of 255 . if < 8  intubate (ask for DNR here) Common 5 causes for unconscious patients: .Heart block OSCE-guide-III.Then back to ABCD Any unconscious patient: .Epilepsy relapse .Electrolytes imbalance .Hypoglycemia / DKA .Meningitis .B. 1 heart: Aortic dissection Chest pain presents with heart racing / SOB / nausea / vomiting / sweating History will be: chest pain analysis / cardiac symptoms / risk factors If blood pressure is low: we only give oxygen / aspirin / and plavix If inferior MI (II.If antero-lateral infarction (V2/3/4/6 & aVL): 5 mg morphine .Stat cardiology consult .↓ – position or deep breathing  Patient is talking to me  air ways ok.Stat cardiology consult .2 brain: Brain tumour / Bleeding in brain ± stroke .3 mg puff X 3 times.Emergency Medicine Heart Attack – Chest Pain (MI or Heart Block) 4 cases: .Catheterization lab  angiography  Heparin (ask about the hospital protocol for heparin – usually 5000 U IV then 1000 U/hr by infusion) or give enoxaparin 1 mg/ kg) . but first rule out the contra-indications for thrombolytics: . and inferior MI (II.. aVF)  Cocaine (unopposed alpha  ↑ blood pressure)  labetalol 20 mg (2 mg/min for 10 min) You give metoprolol 2 mg IV .Chest pain for 45 minutes. III.If no low BP . EXCEPT:  History of asthma  ↓ BP. and the morphine increase the pre-load  2 mg morphine only Given to all patients.2 recent (< 2 weeks): Recent surgery / Recent bleeding . ↓ HR. heart block. aVF)  usually involves the right ventricle. fib: chest pain and the patient is intoxicated .V. aVF)  I need 15 lead ECG / do not give β-blockers Risk of bleeding with thrombolytics is 1%. fid: chest pain and the patient is calm Introduction Brief history ABCD MONA Oxygen Aspirin Nitrates Morphine ± β-blockers No ST elevation ST elevation      … How are you feeling right now?  Os Cf D  PQRST  ↑. but being serious.doc Page 86 of 255 . with ST elevation .V. III.If (II.Chest pain for 45 minutes. this needs consent OSCE-guide-III. III.Catheterization lab  angioplasty  Heparin (enoxaparin 1 mg/ kg)  If the cardiologist is not available  start thrombolytics.If the pt did not consume Viagra recently  If patient is in pain after 3 puffs of nitrates and there is NO hypotension or bradycardia  morphine 2 mg ..  Listen to chest to exclude pneumothorax  breathing is ok  C and D briefly  4 L O2 via nasal canula  Aspirin 325 mg  Plavix 300 mg  0. with normal ECG . 5 minutes apart . ECG normal: order Serial ECG every 5 minutes / if any ↓ VS / any change in pain .3 mg X 3.Emergency Medicine Case 1: Chest pain with initial normal ECG Chest pain for 45 minutes.doc Page 87 of 255 ... 5 minutes apart  D1: brief neuro exam  pt is grossly neurological free  Any results back?  Order chest x-ray?  Cardiac symptoms: heart racing / SOB / nausea / vomiting / sweating / dizziness / LOC  Chest symptoms  Constitutional symptoms  DD: GIT (difficulty swallowing / acidic taste / GERD / heart burn / hx of PUD) / Trauma / relation to respiration / calf pain / swelling  PMH: medications / blood pressure / blood glucose / cholesterol / SAD (especially for smoking and cocaine) / stress  FH: heart diseases / heart attacks       Another ECG Normal ECG Do physical exam: Vitals / compare BP in both arms – ? aortic dissection General status of the patient Eyes / mouth Heart and chest examination LL edema      OSCE-guide-III. manage Introduction Pain history ABCD Brief history … How are you feeling right now? Os Cf D PQRST (R: how about your jaws / back / arms) ↑.↓ – position or deep breathing Start O2 4 L by nasal canula … vitals please: comment and ask for regular VS check 2 IV lines: KVO / take samples and serial ECG .Have you recently used Viagra? If no  nitroglycerine puffs 0.How did you come here today? .Did you take any aspirin? aspirin 325 mg / plavix 300 mg . This is a serious condition. Based on this. you are in a safe place and we have experience in dealing with that. not under influence of alcohol or any substance. and I will ask the patient to sign a LAMA (leaving against medical advice). we need to reopen this clot. Heart attack means one or more arteries providing blood supply to your heart are blocked by a clot. Stat cardiology consult Do we have catheterization lab? What we need to do now is to use medication called “thrombolytic” or “clot buster”. it might cause bleeding in 1% of cases. and based on the BP. I would like first to make sure that you are a good candidate. it is a life saving procedure. (Am I dying Dr?). I do not see signs consistent with cardiac aneurysm.doc Page 88 of 255 . and I will let him go OSCE-guide-III. so we need your consent if you would like to have it! Heparin (ask about the hospital protocol for heparin – usually 5000 U IV then 1000 U/hr by infusion) or give enoxaparin 1 mg/ kg) Can you get the thrombolytics kit please ST elevation:  Lateral MI  Inferior MI The patient wants to leave the hospital: .I would like to make sure he is competent. it has side effects. cocaine) Mr …. inferior MI. however you are stable and doing well.I would explain to the patient: diagnosis / treatment / side effects of treatment / complications of not receiving treatment / alternatives . I can say that you are a good candidate for thrombolytics. it does not look like having dissection. recently have you had any surgery or bleeding? Have you ever had stroke before? Were you diagnosed or told that you have brain mass? Lesion? Based on the ECG.I will document this. ↓ BP.Emergency Medicine ST elevation       Morphine 2 mg (up to 5 mg in antero-lateral MI) β-blockers: metoprolol 2 mg IV (if NO contraindications: history of asthma. heart block. this includes stroke. it seems that you have a heart attack. based on your symptoms and your ECG. however like any other medical intervention. and to rule out suicidal ideation . ↓ HR.  Stat cardiology consult  Do we have catheterization lab? What we need to do now is to use medication called “thrombolytic” or “clot buster”. it does not look like having dissection. we need to reopen this clot.ECG: ST elevation in V 2/3/4. it has side effects. it seems that you have a heart attack.How did you come here today? . based on your symptoms and your ECG. Heart attack means one or more arteries providing blood supply to your heart are blocked by a clot. you are in a safe place and we have experience in dealing with that. This is a serious condition. I do not see signs consistent with cardiac aneurysm.Have you recently used Viagra? If no  nitroglycerine puffs 0. aVL  ant-lat MI .↓ – position or deep breathing ABCD  Start O2 4 L by nasal canula  … vitals please: comment and ask for regular VS check  2 IV lines: KVO / take samples and serial ECG . I would like first to make sure that you are a good candidate. ↓ HR.doc Page 89 of 255 .Did you take any aspirin? aspirin 325 mg / plavix 300 mg . Based on this. cocaine overdose) Mr ….Emergency Medicine Case 2: Chest pain with STEMI Introduction … How are you feeling right now?  Os Cf D Pain history  PQRST (R: how about your jaws / back / arms)  ↑. recently have you had any surgery or bleeding? Have you ever had stroke before? Were you diagnosed or told that you have brain mass? Lesion? Based on the ECG. I can say that you are a good candidate for thrombolytics. ↓ BP. (Am I dying Dr?)... it is a life saving procedure. however like any other medical intervention.3 mg X 3. 5 minutes apart  D1: brief neuro exam  pt is grossly neurological free  Any results back?  Order chest x-ray?  Cardiac symptoms: heart racing / SOB / nausea / vomiting / sweating / Brief history dizziness / LOC  Chest symptoms  Constitutional symptoms  RISK factors  PMH: medications / blood pressure / blood glucose / cholesterol / SAD (especially for cocaine) / stress  FH: heart diseases / heart attacks ST elevation  Morphine 2 mg (up to 5 mg in antero-lateral MI)  β-blockers: metoprolol 2 mg IV (if NO contraindications: history of asthma. this includes stroke. it might cause bleeding in 1% of cases. heart block. however you are stable and doing well. so we need your consent if you would like to have it!  Heparin (hospital protocol or give enoxaparin 1 mg/ kg)  Can you get the thrombolytics kit please Have time Physical exam: listen to lung bases / S1 and S2 / compare BP both arms OSCE-guide-III. inferior MI. and based on the BP. o Can you give me the last ECG please?  NORMAL ECG How do you feel? Do you feel drowsy? Because you have just had cardiac arrest. Manage as the first case  the chest pain with normal ECG Case 4: Chest pain – v fibrillation – intoxicated patient - - - I would like to initiate code blue – ACLS Is this is the patient ECG? Is it the last one? We need to start defibrillation! o Done. you should not leave.We will give you a medication that interacts with cocaine.doc Page 90 of 255 . we would like to make sure you are stable. ? intoxicated o If the nurse gave you “cocaine” and said it was found with the patient. and might happen again.What were you doing in the party? Any alcohol? Drugs? . this is a very serious condition. o Can you give me the last ECG please?  NORMAL ECG How do you feel? Do you feel drowsy? Because you have just had cardiac arrest. we would like to make sure you are stable.Emergency Medicine Case 3: Chest pain – v fibrillation / v tachy - - - I would like to initiate code blue – ACLS Is this is the patient ECG? Is it the last one? We need to start defibrillation! o Done. The patient will be rude / angry / aggressive. we want to make sure you are stable.Any weakness / numbness / tingling / vision problems / hearing problems? .Do you hear voices / see things? Do you think someone want to hurt you? - Manage as the first case  the chest pain with normal ECG OSCE-guide-III. it is important to stay. o If agitated  1 mg lorazepam injection Cocaine (arrhythmias / HTN / neurological manifestations): . ask her to keep it with the hospital security o Mr … you have just had a cardiac arrest. it is very important to tell us if you took any cocaine recently! (if cocaine  Ca channel blockers) . dated and valid? o What does he have? . chest is moving  Monitor O2 for  Listen to lungs. pupils.doc Page 91 of 255 . You can still pace maker 1.One of them is DNR (must be dated. valid. we will not … (if DNR  do nothing) . I would like to see the patient first to make sure he is stable.The other case is: do not intubate / do not defibrillate. fib or V. fib or V.What is this patient code status? Any advanced directives? o Is it signed. for IV fluids and to withdraw samples 2.Leads on sternum and apex D1 Brief neurological scan.For any unconscious patient: ask about advanced directives or DNR! What is this patient code status? .I would like to activate the ACLS code please / start primary survey  Check the mouth. I would like to see the patient first to make sure he is stable. tachy.If no DNR: D2 Third degree heart block:  1 mg atropine  any changes?  Pace maker: . … Collateral  Do we have his file? history  Can we contact his family physician? Or family member?  Does he have a med alert?  Do we have paramedics report?  Can we check his belongings? He is taking …. fib or V. tachy. OSCE-guide-III.Whenever the examiner or the nurse tries to give you an ECG at the room entrance.We will respect his wishes. tachy and report: there are no signs of V. do you hear me? . and then I will look at the ECG. . C  2 large IV lines. heart need to intubation  I would like to get the vital signs please.Glasgow Coma Scale  [if < 8  intubate (ask about DNR)] .Mr … I am Dr … working in the ER. and signed) .Introduction: . assess for V. thiazides and digoxin +  I need to get his K level / digoxin level and I would like to check the dose for digi-bind and digi-fib Physical  Neuro exam exam  Cardiac exam Notes: .Rate: 20 more than his base heart rate .Is this is the last ECG for this patient? I do not see any signs of V.This patient Glasgow coma scale is … .Emergency Medicine Heart Block 2 cases: . listen for patent air way  Give 4L O2 via A  What is the O2 please nasal canula B  Trachea central. Brudzinski's signs     CT scan stat ± LP Others: Chest x-ray / Urinalysis Blood works (CBC / differential / lytes) Septic workup (samples / C&S)     OSCE-guide-III.↓ – position or coughing  Acute neuro: fever / neck pain / stiffness / vision / hearing / gait / falls / weakness / numbness  The patient says: I am diabetic  stat blood glucose (prick) HTN / blood thinners / kidney diseases Kidney cysts / disease / aneurysm Cocaine Associated symptoms PMH FH SH If the patient loses his conscious  manage as unconscious patient  If ↑ BP and ↓ HR  I am suspecting Cushing triad.Can you please lie down?  Put the bed 45°. Kernig's.ABCD: IV lines / D1: Pupils  PQRST  ↑. D3: thiamine 100 mg Secondary  Glasgow coma scale As in the case of the survey unconscious patient Physical exam Management     Orientation (if conscious) Quick cranial nerves Quick neuro (tone/ motor / sensory / reflexes) Nuchal rigidity. put the bed in 45° ABCD  D1: Pupils.To ↓ intra-cranial pressure ? subarachnoid hemorrhage ? meningitis (FEVER) Treatment of meningitis: vancomycin 1 g IV q12h + ceftriaxone 2 g IV q12h ± ampicillin 2 g IV q4h (if >50 years or hx of alcohol use or immunocompromise ) Page 92 of 255 .. I would like to make sure u r stable! ..To keep air ways patent .doc Stat neurosurgery consult Empiric antibiotics IV mannitol 20% – 1 gm / kg Intubate: .Emergency Medicine Headache Introduction CC Analysis of CC Headache for 2 hours (very acute – very serious) Os Cf D  Is this your first time  Did you get any trauma?  Would you describe it as the worst headache in your life? Thunder clap? . Emergency Medicine Acute Abdominal Pain OsCfD /+/ PQRST  will give you an idea In abdominal pain cases:  If you suspect certain diagnosis  scan for the risk factors for that disease  If no diagnosis  continue GIT symptoms and general differential diagnosis You suspect obstruction  nausea / vomiting (COCA+B / coffee ground material) AND bowel movements  if vomiting  screen for dehydration  If you dx obstruction  check risk factors of obst  then rest of GI symptoms  If not obst  scan GIT  near-by systems  PMH for systemic disease If you suspect kidney stones  screen with renal symptoms  If you dx renal stone  check risk factors (diet. uric acid. and now continuous dull pain / PQRST / What ↑ or ↓ (position / eating / bowel movements / vomiting)  Screen for obstruction: ─ Nausea/ vomiting o Relation to pain. Management: (1) NPO / NG tube. OSCE-guide-III. how do you feel now? Analysis: OsCfD: gradual. started colicky. which started first. (6) Correct electrolytes. (4) Stat surgical consult. (2) Oxygen mask. bone pains / fractures)  then rest of urinary symptoms Intestinal obstruction Intro Analysis of CC … But first I would like to ask you.doc Page 93 of 255 . medications. (3) IV fluids. hx of renal stones. (3) No air under the diaphragm. does it relief pain o COCA + Blood (coffee ground material) Impact  Screen dehydration (dizziness / light headedness / thirsty / LOC)  Bowel movements ─ How about any blood? Any time? ─ Still passing gas? Red flags Risk factors for intestinal obstruction:  Previous surgery? What? When?  Fever/ night sweats/ chills / appetite / loss of weight / lumps & bumps  PMH or FH of cancer or benign tumour  Hx of Crohn’s disease (hx of abd pain/ bloody diarrhea) / family hx  Hx of hernia / groin mass  Gall bladder stones / right upper quadrant pain Differential  Gastroenteritis: diagnosis ─ What did you eat yesterday? Place that you are not used to? ─ Diarrhea / blood in stools? ─ Anybody else ate with you and suffered from the same problem  Renal: flank pain / burning sensation / going more to washroom / stone  Liver: yellowish discoloration / itching / dark urine/ pale stools  Hx of HTN / SOB / cough / phlegm (aortic dissection) PMH / FH / SH    X-ray findings of small intestinal obstruction: (1) Multiple air/fluid levels. (5) Foley’s catheter. (2) Dilated loops of small intestine. upright CXR: o May be normal o Localized diverticulitis (ileus. fistulas ─ Barium enema: contraindicated during an acute attack: risk of chemical peritonitis (perforation) ─ Sigmoidoscopy/colonoscopy: o Not during an acute attack. very useful for assessment of severity and prognosis. SBO. (3) PID at young age. discharge (PID) PID  Diagnostic plan: ─ Pregnancy test – β-HCG ─ CBC / ESR ─ Cervical culture (for Gonorrhea and Chlamydia) ─ Syphilis serology  What is the treatment of pelvic inflammatory disease? ─ Cefoxitin 2 g IV every 6 hours X 2 days (covers anaerobic bacteria) ─ Doxycycline 100 mg orally BID X 2 weeks ─ Remove any IUD (if present)  What are the indications of hospitalizing the patient? (1) Pregnancy. (2) Pelvic abscess on U/S scanning / high fever (> 38. (6) Immunodeficiency (patients with HIV infection) or severe illness  Complications of PID: abscess / ectopic / infertility / intestinal obstruction / peritonitis Left lower quadrant abdominal pain – elderly  Diverticulitis  Signs: ─ Fever ─ Peritoneal signs: +ve cough tenderness / percussion tenderness / tenderness / rebound tenderness / DRE: severe pain  DD: diverticulitis / abscess / cancer  Investigations: ─ AXR. bleeding (abortion).doc Page 94 of 255 . partial colonic obstruction) o Free air may be seen in 30% with perforation and generalized peritonitis ─ CT scan (optimal method of investigation). (4) Recurrent PIDs. 97% sensitive.5 °C). Very helpful in localizing an abscess ─ Hypaque (water soluble) enema – SAFE (under low pressure): o Saw-tooth pattern (colonic spasm) o May show site of perforation. only done on an elective basis o Take biopsies to rule out other diagnoses (polyps.Emergency Medicine Right / Left lower quadrant abdominal pain – middle aged female  History: nausea / vomiting / fever / LMP / vag discharge / risky behaviour  Physical exam: MUST mention pelvic and bimanual exam  DD: (Left side): ectopic / PID / ovarian cyst / torsion / kidney stone  DD: (Right side): ectopic / PID / ovarian cyst / torsion / kidney stone / appendicitis / intest obstruction  Investigations: β-HCG / CBC / differential / abd-pelvic US / culture of the vag and cervical secretions  Long term complications: abscess / ectopic / infertility / intestinal obstruction / peritonitis  Signs at PV exam: left adnexal mass / cervical motion tenderness Acute abdomen in a female  missed period (ectopic). thickened wall. (5) Failure to respond to outpatient management. abscess cavities or sinus tracts. malignancy) OSCE-guide-III. Emergency Medicine   Management: (1) NPO / NG tube.doc Pain: OCD / PQRST / ↑↓ Vomiting: number of episodes + COCA AS: fever / malaise / cough / urinary symptoms / diarrhea / gastroenteritis Diabetic history Dehydration: feel weak / pee less / dry tongue and skin Hyperglycemia symptoms: thirst. polyuria. then 1 ml/kg/hr for the next whatever Serial blood glucose ABG / serum ketones CBC / lytes Septic workup (chest x-ray / blood cultures / urinalysis) ECG (for the ↑ in K+) Page 95 of 255 . (b) abscess needing percutaneous drainage ─ Consider after 2 or more attacks. 4:2:1 rule: 4 ml/kg/hr for the first 10 kg. abdominal tenderness. (3) IV fluids. polydipsia. recent trend is toward conservative management of recurrent mild/moderate attacks Diabetic ketoacidosis  acute abdominal pain Analysis Impact Red flags DKA DD – causes of DKA Management of DKA                        OSCE-guide-III. then 2 ml/kg/hr for the next 10. of diabetes mellitus Non-compliance OR wrong dose Recent surgery Pregnancy Trauma MI Infection IV fluids – NS (1 L/hr x 2 hrs then 500 ml/hr x 2 hrs then 250 ml/hr x 4 hrs) Foley’s catheter Insulin drip 2 units / hour – check glucose and lytes every 2 hours When glucose reaches down to 15  fluids will continue as maintenance. (4) Stat surgical consult (5) IV antibiotics (IV ciprofloxacin 500 mg BID / IV Metronidazole 500 mg TID) Indications for surgery for diverticulitis: ─ Unstable patient with peritonitis ─ Hinchey stage 2-4 (large abscess / fistula / ruptured abscess / peritonitis) ─ After 1 attack if: (a) immuno-suppressed. (2) Oxygen mask. 2/3 : 1/3 of D5W : NS + 20 mEq KCl/L. and disturbance of consciousness Complications. and nocturia Acidosis: shallow rapid breathing or air hunger (Kussmaul or sighing respiration).  In the moment.Emergency Medicine Acute Abdomen Patient came to ER with abdominal pain / vomiting / diarrhea / BP 90/60 / HR 120 Acute pancreatitis / dissecting aortic aneurysm (no vomiting) / perforated peptic ulcer Introduction Analysis Impact Red flags DD PMH I can see that you have a lot of pain.doc Page 96 of 255 . I would like to make sure you are stable  What are the vitals pleas? Stable Unstable Proceed to I am going to start my primary survey  ABCD  When you send blood works: add lipase / amylase history ↓ Did you vomit blood? How about coffee ground? (if yes: order blood) Os Cf D / PQRST / ↑↓ / relation to position / breathing / eating  Vomiting  COCA + Blood  Change in the bowel movements  Dehydration  How do you feel right now? What are the vitals please?  Constitutional symptoms Liver / GB  Yellowish discoloration / itching / dark urine / pale stools?  Recent flu-like illness?  Do you have hx of gall bladder stones? Repeated attacks abd pain? Stomach  Hx of PUD? GERD? Acidic taste / heart burn?  Alcohol? How much? When was the last time? Did u drink > usual?  Gastroenteritis (What did you eat yesterday? Place that you are not used to? Diarrhea / blood in stools? Anybody else ate with you and suffered from the same problem?) Medications  If vomited blood: Do you take steroids / NSAIDs / blood thinners? Kidney  Flank pain? Burning sensation? Dark urine? Frequency? Aorta  Hx of HTN / atherosclerosis / DM / ↑ cholesterol / smoking / SOB Trauma  Did you have trauma?  Medications / allergies / long term disease?  Physical exam: abdominal exam Face Eyes for jaundice / mouth for dehydration / ulcers 7 8 Inspection  No Cullen’s sign / no Grey Turner sign  Cough tenderness 7 Cullen’s sign: peri-umbilical ecchymosis. also with retroperitoneal bleeding OSCE-guide-III. It arises from spread of retroperitoneal blood associated with: pancreatitis / ruptured ectopic preg / ruptured aortic aneurysm / ruptured spleen / perforated duodenal ulcer 8 Grey-Turner sign: ecchymoses of the skin of the flanks. bear with me for few minutes and I will give you a pain killer as soon as I can. ASK FOR X-RAY (3 view x-ray abdomen)  ? perforation ─ Perforation  severe guarding. and I can not release  OSCE-guide-III. I understand.doc Page 97 of 255 . ─ I can assure you that he is well taken care of.C. will not be able to proceed ─ Obstruction ─  Patient is obviously in severe pain. ─ All the details of his medical information is absolutely confidential. ─ In order for me to examine you properly. she is asking about his condition?! ─ I am still doing my examination. can you bear with me for few minutes then I will give you pain killer as soon as I can. can you please lie on your back! ─ Do you feel relieved like that. I need to get good look at your abdomen. Finally she will lie down. I will not be able proceed with examination Management I am suspecting acute peritonitis (? perforated peptic ulcer)  NPO / nasogastric tube  O2 mask  IV fluids (250 ml/hr) / Cross match blood I am suspecting pancreatitis    NPO / nasogastric tube O2 mask ICU admission   Pain killer: Meperidine 1 mg / kg or Fentanyl Stat SURGICAL consult  Pain killer: Meperidine 1 mg / kg or Fentanyl Stat GIT consult    3 view abdominal x-ray Abd U/S CT abdomen    3 view abdominal x-ray Abd U/S for gall stones CT abdomen  Lab: CBC / blood sugar / calcium / amylase / lipase  Lab: CBC / blood sugar / calcium / amylase / lipase Albumin level / serum Ca   DD:  Perforated PUD: vomiting coffee ground material  Aortic dissection: NO vomiting / severe pain shooting to the back  Acute pancreatitis: ─ NO upper GIT bleeding ─ Fever (due to chemical irritation not infection) ─ Pain improves when leaning forward ─ Paralytic ileus ─ Tetany Ethical question: The patient girl friend is on the phone. It will be only few minutes.Emergency Medicine Auscultation Percussion Palpation  Patient in severe pain: I can see that you are in pain. do you want to give it a try? Slowly! Do you want me to help you! It is crucial to reach a proper diagnosis! ─ If still refusing  offer 2 mg morphine S. for that reason. and we will do our best to help him. heart burn  Risk factors for ca stomach: smoking  Do you take any blood thinners? Aspirin? Steroids? NSAIDs? Medications Any medications / allergies Family history of gastric cancer    ↑ or ↓ fluids (if stable: fluids 250 ml/hr) and monitor vitals General appearance of the patient: cachectic / distressed / … I would like to check if there is any postural drop in the blood pressure Exam Liver exam: extra-hepatic signs of liver cell failure /+/ Bruises Abdominal exam: epigastric mass / pain / liver / ascites If painful: manage as acute abdomen case (perforated PUD) Management STAT GIT consult for UPPER GIT endoscopy IV pantoprazole (80 mg bolus then 8 mg/hr) IV octereotide (25 mcg/hr)  portal circ VD  ↓ portal pressure Abdominal x-ray Admission to ICU Longer term management:  If portal HTN: non-selective β-blockers  Advice on cutting down the alcohol  Advice to follow-up with the family doctor Vitals General           OSCE-guide-III.doc Page 98 of 255 . … make sure that you are stable. how much / for how long / more than usual? SAD?  Long term liver disease? ↑ Abdomen size? Bruises? Memory? Concentration? Numbness LL?  Risk factors for hepatitis: piercings.Emergency Medicine Upper GIT bleeding Patient vomited blood. bld transfusion. unprotected sex Stomach  History of PUD / pain / GERD. … protection … Mr …. then I will ask you … How do you feel right now? Be aggressive with fluids – 2 L IV  Os Cf D / COCA + dehydration  Associated with pain (painless: esophagus / pain: stomach)  PQRST  Nausea / vomiting / diarrhea  Recently. any blood in stools / dark stools?  Weak / drowsy / …  Constitutional symptoms – cancer stomach Liver  Yellow / itchy / urine / stool  Alcohol. esophageal: varices / gastric: perforated PUD Introduction ABCD Analysis Impact Red flags DD PMH FH SH Because this is a case of GI bleeding. … make sure that you are stable.Emergency Medicine Lower GIT bleeding Introduction  Because this is a case of GI bleeding.doc Page 99 of 255 .B. then I will ask you …  How do you feel right now? ABCD Be aggressive with fluids – 2 L IV  D1: you might find some deficits  previous cerebro-vascular accident (may be history of weakness and numbness) Analysis  Os Cf D / COCA + dehydration  Associated with abdominal pain  PQRST / ↑ – ↓ / related to meals  Vomiting blood? Nausea / vomiting? Impact  Weak / drowsy / … Red flags  Constitutional symptoms – cancer colon DD Liver  Liver: yellow / itchy / urine / stool Colon  Hx of dx / screening / for Ca colon? When? What was the result?  Recent change in bowel movement?  Recent changes in calibre of stools?  Risk factors for Ca colon: Fm hx of ca colon / diet (↓ fibres ↑ fat)?  Risk factors ischemic colitis? Atherosclerosis (similar to CAD)? Medications  Do you take any blood thinners? Aspirin? Steroids? NSAIDs? PMH Any medications / allergies FH Family history of colon cancer SH SAD Vitals ↑ or ↓ fluids (if stable: fluids 250 ml/hr) General  General appearance of the patient: cachectic / distressed / …  I would like to check if there is any postural drop in the blood pressure Exam  Abdominal exam: ─ If soft abdomen. … protection …  Mr …. and no findings (PE disproportionate to the pain)  ischemic colitis Management  STAT SURGERY consult  NPO / IV fluids / Foley’s catheter  Admission to ICU  Abdominal x-ray  CT angiography DD With pain ─ Cancer colon: chronic intermittent ─ Ischemic colitis: acute / sudden Painless ─ Diverticulosis ─ Angiodysplasia ─ Upper GIT bleeding ─ Aspirin ─ Hemorrhoids Colitis: Radiation / Infectious / Ischemic / IBD (UC > CD) Abdominal x-ray findings: ─ Intestinal ischemia: thumb print sign ─ Ischemic colitis: pneumatosis intestinalis (coli) N. ischemic bowel  metabolic acidosis OSCE-guide-III. aVL: antero-lateral MI (left coronary)  II. aVF: inferior MI (right coronary. III. posterior and inferior surfaces)  Hear block – third degree /+/ Bundle branch block  Hyperkalemia /+/ Hypokalemia /+/ Hypercalcemia  Digitalis toxicity OSCE-guide-III.doc Page 100 of 255 .Emergency Medicine ECG  Normal  V fib /+/ V tachy /+/ Torsades du pointes  A fib /+/ Atrial flutter  ST elevation: o Pericarditis: all leads o MI:  V 2/3/4 ± V5/6. Rate:  Regular: 300/number of big squares (R-R)  Irregular: Number of “R”s x 6 2. Axis Normally.doc Page 101 of 255 . Rhythm  check for P wave in lead II Tachyarrhythmias:  Sinus tachycardia  Irregular irregularities: A. i. I and III facing.2nd degree:  Mobitz type I: P-R intervals increasing. with missing QRS  Mobitz type II: P-R intervals constant.  Wide QRS in V6 (M mountain): LBBB. A mnemonic to remember ECG changes is WiLLiaM MaRRoW. I and III opponents. II. Diagram showing how the polarity of the QRS complex in leads I. and III can be used to estimate the heart's electrical axis in the frontal plane: Lead I negative and aVF positive: Rt axis deviation / Lead I positive and aVF negative: Lt axis deviation. with LBBB there is W in V1 and M in V6 and with a RBBB there is M in V1 and W in V6 OSCE-guide-III.3rd degree: P-P has a rate. QRS in leads I. Tachy / V.Emergency Medicine 1. 4. and the R-R has another rate 3. . II. Flutter  Rapid SVT  Ventricular arrhythmias: Premature ventricular beats / V. Fibrillation  Saw teeth (regular irregularities): A.  Right axis deviation: QRS in I is negative (downwards ↓). III is negative (downwards ↓).e. but every P  QRS. if in V1: QRS is upwards ↑ & wide: RBBB. Fib / Torsades de pointes Bradyarrhythmias: (< 60/min)  Sinus bradycardia  Heart block: . III are positive (upwards ↑).1st degree: P-R intervals increasing. Bundle Branch Block:  Normally QRS in V1 is downwards ↓. with missing QRS .  Left axis deviation: QRS in II. Wide QRS 3.Spiked T wave Page 102 of 255 . Others: Hypokalemia ST segment depression.Emergency Medicine 5. large U waves. ST segment: Angina STEMI AND Non-STEMI 6. OSCE-guide-III. and a slightly prolonged PR interval.doc Hyperkalemia 1. inverted T waves.Flat P wave 2. is he able to recognize you? Able to talk? To move? Confused? . I will ask you some questions and give you some instructions.doc Page 103 of 255 .Can you support and monitor him.Did he regain consciousness? After he regains consciousness.Did he lose conscious? Which happened first.Did you watch that? .What is your child weight? .Did he fall from height? Did he hit his head? . and do not call me.Does he feel any weakness. I need your help here. remove any object near him? . however.Before I proceed. seizing or LOC? .Does he have fever? When did it start? Any medical advice? Any diagnosis? Treatment? Is he taking the medications? Any reason? . .Can you put him on his side?  analyze during . numbness? . meningitis: 1st time send the ambulance.Is your child alert/conscious? Is he breathing? What is his color? . .Did you call the ambulance? I will ask the nurse to call for you .I know this is stressful time for you.Is he breathing? Did he turn blue? . may I know who is on the line? . I am Dr … the physician in charge at the (clinic/ER). panicked as her child is seizing for 3 minutes Introduction + Reassurance Stabilize Analyze the event During After Fever Rule out BINDE PMH FH Counselling .Constitutional symptoms Meningitis / pneumonia Neurological deficits / diabetes mellitus Febrile seizure / epilepsy The seizing child counselling Notes:  Febrile seizure vs.Was he shaking? Certain part of the body or whole? .Do not put anything in his mouth. do not fix him . I will call you back.Good evening.Any tongue biting? Did he wet himself? Roll up his eyes? .How long did it last? How did it stop? Spontaneously? .Can you describe the seizure for me please! . hang on.Monitor till he stops seizing  assess for focal symptoms  after . I would like to know the name of your child.Is it the first time? Or did it happen before? .Is this his first time? . 2nd time: send the ambulance if: the seizure is > 15 minutes or > 2 attacks in 24 hours OSCE-guide-III.Emergency Medicine Phone calls THE SEIZING CHILD – PHONE CALL The mother is on the phone. your phone number and your address please! In case the line is disconnected. may I know who is on the line? .Are there other children at home? Any history of repeated illnesses at before? home? Repeated visits to ER? BINDE .Did you call the ambulance? I will ask the nurse to call the ambulance and we will send it to your home now .Before I proceed. hang on.Yes. I will need your help here. and if there is any of the medications can you take it out? . and do not call me.Can you check his mouth. you can just hold him/her? Analyze the event . analyze the event Or did this happen .If yes.Can you tell me what happened? . can you wash it with some water? . I will ask you some questions and give you some instructions.Emergency Medicine THE CHILD SWALLOWED MEDICATIONS / CAUSTIC MATERIAL The mother is on phone.If < 8 yrs 5 / 1 . please give them any bottle that the child might have taken medications from? . who is taking this medication? Why for? Was it recently renewed? Is there vitamins? Sleeping pills? Aspirin? When ambulance arrives.Can you check the child lips color? .Which medication did he take? If does not know. this is important for the sake of your child.Good evening.I would like to ask you to take a deep breathe to calm down. I will call you back.For how long was he/she left alone? Any reason? Is it the first time? .B: Was he a full term? Did he need special attention? Any congenital anomalies? . guide her: .doc Page 104 of 255 .Good to hear that .How much did he take? .Is he breathing? Talking to you? . start it now .N: Is he a fussy baby? .Do you know how to do CPR? .I: Are his/her shots up-to-date? .No.You need not to give him/her any thing by mouth .Is your child alert? + Stabilize Yes NO . ok. your phone number and your address please! In case the line is disconnected.If there are any remnants of the chemical on his mouth or face.What is your child weight? .D: Is he a difficult baby? PMH OSCE-guide-III.If > 8 yrs 15 / 2 ABC . I am Dr … the physician in charge at the (clinic/ER).You do not need to do anything. I would like to know the name of your child.Does he recognize you? . panicked as her child swallowed medication / caustic material at home Introduction + Reassurance . Primary survey . . Did you get a gush of air? o Yes. or who is responsible for this: encourage her and tell her that you are responsible . . neck / listen to lungs.Can you re-check the trachea centrality. .pre-arrangement with the place that will receive the patient OSCE-guide-III.Can you look to his chest. I did not do it before. anterior Axillary line C: Vital signs / fluids / withdraw blood samples / D: . ok. heart .I would like also to intubate the patient What are the requirements to transfer patient from a center to another center? . do not try to call me. or I am afraid I can not do it.Now.Open his mouth. may I know who is on the line? . you have just saved the patient life .I am Dr …. just hang the line and I will call you back. and give fluids (2 L NS bolus) and then withdraw samples for … .Accompanied by two trained medical personnel (paramedics.Can you please insert two wide bore (16 G) IV lines in the patient ante-cubital fossae. Can you feel air coming from patient mouth? .Can you tell me his Glasgow coma scale! Check his papillary reaction (D1).Intubated and on ventilator . I would like to get your phone number (and address if reasonably far). BP 90/60 and HR 120. the physician in charge in the emergency room.Secured IV lines and fluids . jugular veins and HR again for me please o If stable: can you please fix a chest tube in the 5th intercostal space. and make sure it is clear. 4 L/min via a nasal canula . and if by any chance the line gets disconnected.doc Page 105 of 255 . physicians) .Does he have a neck collar? .Are you a nurse? Do you have someone else with you to help? .Can you find any source of bleeding? .I am suspecting tension pneumothorax.Can you open the neck collar window and check his trachea and neck jugular veins. is it bilateral symmetrical movements of the chest wall? . nurses. Manage over the pho ne.What is his O2 saturation? Can you give him O2 please. in the MCL (on the upper margin of the third rib) o If she said. A/B: Mouth / O2 / inspect chest.Can you describe the case / situation for me! .Can you listen to air entry on both sides? . that is great. is the trachea central or not? Are the JV engorged? . I would like you to insert a wide bore needle (16 or 14 G) into the second intercostal space.Is the patient alert? . she has a patient of trauma after a car accident. check it for any FB or blood. and patient name.Emergency Medicine EMERGENCY TRAUMA – PHONE CALL A nurse is calling you from a remote rural medical center.D3: Give universal antidotes: o Give 100 mg thiamine o If ↓ RR  naloxone o If ↓ blood sugar dextrose . doc Page 106 of 255 .Physical Examination Physical Examination OSCE-guide-III. the temperature: ask about it specifically . the patient is stable.If vitals are given: based on the vitals. by palpation. by inspection …. percussion. listen (use simplified non-medical language) Report to the examiner – every thing – in medical terms.g.If SOB: during my exam.Vitals are not gives: o I would like to get the vitals before I start! o I am going to start my exam by measuring your vital signs that is your blood pressure. Also. make sure he will not fall. please do not hesitate to let me know and stop me . then I will do physical exam. if you feel that you can not continue. we reach a working plan Vital signs: . feel. I understand that you are here because you have been having … For the next few minutes I am going to do physical exam for your … and I will need to ask you questions during my exam.If the vitals are missing one. e. I will be asking you to do some movements and manoeuvres. if you feel any discomfort or pain. surround him Make sure you talk to the patient: I am going to look. heart rate.doc Page 107 of 255 . press. Hopefully towards the end. And I will start by measuring your heart rate OSCE-guide-III. … Cover the patient after you finish Thank the patient and the examiner Introduction: . I would like to proceed. please stop me If there is history taking and then physical exam: . tap.Good evening Mr …. Or the patient is unstable! Or comment: with mil fever . tell him that you will give pain medication as soon as you can If you ask the patient to walk. I am Dr … I am the physician working in the clinic today / I am the physician in charge in the ER now.If you have any questions or concerns please feel free to ask me / to bring it up .Physical Examination Medical Physical Exam ─ ─ ─ ─ ─ ─ ─ ─ ─ ─ ─ ─ ─ ─ Always be nice and smile Use the alcohol anti-septic Introduction Drape / cover the patient properly Warm your hands before touching the patient Warm your stethoscope If you are going to make painful manoeuvres: explain before / apologize for the pain / do not repeat Make sure you put the patient in the proper position at ease Whenever you notice the patient is in pain.I will be asking you some questions. Inspection: SEADS (scars / erythema / atrophy / deformity / swelling) / specific findings (bulk of muscles / bony symmetry) .If full CVS exam  peripheral vascular assessment: abdominal bruits / legs pulses palpation / chest exam Musculoskeletal examination:  Introduction / Vital signs / General inspection of the patient: pt is sitting comfortably … .Inspection: face / hand / neck / chest / back .doc Page 108 of 255 .Palpation: (TTC) tenderness / temperature / crepitus / effusion .Special test: mechanical (shoulder / elbow / hip / knee / ankle) o To complete my exam.ROM: active (if normal.Auscultation: in Z format A-P-T-M o Leg exam for edema o Lung bases .Auscultation: regular / special tests o Then end with cardiology exam Cardiac examination:  Introduction / Vital signs / General inspection of the patient: pt is sitting comfortably … .Inspection: face / hand / neck / chest / heart (PMI) .Percussion: dullness / percussion note / diaphragmatic excursion .Orientation: what is your name sir? Where are you? Time? Place? .Palpation: superficial / deep / special tests Respiratory examination:  Introduction / Vital signs / General inspection of the patient: pt is sitting comfortably … .Upper and lower extremities: o Inspection o Palpation / bulk o Tone o Motor power o Sensory o Reflexes .Cranial nerves .Cerebellar signs / Coordination . I would like to do:     Check the pulses of the limb (upper or lower) Brief neurological examination of the limb One joint above and one joint below examination The other side joint Neurological exams:  Introduction / Vital signs / General inspection of the patient: pt is sitting comfortably … . NO need to do the passive) / passive / against resistance .Palpation: apex / left para-sternal areas for heaves / valvular areas for thrills .Physical Examination Abdominal examination:  Introduction / Vital signs / General inspection of the patient: pt is sitting comfortably … .Cortical sensations: two points discrimination OSCE-guide-III.Gait / Romberg test .Percussion .Palpation: tenderness / tactile fremitus / chest expansion .Inspection .Auscultation: bowel sounds / bruits (aortic / renal / iliac) . no scars from previous surgeries. can you please cough  Check pt face for tenderness  there is cough tenderness  If no  cough another time  no herniation along the middle line. not distended.Physical Examination Abdominal examination: .doc Page 109 of 255 . pt is lying down comfortably. I am going to start away from there: o Percuss in 2 X 2 lines. umbilicus is inverted. I am going to listen to your abdomen. you can look back if you wish - Auscultation. Start away from the painful area:  I am checking (name the 4 quadrants or the 9 areas of the abdomen). normal contour. I will take a look at your abdomen: o By inspection: abdomen is flat. Thank you. (NO) tenderness.General inspection of the patient: o By general inspection. no obvious distress o Can I take a look at your eyes.Introduction . can you tell me which causes more pain! (any point except McBurney’s) OSCE-guide-III. I am going to tap on your abdomen. abd is moving with respiration. can you point to your painful are.I am going to drape you now! o Bed flat o Can you please put you hands to your sides o Allowing the patient to bend his/her knees so that the soles of their feet rest on the table will also relax the abdomen! - Inspection. guarding or rigidity o I am going to apply more pressure now: no obvious masses. Upwards please? No pallor o Would you please open your mouth: no signs of dehydration or vomiting o Can I take a look at your hand?  Temperature is fine / and skin is moist  Normal capillary refill (< 3 seconds)  No obvious clubbing . no obvious bulge or mass o Would you please look to the left side. would you please look downwards? No jaundice. warm my stethoscope: o McBurney’s point: normal bowel sounds (hyperactive / decreased) o 2 inches above the umbilicus  no aortic bruits o 2 inches above / 2 inches bilateral from umbilicus  no renal bruits o 2 inches below / 2 inches bilateral from umbilicus  no iliac bruits - Percussion: now. and percuss to side  for ascites o No percussion dullness / normal tympanic percussion note / no percussion tenderness / no ascites - Palpation: o I am going to feel your abdomen. no bruises. no organomegaly o I am going to feel your kidneys now (bimanual)  no enlargement.Vital signs . no tenderness of the kidneys o I am going to do some special tests:  Murphy’s sign (Rt costal margin)  can you take a deep breath  Rebound tenderness: I am going to press and release my hand. tell me if there is pain: • Not painful • Pain to the left side: left iliac tenderness / negative Rovsing's sign • Pain to the right side  positive Rovsing's sign /+/+/+/+/+/+/+/ COVER THE PATIENT ABDOMEN /+/+/+/+/+/+/+/  Psoas sign: Rt LL extended. I am going to: o Tap on your back.Spleen changes position with respiration (goes down with inspiration) .doc Page 110 of 255 . thank you.When he sits up  press on the sacro-iliac joints  sacroiliitis OSCE-guide-III. check the costo-vertebral angle for renal tenderness o Listen to the back of your lungs  no crepitation Can you please lie down. can you press up against my hand (actively flex thigh at the hip)  Obturator sign: Rt hip flexed. if pain  +ve sign - - Can you please sit up now. vaginal and bimanual exam. knee flexed. looking for:  Bleeding and/or discharge  Adnexal masses / tenderness  Cervical motion tenderness  To obtain samples for culture and sensitivity o Digital rectal exam. I would like to complete my exam by doing pelvic and vaginal exam / and digital rectal exam o Inguinal LNs o Pelvic. looking for:  Bleeding  Piles / haemorrhoids  Fissures How to differentiate between spleen and kidney by palpation: .Spleen does not ballot If there is history of Crohn’s disease: . I am going to press on your thighs down.Physical Examination   Press at McBurney’s point  is it painful Press left iliac area.You can feel the lower border of the spleen . I will rotate your leg (int and ext rotation). can you relax yourself.Inspection: o General: truncal obesity / moon-like face • Red eyes • Mouth: ulcers o Hands: nail changes / clubbing / skin rash / psoriatic changes / joint tenderness o Abdomen: stria / scars o Lower limbs: erythema nodosum o DRE: fissures / fistulae . General: o o o o o o o - Inspection: o - o 2 X 2 lines tap Liver: MCL (from above downwards and from down upwards) liver span Spleen: ant axillary line  last intercostal space / ask pt to take deep breathe in / then repercuss for the spleen Ascites: from midline to the side. support from left back. time and place . Spleen: patient elevates his LEFT side 45°. I would like to ask for protection for me and my team please (gloves / gowns / masks / goggles) . and not nodular. INR. while the patient is breathing in and out (push during inspiration. no dullness  so there is no need to perform the shifting dullness (to be clinically palpable: ascites > 500 ml) Palpation: o o - Listen to liver: no hepatic rub / hum / or bruits Listen to spleen: no rub / no hum Percussion: o o o - No bruises / normal hair distribution / no caput medusa / no dilatation of collateral veins / no obvious ascites / no scratch marks Auscultation o o - Patient is lying comfortably no signs of obvious distress Eyes  sclera: no jaundice Mouth: no fetor hepaticus / no clots / no vomitus Face: no parotid gland enlargement Hands: no clubbing / nail changes / palmar erythema / Dupuytren’s contracture / atrophy of thenar and hypothenar muscles / look for IV marks / stretch your hands please (flapping tremors) Can you please lower your gown: no gynecomastia / no spider nevi Check lower limbs edema Liver: start from the right iliac fossa and go upwards. Start from above the umbilicus towards the spleen  spleen is not palpable I would like to complete my exam by doing: o Digital rectal exam: for hemorrhage / piles o Check for testicular atrophy     DD of hepatitis: (alcoholic / viral / drug-induced) / cirrhosis / liver cancer Worst prognostic signs: ascites / encephalopathy Investigations: AST:ALT > 2:1. perform focused examination for liver cell failure . do not move your hand from the patient)  margin of liver is not palpable.doc Page 111 of 255 .Physical Examination Liver Examination Patient vomited blood 20 minutes ago.Because the patient is vomiting blood. GGT. albumin. CBC Treatment of encephalopathy: treatment of precipitating causes / Lactulose (15-30 ml tid) / Rifaximin (550 mg PO bid)  DD of ascites: liver cirrhosis / renal failure / heart failure / TB / malignancy / Hypoalbuminemia Treatment of ascites: Sodium restriction (20-30 mEq/d) / Diuretic therapy / Therapeutic paracentesis may be performed in patients who require rapid symptomatic relief for refractory or tense ascites  OSCE-guide-III.Orientation: I am going to ask you some questions which are part of my physical exam. Do you know where you are now? What is the time? Do you know why you are here?  Patient is oriented to people. not tender.Introduction / Vital signs . Physical Examination Nasal bleeding  Hematological Examination - Because the patient is bleeding. along the groin crease)  Deep: • Transverse: along the groin crease • Longitudinal: medial to the femoral artery (Lat  Med: NAVAL – Nerve Artery Vein LNs) o I would like also to check the popliteal LNs Check for bony pains: sternum and 3-5 spots on the vertebral column (I am going to press on your back) - CAN YOU LIE DOWN PLEASE? .doc Page 112 of 255 . I would like to do: o Fundoscopic retinal exam o Orientation o Brief neurological exam:  Hands: pull/push/ do you feel the same  Legs: pull/push/ do you feel the same Patient is grossly neurologically free N.Abdominal exam: o Inspection o Auscultation o Percussion: liver / spleen o Palpation: liver / spleen o I would like to: perform vaginal and pelvic exam - Finally.Listen to heart / lungs . OSCE-guide-III. focal neurological deficits to DD between TTP and HUS. I would like to ask for protection for me and my team please (gloves / gowns / masks / goggles) - Introduction Vital signs - General: o Patient is lying comfortably no signs of obvious distress  Nose: open the speculum antero-posteriorly  Use the otoscope for ENT (nose / ear / mouth) o Look for bruises / petichae  if you find them  continue hematological exam - I ma going to feel your glands: o Occipital / post-auricular / pre-auricular / sub mandibular / sub mental / anterior / posterior cervical / supra-clavicular o I would like to check the axillary LNs (ant / post / med / lat / apical) o I would like to check the inguinal LNs:  Superficial (transverse.B. focal deficits. increased LDH. decreased haptoglobin  Negative Coombs' test  Creatinine. giant platelets • Bleeding time: increased / PT and aPTT: normal • Anti-platelets antibodies o Bone marrow: increased number of megakaryocytes (critical test to rule out other causes of thrombocytopenia for age > 60 years.g. meningococcus. aPTT. HUS)  PT. urea. fibrinogen: normal  Markers of hemolysis: increased unconjugated bilirubin.Investigations: o CBC: thrombocytopenia o Peripheral blood film: decreased platelets.DD: o ITP (Immune Thrombocytopenic Purpura) o TTP (Thrombotic Thrombocytopenic Purpura) o HUS (Hemolytic Uremic Syndrome): ITP TTP HUS  Remitting / relapsing  Predominantly adults  Predominantly children course  Thrombocytopenia  Severe thrombocytopenia  Mild fever  Micro-angiopathic  Micro-angiopathic hemolytic anemia hemolytic anemia  Splenic discomfort (MAHA) (MAHA) (mild engorgement)  Renal failure  Renal failure  Neurological symptoms (headache.Treatment: o Steroids (methylprednisolone 1 g/d for 3 days. decreased haptoglobin o Kidney function tests (urea / creatinine for HUS) . myelodysplasia) o Markers of hemolysis: increased unconjugated bilirubin. seizures)  Fever  CBC and blood film: decreased platelets and schistocytes Investigations (both TIP.e. to follow renal function  Stool C+S (HUS)  Plasmapheresis ± steroids Management (both TIP. e. HUS)  Platelet transfusion is contraindicated (increased microvascular thrombosis)  Plasma infusion: if plasmapheresis is not immediately available  TTP mortality – 90% if untreated OSCE-guide-III. isolated thrombocytopenia with no clinically apparent cause) .Physical Examination Questions: . confusion. then prednisone 1.5 mg/kg/day) o Immunoglobulins (if low platelet count): IVIG 1 g/kg/d X 2 days o Splenectomy o Vaccination (pneumococcus.doc Page 113 of 255 .Diagnosis: o ITP (Immune Thrombocytopenic Purpura)  Most common cause of isolated thrombocytopenia  Diagnosis of exclusion (i. HIB) . increased LDH. I am going to feel your chest o Tenderness: ▬ o Check the ant chest wall: no tenderness Tactile fremitus: can you say 99 for 4-5 times. I am going to demonstrate the special tests that should be done if there is consolidation. comment on respiratory distress .Vital signs.Auscultation.doc Page 114 of 255 .Palpation.General inspection of the patient. I am going to listen to your lungs o Mid clavicular line – both sides o Normal vesicular breathing sounds bilaterally. swellings.Physical Examination Chest Examination . then I will check the front again /+/ Can you cross your arms please .Now.Percussion. no rhonchi. I am going to tap on your chest o Apex (2) / back (6) / sides (2)  normal percussion note: . I am going to feel your chest o Tenderness: no tenderness o Tactile fremitus: can you say 99 for 4-5 times. no wheezes . no intercostal retraction Look from the side: no increase in the A/P diameter of the chest / no barrel chest Palpation. now.Percussion.Inspection: chest is symmetrical. no accessory ms used for breathing. especially: tachypnea / temperature . can you lower your gown please ▬ o Trachea is central. I am going to listen to your back o Whenever you feel the stethoscope touching your back. or deformity . I am going to tap on your chest – on both sides o Apex (2) / MCL (6) / sides (2)  normal percussion note: . whenever you feel my hands on your back .Cardiology exam: Auscultation OSCE-guide-III. no nasal flaring / laboured breathing Eyes: jaundice / pallor Mouth / can you flip your tongue please: no central cyanosis / dehydration / exudates or secretions Hands: no nicotine staining / clubbing / peripheral cyanosis / normal capillary refill Chest is symmetrical. whenever you feel my hands on your chest  normal tactile fremitus ▬ ▬ Any lung pathology  ↑ tactile fremitus (↑ conduction) Any pleural pathology  ↓ tactile fremitus (insulation o Chest expansion: normal chest expansion /+/ I will continue to examine you from the back. can you please take a deep breathe in and out from your mouth o Normal vesicular breathing sounds bilaterally. no scars.Introduction . no rhonchi. no wheezes .Inspection: o General: ▬ ▬ ▬ ▬ o Neck. no accessory ms used for breathing / LNs Chest: ▬ ▬ - Face: symmetrical.Auscultation. now. with auscultation: o Can you say letter “E”  egophony (normal: ee / over affected area: ay) o Can you say “99”  ↑ vocal fremitus o Whisper 1/2/3: whispering pectorology (↑ in audibility) /+/ I will now go to examine you from the front again /+/ . 3. OR THEN 250 mg qDay for 4 day Doxycycline (total of 5 days)  Clarithromycin 500 mg PO bid for 7-14 days Comorbidities.doc Page 115 of 255 .375 gm IV q6h) ─ MRSA: Vancomycin 1 gm IV q24h Pneumococcus vaccination: ─ Elderly patients > 65 yrs old ─ COPD ─ CHF ─ Cirrhotic ─ Cancer ─ Immunocompromised: steroids / HIV / DM / splenectomy ─ Leukemia / lymphoma 9 Especially in the presence of risk factors: group home / hospital infection / immunocompromised OSCE-guide-III.Physical Examination Pneumonia ▬ ▬ CURB 65 score – Pneumonia Clinical Prediction Tool Confusion Urea: > 7 mmol. co-morbidities …). or Beta-lactam + Macrolide  Levofloxacin 750 mg PO q24h use of antibiotics OR for 5 days in last 3 months Respiratory fluoroquinolones OR  Amoxicillin 1000 mg PO tid Inpatient Ward + Clarithromycin 500 mg PO bid ICU9 3rd gen cephalosporin + Ceftriaxone 1 g IV q24h Macrolide + Azithromycin 500 mg IV q24h OR for 5 days (step to oral when Beta-lactam + Macrolide tolerated) Other medications for the management of pneumonia: ─ Susceptible for pseudomonas / recent use (within 3 months) of antibiotics or cortisone: piptazo (piperacillin / tazobactam). BUN > 19 Respiratory rate: > 30 breaths / min Blood pressure: systolic < 90 and diastolic < 60 65: or older in age Management 0–1 Can treat as outpatient 2–3 Consider hospitalization 4–5 Consider ICU Mortality <5% <10% <30% Pneumonia severity index: another clinical index (scored on age. IDSA / ATS Community Acquired Pneumonia Treatment Guidelines 2007: (IDSA: Infectious Diseases Society of America. ATS: American Thoracic Society) Setting Circumstances Treatment Medications Outpatient Previously well Macrolide  Azithromycin 500 mg PO once. used to determine whether to admit the patient to the hospital or not. resp rate. doc Page 116 of 255 .Physical Examination Case: HIV positive man – C/O: SOB for 1 week / cough / fatigue ▬ ▬ ▬ ▬ ▬ ▬ ? Pneumonia (? PCP) DD of unilateral lobar reticular pattern on CXR: PCP (pneumocystis pneumonia) / Kaposi sarcoma in the lung / Lymphoma / Atypical pneumonia (mycoplasma / chlamydia) DD of bilateral hilar infiltrates: PCP / TB / CMV Signs of HIV: check the entire skin for Kaposi sarcoma / mouth and pharynx for thrush or oral hairy leukoplakia / LNs / check the abdomen for liver and spleen Investigations: o O2 saturation / ABG o CBC / differential / CD4 count o LDH (↑ in 95% of PCP cases) o Blood culture o Sputum for cytology / culture / gram stain / acid fast stain o Bronchoscopy: bronchial washing Treatment of PCP: o Septra DS (trimethoprim / sulfamethoxazole: 160mg/800mg): 2 tablets q8h x 14 days then continue prophylactic 1 tab od o Severe illness (PO2 < 70 mmHg):  Admit  O2  Septra  Steroids: prednisone 40 mg bid x 5 d then 40 mg od x 5 d then 20 mg od x 5 d o If patient is allergic to TMP/SMZ:  For treatment: IV pentamidine or atavaquone  For prophylaxis: Dapsone or atavaquone HIV routine prophylaxis: o Based on CD4 count:  CD4 < 200  prophylaxis for PCP: TMP/SMZ or Dapsone  CD4 < 100  prophylaxis for toxoplasma: TMP/SMZ or Dapsone  CD4 < 50  prophylaxis for mycobacterium avium complex: azithromycin (1/week) or clarithromycin (2/d) o Regardless the CD4 count:  PPD testing for TB. considered positive if > 5 mm induration  INH for 9 months  Vaccinations for pneumococcus / influenza / hepatitis B OSCE-guide-III. I am going to assess your blood pressure twice. S4 at the apex (with the bell)  Aortic insufficiency murmur  Lung bases Pulsation / Bruits (aortic / renal / iliac) Inspection / Palpation (temperature / pulsations) / LL BP Abdominal exam Lower limbs exam .Physical Examination Cardiac Examination – Essential HTN case + 65 years old gentleman. during which I am going to ask you some questions: OSCE-guide-III. (feet dangled)? I need you to relax for 2 minutes. one while lying down.Vital signs. then increase 30 mmHg.  Look for S3. for follow-up visit + 35 years old gentleman with newly diagnosed high blood pressure. then auscultate  Orientation  Mouth for dehydration  Eyes for pallor  Lung bases auscultation  Sacral edema  Lower limb edema No difference  Asses the radial pulse bilaterally  Hands: capillary refill / clubbing / temperature  Fundoscopic examination: disc edema / retinal hge  Carotids: bruits / palpate  JVP: measure / Kussmaul sign / hepato-jugular reflux  Symmetrical / PMI  Apex  Heaves (Lt para-sternal / with knuckles)  Thrills (A-P / with palmar aspect of distal phalanges of fingers) Do timing by simultaneously checking the carotid pulse. your HR today is … which is normal  Do the BP: get the systolic by pulse palpation. then increase 30 mmHg. especially: BP / HR o Ask the patient to lie down. please assess + If 10 minutes case: vitals + examination + If 5 minutes: only examination Analysis ▬ ▬ Impact ▬ Red flags ▬ Introduction HR and BP (lying down) Sit up. no murmurs.  Areas A-P-T-M: normal S1.Introduction . the other after you sit up  I will start by measuring HR for 10 seconds. then auscultate  your BP today is … o Can you please sit up on the edge of the bed. before I proceed to other measurements.doc Page 117 of 255 . with high blood pressure for 20 years. relax for 2 minutes HR and BP (sitting up) General Neck Heart Carotid JVP Inspection Palpation Auscultation For how long? Medications? How frequent do you check your BP? Is it controlled? End organ damage? Proteinuria / visual changes / headaches / confusion? Other cardiovascular risk factors: smoking / DM / high cholesterol / history of MI in the family Can you please lie down Get the systolic by pulse palpation. S2. no ↑ 20 in HR) General exam: o Comment on the general status / respiratory distress of the patient o Assess the radial pulse bilaterally: pulse is normal. check the apex for S3. normal volume and contour JVP:   - Orientation: Do you know where you are? Why? Date? Mouth for dehydration Eyes for pallor Can I listen to the back of your lungs (lung bases)  no crackles Press against the sacrum for sacral edema Press on the lower limbs. constrictive pericarditis) Press on the liver for 10 seconds  normally the JVP will ↑ for few seconds then will ↓  if it does not ↓  hepato-jugular reflux Chest inspection: o Chest is symmetrical. no obvious pulsations o PMI (point of maximum intensity) is not obvious / or is obvious Palpation: o Palpate the apex and locate it  apex is palpable at … in the MCL. with no delay o Hands: normal capillary refill / no clubbing / warm moist skin o I would like to do fundoscopic examination (I am going to shine a light in your eyes to examine the back of your eyes): looking for disc edema / cotton wool exudates / retinal hemorrhage / nipping of the veins I am going to put the bed in 45° position. and lean forward  no enhancement of aortic insufficiency murmurs o Listen to the lung bases  no basal lung crackles I am going to put the bed flat now: o Abdominal exam: inspect for pulsations / auscultate for bruits o Lower limb exam:    OSCE-guide-III. regular. no murmurs o Flip the bell. for edema Measure the JVP Kussmaul test: take a deep breathe and hold it. S2. not displaced o Palpate the left para-sternal area by knuckles: no Rt ventricle heave o Palpate the Aortic / Pulmonary areas by finger tips  no thrills Auscultation: o In Z format A-P-T-M: normal S1. equal on both sides.Physical Examination       o - - Measure the sitting HR and BP  no difference in readings  no orthostatic hypotension (no changes: no ↓ 20 mmHg in systolic. exhale it out and hold. measure the JVP: • Normally it will ↓  negative Kussmaul test • If ↑ JVP  positive Kussmaul test (restrictive cardio-myopathy. no ↓ 10 mmHg in diastolic.doc Inspection: symmetry / SEADS / nails hypertrophy / skin changes Palpation: temperature / pulsations / radio-femoral delay I would like to measure the blood pressure in lower limb Page 118 of 255 . not hyperdynamic (vol ovrld). S4 o While listening to the left para sternal area: can you please take a deep breathe in. examine your neck: o Carotids:    o  - - - Auscultate for bruits  use the bell If no bruits  palpate the carotid arteries (ONE AT A TIME) Pulse is regular. not sustained (pres ovrld). S4 at the apex (with the bell)  Listen to lung bases Abdominal exam  Inspection: no obvious masses. Stretch your hands please  fine tremors (hyperthyroidism)  Fundoscopic examination: disc edema / retinal hemorrhage Neck Thyroid  No masses  Palpate for the thyroid  Percuss for the retro-sternal thyroid dullness JVP  JVP Chest  Palpate the apex  Look for S3.Physical Examination Secondary Hypertension A 25 years old young man with HTN. Cardiac  Coarctation of the aorta  Cocaine (nasal septum) Renal  Poly-cystic kidney  Renal artery stenosis Endocrine  OCPs  Cushing disease / syndrome  Pheochromocytoma  Primary hyperaldosteronism  Hyper / hypo thyroidism Patient disrobed to underwear. draped below waist Introduction Can you please lie down Vitals  There is systolic and diastolic HTN  I want to rule out orthostatic hypotension (pheochromocytoma)  I would like to compare UL and LL (coarctation of aorta)  No tachycardia (pheochromocytoma / hyperthyroidism)  No bradycardia (hypothyroidism) General  Orientation (brain edema)  Truncal obesity / cervical fat pad / moon face (Cushing)  Check for proximal ms weakness (Cushing)  Check the eyes from the side for proptosis.doc Page 119 of 255 . lid lag test (hyperthyroidism)  Asses the radial pulse bilaterally  Hands: capillary refill / temperature / skin dry or wet (thyroid). no café au lait spots  Auscultation: bruits (aortic / renal)  Palpation: feel for renal / supra-renal mass Lower limbs exam  Edema  Rule out femoral / radial delay Brief neuro exam  Knee or ankle reflex ▬ Notes: ▬ ▬ Watch for labile white coat HTN Any HTN in middle age  secondary HTN  most likely kidney disease OSCE-guide-III. no stria. e.  Available combinations: Altace plus (ramipril + diuretic) / Diovan H o Notes on ACEIs:  Contraindications of ACEIs: Angio edema / Bilateral renal artery stenosis  ACEIs are nephroprotective except in acute renal injury  nephrotoxic  If patient on ACEIs developed cough  switch to ARBs ─ HTN emergency: Hydralazine: 20 – 40 mg IV or IM. 4-6 times/week ─ Pharmacological: o First line: Diuretics. within 4 weeks o If (target organ damage OR diabetes mellitus OR chronic kidney disease OR blood pressure > 180/110 mmHg)  diagnose HTN o Else (BP: 140-179 / 90-109 mmHg)  24 hours BP monitor (diagnose if mean awake sBP >135 mmHg and/or dBP > 85 mmHg or mean 24 hours sBP >130 mmHg and/or dBP > 80 mmHg) Management: ─ Target BP is < 140/90 mmHg. LDL. add another first-line drug  Do NOT give β-blockers and Ca ch blockers  may cause heart block  Do NOT give ACE and ARBs  both ↑ K+.g.g. confusion)  diagnose HTN o Else (provided 2 more readings during same visit)  Search for target organ damage: history (cardio-vascular risk factors) / examination  Investigations: ▬ CBC / Na+. hydrochlorothiazide 12.doc Page 120 of 255 . HDL.5-24. Ramipril 2. < 130/80 if DM or chronic kidney disease ─ Life style modifications (initial management): o Smoking cessation and decrease alcohol consumption o Diet: ↓ salt / ↓ cholesterol and saturated fats / follow Canada's Guide to Healthy Eating o Weight: maintain healthy BMI (18.5 – 5 mg PO od  Gout: Amlodipine (5 mg PO od) OR Candesartan (4 – 8 mg PO od)  Elderly (especially if IHD): ─ ACEIs ─ β-blockers: metoprolol 25 mg bid Especially if CHF / EXCEPT: asthma / bradycardia  Pregnant: ─ Hydralazine: 10 mg PO qid for few days then 25 mg PO qid ─ OR α-methyl dopa: 250 mg PO bid  If > 3 cardiovascular RF: statins / ASA o If partial response to standard dose monotherapy.Physical Examination Hypertension  ─ ─ ─ ─   Predisposing Factors Family history ─ Sedentary lifestyle ─ Excessive salt intake / Obesity ─ Smoking fatty diet Alcohol consumption ─ Male gender ─ African American Stress ─ Age >30 ─ Dyslipidemia Diagnosis: ─ Visit ONE: o If hypertension urgency or emergency (sBP > 210 or dBP > 120) or end organ damage (e. TG) ▬ Kidney function tests / Urinalysis / Renal Doppler ▬ ECG / Echocardiogram ▬ For secondary HTN: TSH / Plasma aldosterone / renin levels / 24 hours urine for metanephrines / VMA  Life style modifications (↓ salt / ↓ alcohol / ↓ cholesterol / exercise)  Follow-up visit within 4 weeks ─ Visit TWO.5 – 25 mg PO od Except:  DM: ACEIs. repeated as necessary.9) o Moderate intensity dynamic exercise: 40-60 minutes. decrease the dose in case of renal impairment OSCE-guide-III. K+ / fasting blood sugar / lipids (total cholesterol. OSCE-guide-III. and there is theoretical possibility of dislodging the DVT. and also negative Homans' sign does not rule out the DVT diagnosis (poor negative predictive value).Physical Examination SOB – shortness of breathe Patient who had a car accident 24-48 hours ago developed SOB. 3rd day: DVT / PE Brief cardio-pulmonary hx Vitals General Neck Chest Trachea JVP Inspection Palpation Percussion Auscultation Lower limbs exam Common causes: Cardiac (CHF) / Lungs (asthma / COPD / pneumonia / PE) Life threatening conditions: MI / PE / pneumothorax / aortic dissection I would also like to rule out orthostatic hypotension (? hypovolemia)  General assessment of the patient  Orientation (SOB  hypoxia  confusion)  Face: eyes / no signs of respiratory distress / mouth (no signs of dehydration / central cyanosis)  Hand: temperature / capillary refill / skin / no peripheral cyanosis / no nicotine staining / comment on both pulses  Sacral edema  Trachea centrality  JVP: not engorged  Symmetry  Accessory muscles breathing / intercostal retraction  Obvious pulsations / PMI  If car accident (bruises on chest wall)  Tenderness  Tactile fremitus  Apex  Para-sternal heaves  Chest back for any dullness  Heart (ask the patient to turn slightly to left)  Lungs: 8 spots scan for air entry  Lung bases  Inspection: SEADS  Palpation: temperature / capillary refill / peripheral edema / pulses / press for pain in calf muscle area  Special tests: .Homans' sign10: there is pain in the calf or popliteal region with examiner's abrupt dorsiflexion of the patient's foot at the ankle while the knee is flexed to 90 degrees Indications for intubation: ABG showing poor PO2 (60s) / elevated PCO2 (80s) / acidosis / GCS score < 8 10 A positive Homans' sign does not positively diagnose DVT (poor positive predictive value).doc Page 121 of 255 . Complications: 1st day: atelectasis / fat embolism. difference should be < 2.Measure leg circumference 10 cm below tibial tuberosity.5 cm between the two calves . antiphospholipid antibody syndrome. SERMs) DD: muscle strain or tear. aging.Physical Examination DVT Clots clinical probabilities / risk factors (Virchow's Triad): ─ Vascular injury (endothelial damage). cellulitis. obesity. chronic venous insufficiency) inhibits clearance and dilution of coagulation factors ─ Hyper-coagulability. or within 6 mo or palliative) +1 Paralysis or recent plaster immobilization of the lower extremities +1 Recently bedridden for >3 d or major surgery < 4 wk +1 Localized tenderness along the distribution of the deep venous system +1 Swollen unilateral superficial veins (non-varicose) +1 Entire leg swelling +1 Calf swelling >3 cm compared with the asymptomatic leg +1 Pitting edema (greater in the symptomatic leg) +1 Previous DVT documented +1 Alternative diagnosis (as likely or greater than that of DVT) -2 A Wells score can be interpreted in a binary (likely vs. unlikely) or ternary (low. lymphangitis or lymph obstruction. or high probability) fashion. stroke. malignancy. invasive and higher risk OSCE-guide-III. HRT. ─ Venography is the gold standard. ─ Other non-invasive tests include MRI. ruptured popliteal cysts. trauma. venous valvular insufficiency. recent surgery ─ Venous stasis.doc Page 122 of 255 . and arterial occlusive disease Wells Clinical Score for Deep Venous Thrombosis Clinical Parameter Score Active cancer (treatment ongoing. long travel or flight. surgery.g. e. post-operative. OCP. moderate. Ternary interpretation Binary interpretation ≥3 High probability score ≥ 2 DVT likely 1 or 2 Moderate probability score < 2 DVT unlikely ≤0 Low probability In the unlikely group (low-to-moderate risk of DVT)  D-dimer assay (ELISA): ─ Negative  rule-out DVT ─ Positive  duplex ultrasonography o Negative  rule-out DVT o Positive  treat for DVT In the likely group (moderate-to-high risk of DVT)  duplex ultrasonography ─ Negative  repeat clinical evaluation and ultrasonography in 1 week ─ Positive  treat for DVT When discordance exists between the pretest probability and the duplex ultrasonographic study result. but is expensive. CHF. hormone related (pregnancy. further evaluation is required. immobilization (post-MI. o Warfarin should be dosed to maintain INR at 2-3. recurrent thromboembolism despite adequate anticoagulation.5 Heart rate >100 beats per minute + 1. warfarin should be discontinued for at least 4 days pre-op.000 ill). Disadvantages: must monitor aPTT ─ Low molecular weight heparin (LMWH): administered SC. cancer. or those who require emergent surgery without time to initiate anticoagulation ─ Special considerations: o Pregnancy: treat with LMWH during pregnancy.5 Clinical signs of deep vein thrombosis +3 Alternative diagnosis less likely than pulmonary embolism + 3 Hemoptysis +1 Cancer (treated within the last 6 mo) +1 Score 0-1 2-6 ≥7 Clinical Probability of PE Low Intermediate High OSCE-guide-III. then warfarin for 4-6 weeks post-partum o Surgery: avoid elective surgery in first month after venous or arterial thromboembolic event ─ Preoperatively: IV heparin may be used up to 6 hrs pre-operatively. o Should be initiated with heparin overlap – dual therapy for at least 5 days. then every 2-4 weeks o Duration of anticoagulant treatment (with warfarin unless otherwise noted): ─ First episode DVT with transient risk factor: 3 months ─ First episode DVT with ongoing risk factor (e.0 for two consecutive days. Surgery safe when INR <1. antiphospholipid antibody) or > 1 risk factor: consider indefinite therapy ─ Recurrent DVT (2 or more episodes): indefinite therapy ─ IVC filters: useful in those with contraindications to anticoagulant therapy. Discontinue heparin after INR >2. recurrent embolism with pulmonary HTN.Physical Examination Initial treatment of DVT: ─ Unfractionated heparin (UFH): requires bolus (7500-10. Monitor INR twice weekly for 1-2 weeks. then weekly until INR stable. the guideline notes that the Wells rule performs better in younger patients without comorbidities or a history of venous thromboembolism) Clinical Characteristic Score Previous pulmonary embolism or deep vein thrombosis + 1.5 ─ Postoperatively: IV heparin or LMWH can be started 12 hours after major surgery Wells Prediction Rule for Diagnosing Pulmonary Embolism For Predicting Pretest Probability of PE (N.5 Recent surgery or immobilization (within the last 30 d) + 1. followed by continuous IV infusion (1000-1500 IU/h). Advantages: rapidly reversible by protamine. Predictable dose response & fixed schedule Long-term treatment: ─ Warfarin: standard treatment.doc score > 4 score ≤ 4 Simplified Wells PE likely PE unlikely Page 123 of 255 .g.B. high bleeding risk o Duration of long-term anticoagulation treatment: ─ If reversible cause for PE (surgery.000 U) followed by infusion starting at 20 U/kg/hr – aim for aPTT 2-3 times control ─ Long term anticoagulation: o Warfarin – start the same day as LMWH/heparin – start at 5 mg PO od – overlap warfarin with LMWH/heparin for at least 5 days and until the INR is in target range of 2-3 o LMWH instead of warfarin for pregnancy.doc Page 124 of 255 . aPTT ± renal function ± liver function o For SC LMWH: dalteparin 200 U/kg once daily or enoxaparin 1 mg/kg bid – no lab monitoring – avoid or reduce dose in renal dysfunction o For IV heparin: bolus of 75 U/kg (usually 5. go straight to spiral CT or V/Q 2. prevents new clots and allows endogenous fibrinolytic system to dissolve existing thromboemboli over months o Get baseline CBC. injury. Use D-dimers only if low clinical probability. INR. pregnancy. works better than IV thrombolytic therapy and fewer contraindications) ─ IVC filter: only if recent proximal DVT + absolute contraindication to anticoagulation OSCE-guide-III. otherwise. active cancer. If using V/Q scan (CT contrast allergy or renal failure): ─ Negative V/Q scan rules out the diagnosis ─ Inconclusive V/Q scan requires leg US duplex to look for DVT (q2d) ─ High probability V/Q scan only rules in the diagnosis if have high clinical suspicion CXR of PE: may be normal / wedge-shaped infiltrate / unilateral effusion / raised hemidiaphragm Treatment of PE: ─ Admit for observation (patients with DVT only are often sent home on LMWH) ─ Oxygen: provide supplemental O2 if hypoxemic or short of breath ─ Pain relief: analgesics if chest pain – narcotics or NSAIDs ─ Acute anticoagulation: therapeutic-dose SC LMWH or IV heparin – start ASAP o Anticoagulation stops clot propagation.Physical Examination Evaluation of suspected PE: ─ Low clinical probability of PE  D-dimer: o Negative: ruled out o Positive  CT scan with contrast: ─ Negative: ruled out ─ Positive: ruled in ─ Intermediate OR high probability: o CT scan: ─ Negative: ruled out ─ Positive: ruled in Notes: 1.): 3-6 months ─ If PE unprovoked OR ongoing major risk factor (active cancer): indefinite ─ IV thrombolytic therapy: o If patient has massive PE (hypotension or clinical right heart failure) o Hastens resolution of PE but may not improve survival or long-term outcome ─ Interventional thrombolytic therapy (massive PE is preferentially treated with catheter directed thrombolysis by an interventional radiologist. etc. this is how it feels. Then dangle the feet and check the color  no pallor on elevation. check the color of the foot. if you feel any pain / numbness / tingling please let me know.Physical Examination Peripheral Arterial Disease Examination Introduction Vitals General Inspection Palpation Abdomen Neurology Burger test Special tests Based on the vitals. can you please close your eyes! Tell me when you feel it touching you! Check both lower limbs from distal to proximal  If light touch is ok. I would like to arrange for ankle / brachial index  Investigations: ▬ CBC ▬ Fasting blood sugar / lipid profile ▬ ECG ▬ Angiography (side effects: nephrotoxic / allergy / aneurysm risk) ▬ Doppler U/S study of the arterial tree both lower limbs Treatment: ▬ Life style modifications (refer to HTN) ▬ Foot care ▬ Graded exercise ▬ Surgery (if severe disability) OSCE-guide-III. no rubor on dependence  Burger test is negative. here is a piece of cotton.doc Page 125 of 255 . do not proceed with more tests I would like to raise your legs. I would like to proceed … Drape the patient / expose the lower limbs (triangular)  I would like to take a look at your feet. for 1-2 minutes. do you want me to help you?!  SEADS  No signs of arterial insufficiency: no hair loss / no shiny tight skin / no hypertrophic nails  Temperature  Capillary refill (< 3seconds)  Pulses: dorsalis pedis / posterior tibial / popliteal / (to examiner) I would like to check the femoral arteries  Drape the patient / I would like to examine your abdomen / can you please uncover your abdomen  Listen for bruits (aortic / renal / iliac)  Check for light touch. can you please remove the socks. the patient is stable. Then move to the next joint. medial malleolus. the patient is stable. tell me when it stops  intact / decreased / absent Monofilament test: to distinguish between the light touch and pressure sensation / 10 points on the foot (9 on the sole.doc Drape the patient / expose the lower limbs (triangular) I would like to take a look at your feet. no active bleeding or oozing No other ulcers in the same foot / check the other foot / check in between toes  no evidence of infection in between toes / in nails No pigmentation around the medial and lateral malleoli SEADS – quadriceps wasting / swollen joints No signs of arterial insufficiency: no hair loss / no shiny tight skin / no hypertrophic nails Temperature Capillary refill (< 3seconds) Pulses: dorsalis pedis / posterior tibial / popliteal / (to examiner) I would like to check the femoral arteries Drape the patient / I would like to examine your abdomen / can you please uncover your abdomen Listen for bruits (aortic / renal / iliac) Check for light touch. shape (round. here is the sensation you will feel. Start with head of metatarsal. I would like to proceed …        Palpation    Abdomen   Neurology        Burger test Special tests Balance (if DM focused exam case) OSCE-guide-III. can you please remove the socks. … Vibration: tuning fork. do you want me to help you?! Describe the ulcer: location (in the sole at base of 1st metatarsal). irregular). while with closed eyes: pt loses balance Page 126 of 255 . do not proceed with more tests Light touch sensation is absent distal to the level of … cm above ankle Proprioception: I will move your toe.Physical Examination Diabetic Foot Diabetic patient with long hx of diabetes. this is how it feels. and one on the dorsum above the big toe meta-tarso-phalangeal joint) Ankle reflex. margins not elevated. size (… cm). can you please close your eyes! Tell me when you feel it touching you! Check both lower limbs from distal to proximal If light touch is ok. has an ulcer for few days Introduction Vitals General Inspection Based on the vitals. tibial tuberosity. close your eyes please. tell me is it up or down. if you have time: knee reflex and Babinski I would like to arrange for ankle / brachial index   Gait ± ataxia Romberg test: positive (with open eyes: pt can balance himself by vision. here is a piece of cotton. Physical Examination Neurological Examination o o o - Introduction Vital signs General inspection of the patient: pt is sitting comfortably … Orientation: what is your name sir? Where are you? Time? Place? Cranial nerves Upper and lower extremities: o Inspection o Palpation / bulk o Tone:  Just relax please, let me do everything for you. I am going to check the tone in your Rt arm …  Tone is normal, no hypo or hyper tonia o Motor power (5  0)  5 full power  4 less than full power (like Lt hand in Rt handed person)  3 can do the movement against gravity  2 can do the movement with the gravity eliminated  1 muscle twitches, not able to initiate movements  0 no power – no movements o Sensory:  Light touch: • Pin prick or piece of cotton • First check on forearm or sternum • Can you close your eyes please • Distal to proximal • Bilateral  sensation is equal bilaterally  Posterior column (B12 deficiency / alcohol / syphilis): • Vibration sense: tuning fork / test on sternum / tell me when it stops / start distal / if intact move on / if not intact go proximal on the next joint • Proprioception: eyes closed / start with the big toe or thumb / is it moving or not? / is it up or down? o Reflexes:  0 absent  1 weak (hyporeflexia)  2 normal  3 hyper reflexia  N.B. Babinski reflex: I am going to tickle the bottom of your foot: • Planter flexion: normal response • Big toe dorsiflexion and toes fanning: UMNL (e.g. stroke) OSCE-guide-III.doc Page 127 of 255 Physical Examination - - - - Gait – ATAXIA o Can you take few steps for me please? o Protect the patient, surround him with your arms, and walk with him Romberg test o Can you put your legs together! o Can you close your eyes please! o Watch (protectively) for few seconds!  Ataxia due to peripheral neuropathy (B12 deficiency / DM / syphilis): ↑↑ with eyes closed  Cerebellar ataxia: no ↑ with closed eyes (always on) Cerebellar signs (stroke / alcohol / tumours / para-neoplastic / …): o Nystagmus:  Can you follow my finger please (move it side to side) • Physiological: transiently then corrected • Central: horizontal or vertical • Peripheral: horizontal only. Conditions: benign positional vertigo / acute labyrinthitis / drugs o Finger to finger:  Patient hand must be extended  Move the examiner hand  Check both upper limbs o Finger to nose test: lesion in the cerebellum on the same side.  Intentional tremors  Loss of coordination o Heel to shin: lesion in the cerebellum on the same side Cortical sensations: two points discrimination Mini-mental status exam OSCE-guide-III.doc Page 128 of 255 Physical Examination Cranial Nerves Examination - Vital signs Comment on the patient general condition CN I: - Do you have problems with smells? Can you please close your eyes? - What is that? Coffee / ammonia - What is that? Ammonia / coffee CN II: The optic nerve: - Visual acuity: Do you wear glasses? reading / color (Snellen chart at 1 foot distance – 35 cm) - Visual fields: eye by eye / by confrontation (when you see my fingers wiggling) - Pupillary reflex: I am going to shine light in your eyes, please look straight to the wall, each eye: direct and consensual (afferent: CN II, efferent: CN III) - I would like to do fundoscopy examinations, looking for: disc edema, retinal hemorrhage, neovascularisation, nipping of the veins CN III, IV, VI: - By inspection; both eyes are symmetrical, no deviation, no nystagmus, no head tilting, no ptosis (CN III: opens, CN VII: closes) - I would like you to follow this pen, without moving your head please, and if you see things double or blurred at any time, please let me know  move the pen in large Hshaped manner, then conversion - Normal extra-ocular muscles movements, no nystagmus or double vision CN V: - Motor: o By inspection: no atrophy of the temporal or masseter area o Can you please clench, feel the temporalis and masseter o Can you open your mouth against my hand? - Sensory: o This is a piece of cotton, and this is how it feels, I am going to touch your face, and whenever you feel it, please tell me. Can you close your eyes please? o Touch the face in symmetrical areas; cover the ophthalmic, maxillary, and mandibular areas. Does it feel the same? o Facial sensation of the trigeminal nerve is intact and equal on both sides - Reflexes: o Corneal reflex (afferent: CN V, efferent: CN VII) OSCE-guide-III.doc Page 129 of 255 Physical Examination CN VII: (mainly motor) - Motor: o Face is symmetrical, no deviation of the angle of the mouth, normal naso-labial folds, no drooling o Can you please copy me:  Raise your eye brows, can you frown (wrinkle your forehead) please?  Can you close your eyes, and do not let me open them?  Can you blow your cheeks? Whistle?  Can you show me your teeth? o If there is a mouth angle deviation, it is deviated to opposite side of lesion o UMNL vs. LMNL:  UMNL: intact upper face muscles (the nucleus receives double sided innervation)  LMNL (Bell’s palsy)  ptosis; all face muscles are affected - Sensory: o To complete the facial nerve examination, I would like to check the sensations in the outer 2/3 of the tongue o I would like to check the ext auditory canal and the tympanic membrane to rule out herpes zoster of the facial nerve - Reflexes: o Jaw reflex o Corneal reflex (afferent: CN V, efferent: CN VII) CN VIII - Check by whispering (ABC – CBA), while rubbing fingers in front of the other ear OR by rubbing your fingers - Because the hearing is normal, I am going to skip Weber and Rinne tests o Rinne: place the tuning fork in front of ear, then on the mastoid process o Weber: place the tuning fork on the forehead CN IX, X: - Patient voice is normal, no hoarseness - Can you swallow a sip of water please? Normal swallowing - Can you open your mouth please? Soft palate is symmetrical, uvula is central o Uvula deviates to the opposite side of the lesion - To check the reflexes: I need to do the gag reflex CN XI: - Can you please shrug your shoulders? - Turn your head to the right, and to the left. I am going to resist you. I feel for the opposite side sterno-mastoid CN XII: - Can you please open your mouth? Can you stick your tongue out? o Tongue is central, no deviation. No fasciculations or atrophy of tongue. o If there is a lesion, the tongue deviates towards the lesion side - Can you please move it to the right and to the left? Can you stick it against your cheeks?  Normal movements of the tongue OSCE-guide-III.doc Page 130 of 255 Physical Examination Tremors ? Parkinson disease Introduction Vital signs General comment Inspection Shaky right hand I would like to rule out any orthostatic hypotension Pt is sitting comfortably …, with Rt hand tremors Tremors Right hand tremors, not obvious on the left hand Count from 10 to 1 please  tremors ↑ with mental activity  consistent with Parkinson disease, and rules out anxiety  Stretch your hand plz / no fine tremors  r/o hyperthyroidism  No flapping tremors  rule out liver failure  Finger to nose / no intentional tremors  r/o cerebellar dis Patient tremors consistent with Parkinson disease, resting tremors, beads rolling, and limited to Rt hand (…). No head nodding (No) limited facial expression, decreased eye blinking, drooling Face Palpation Check the wrist and elbows: Rigidity  (No) cog wheeling Positive with parkinsonism  (No) lead pipe rigidity  (No) clasp knife spasticity Positive with stroke Standing / walking  Would you please stand up! Do you need help  patient finds Postural instability difficulty in standing up  Can you walk few steps for me please: comment with + or - Stooped posture - Shuffling (festinating) gait - Decreased arm swinging - Patient turns in blocks Special Tests  Rapid alternating movements (hand supination & pronation / oppose thumb to fingers)  dysdiadochokinesia  Can you please repeat “British constitution”  monotonous  Can you write a sentence for me  micrographia  Can you draw a spiral parallel to this (draw spiral on paper) I would like to do the mini-mental status exam  Treatment of Parkinsonism:  Pharmacologic ▬ Mainstay of treatment: Sinemet (levodopa / carbidopa). Levodopa is a dopamine precursor, carbidopa decreases peripheral conversion to dopamine o Levodopa related fluctuation: delayed onset of response (affected by mealtime), end-of-dose deterioration (i.e. “wearing-off”), random oscillations of on-off symptoms o Major complication of levodopa therapy is dyskinesias ▬ Treatment of early PD: DA agonists, amantadine, MAOI ▬ Adjuncts: DA agonists, MAOI, anticholinergics (especially if prominent tremors), COMT inhibitors  Surgical: thalamotomy, pallidotomy, deep brain stimulation (thalamic, pallidal, subthalamic), embryonic dopaminergic stem cell transplantation OSCE-guide-III.doc Page 131 of 255 Then while swallowing a sip of water  thyroid movement is normal. nodules. give a sip of water.Exophthalmos – stand by the patient (stand behind the right shoulder and look from above) . and no tenderness  Lymph nodes: .Lid lag (can you follow my finger without moving your head – from above downwards)  Proximal muscle weakness: .Fine tremors .Sub-mandibular and cervical  DIRECT percussion on upper part of sternum  Checking for retro-sternal extension (no retro-sternal dullness)  BOTH lobes  For thyroid bruits  Reflexes grades: 0 absent 1 hypo 2 normal 3 hyper (brisk) 4 hyper with clonus (ankle) Whenever you ask the patient to swallow. bi-manually .doc Page 132 of 255 . HR Can you stretch your hands: .Nail changes .Physical Examination Thyroid Exam Introduction Vital signs General Thyroid Exam Inspection Palpation Percussion Auscultation 11 12 BP.Can you shrug your shoulders (bilaterally against my hand) please  Knee reflex: brisk11 reflex  Peritibial myxedema: indicates hyper-thyroidism Patient is sitting on a chair Can you swallow12 please?  no apparent thyroid enlargement  Thyroid gland: . it is difficult to swallow on an empty mouth OSCE-guide-III.From behind the patient.Hair loss (hypothyroidism)  Examine the eyes: . I do not feel any masses.Palms for sweating . Physical Examination Dermatomes OSCE-guide-III.doc Page 133 of 255 . no atrophy / SEADS Palpation I am going to feel your shoulder. any pain . triceps. hypothenar  are symmetrical / no deformity / no atrophy Motor Power Deltoid C5 Biceps C5/6 Triceps C7/8 Sensory C4: deltoid C5: biceps – lateral aspect Test light C6: thumb C7: middle finger C8: touch little finger T1: elbow – medial aspect Reflexes  Biceps and brachio-radialis C5/6  Triceps C7/8 Radial pulse Pulse Post encounter: what is the level of the lesion? C6 nerve root lesion (C5-C6): weak biceps. deltoid. weak triceps reflex Diagnosis: Osteoarthritis of the cervical spine at … level X-ray findings: osteophytes of the cervical vertebrae / narrowing of disc space / subchondral sclerosis (increased bone formation around the joint). biceps.doc Page 134 of 255 . subchondral cyst formation Management: ▬ Soft neck collar ▬ NSAIDs / acetaminophen ▬ Physiotherapy ▬ If worsening: neurosurgery consult / CT myelography / nerve conduction studies OSCE-guide-III.Neck pain is not associated with muscle contractures Spurling test: stretching the nerve  reproduces the pain Upper limb examination and neurological screening Neurological screen Part of my exam is to check your upper extremities.Temperature is normal . trapezius and sternomastoid Thyroid: can you swallow for me please? Check the LNs: no enlarged LNs Can you touch your chest by your chin? Can you look to ceiling? Can you turn your head to the right? And to the left? Can you tilt your head? To the right and to the left? .Normal flexion. extension.Tenderness: pressing on the spinal processes and para-vertebral muscles. rotation and lateral flexion Test again against resistance . weak biceps reflex C7 nerve root lesion (C6-C7): weak triceps. normal bulk. can you lower your gown please? From the back: SEADS (Swelling / Erythema / Atrophy / Deformity / Scars) From the side: normal cervical and thoracic curvatures Can you cough please. forearm. thenar. can you roll up your sleeves please! Inspection Upper extremities are symmetrical.No neck pain with Valsalva manoeuvre I would like to feel you back please .Physical Examination Neck Examination Patient complaining of pain in the neck Vitals General Inspection  Palpation         ROM     Powers  Special tests   Can I get the vital signs please Comment on patient general condition: patient is sitting comfortably … I would like to take a look at your neck. numbness. tingling  Swelling.Touch base of your little finger (thumb opposition) .Physical Examination Carpal Tunnel Syndrome Hand pain for 6 months History Analysis CC  Os Cf D / PQRST / ↑↓  Radiation: how about your elbow? Your fingers?  Triggers: is it related to time? To work? What do you do? AS  Weakness. styloid process. carpal bones.Move it all the way to opposite direction . metacarpal bones.Point to the ceiling (with hand supine. to the left  Can you make a fist / fan your fingers  there is no obvious damage to the nerves / muscles / and tendons of the hand  Thumb movements: . styloid process. digits ROM  Flexion / Extension /+/ move your hand to the right.doc Page 135 of 255 . flat) .Touch the tips of your fingers Power  Like ROM but against resistance  Thumb 90°. base of the thumb. joint line. DIP flexed: do not let me straighten it  Biceps ROM / against resistance / biceps reflex (C6) Sensory  Check with cotton tip. redness. distal ulnar bone. warm  How about the other hand? Impact  How did this affect your life? Your work? Red flags  Constitutional symptoms Differential  Hx of neck pain / injury / trauma diagnosis  Hx of diabetes / symptoms  Hx of thyroid disease / symptoms of hypothyroidism  Hx of autoimmune disease / Rh arthritis / symptoms  Headache / visual disturbances / shoes are getting tight  Pregnancy / LMP PMH / FH Physical examination Vital signs Inspection  SEADS (thenar / hypothenar ms)  No nail changes.  Tinel's sign / tap on the carpal tunnel Investigations: EMG / nerve conduction studies Treatment: ▬ Modify nature of work ▬ NSAIDs ▬ Wrist splint ▬ Local corticosteroids injection ▬ Surgical decompression OSCE-guide-III.  For the ring finger: check both sides: ulnar / radial Special tests  Phalen's test. no nodules / no deformity Palpation  Temperature: is normal  Tenderness: palpate distal radial bone. Consult plastic surgery for micro-vascular repair. digits  Capillary refill  Feel the pulse  radial ± ulnar  Allen’s test ROM  Do NOT ask patient to apply force against resistance as this may rupture a partially severed tendon. may suture the skin and arrange for pt to be seen by plastic surgeon next day. . Test active ROM only Sensory  Check with cotton tip or pin-prick. for light touch  Check two point discrimination on either side of each digit  For the distribution of the three nerves: ulnar / radial / median Neuro-motor examination of the hand Median Sensory Radial aspect of index finger pad Motor Flex DIP if index finger (FDP) extrinsic Motor Thumb to ceiling with palm up intrinsic (abductor pollicis brevis) Ulnar Radial Ulnar aspect of little finger pad Dorsal web space of thumb Flex DIP of little finger (FDP). extensor carpi radialis (extensor pollicis longus) Abduct index finger (first dorsal interosseous) Structures lacerated Diminished ulnar territory sensation Allen test shows (no) refill from the ulnar circulation FDS weakness in little finger and ring finger Ulnar nerve Ulnar artery Flexor retinaculum. flexor digitorum superficialis: hold all fingers – except one you are testing – in extension and ask pt to flex the remaining finger . distal ulnar bone. is hand pink and warm?  Tenderness: palpate distal radial bone. flexor digitorum profundus: hold the MCP / PIP joints in extension.PIPs. Extend wrist and thumb.doc Page 136 of 255 . carpal bones. styloid process. base of the thumb.Physical Examination Hand Laceration / Wrist Laceration Physical examination Introduction Gloves / vital signs Inspection  If big wrist laceration (BIG band aid): DO NOT remove the band aid. ulnar two divisions of FDS Management: clean and explore wound under local anesthesia and sterile conditions. should be removed under anesthesia  If small laceration. styloid process. If at night.MCP joints: intrinsic (lumbricals) Power  Do NOT ask patient to apply force against resistance as this may rupture a partially severed tendon. metacarpal bones. OSCE-guide-III. joint line. remove the band aid  describe the laceration: linear / size / position / elevated margins / active bleeding / oozing  Other ulcers / other hand  SEADS (Swelling / Erythema / Atrophy / Deformity / Scars)  No nodules / no deformity Palpation  Temperature: is normal. and ask the pt to flex DIPs.DIPs. Test active ROM only  Median nerve: thumb abduction (thumb to ceiling)  Ulnar nerve: finger spread (fanning)  Radial nerve: wrist extension  Tendon examination of the hand (flexor tendons): . increases when extension (or leaning backwards). walking down the hill (arching the back)  spinal canal stenosis Pain wakes patient in morning.doc parathesia age > 50 years old IV drug user neuro-motor deficits Page 137 of 255 .Physical Examination Back Pain Acute   Acute on top of chronic Chronic Herniated disc Muscle spasm Pain at end of day shoots to legs. stiff  Ankylosing spondylitis  limitation in all directions Neurological exam Mechanical exam Red flags for back pain: BACK PAIN  B bladder or bowel dysfunction  A anaesthesia (saddle)  C constitutional symptoms  malignancy  K chronic disease     P A I N OSCE-guide-III. improves at end of day. increases when leaning forward:  Disc herniation  Degenerative disease (OA) Pain related to position. your back or your legs? ↑↓  How about lying down? Stretch your back? Coughing? Moving? Leaning forward or backward?  Any medication? Did it help?  How does it affect you?  Mets to lungs / liver / brain  Constitutional symptoms  Overweight In addition to your pain. whenever you want to lie down or rest. feel free to do so Os Cf D  What were you doing? What do you do for work?  Did you hear any crack sound?  Were you able to stand up and continue? Were you able to move? Did anyone help you?  Is this the first time?  How about at rest? During night?  Recently did you have any discomfort? Milder back pain? How did it affect you? PQRST  Does it shoot to your thighs? Toes?  Which is bothering you more. tingling  Difficulty with balance. sit. did you notice any other symptoms:  Weakness. falls  Any difficulty passing urine? How about bowel Cauda equine movement? Did you find that you soiled yourself?  Any numbness in the buttocks area?  Do you have morning erection? Any sexual dysfunction? Cancer prostate / bladder Cancer prostate Page 138 of 255 .doc How do you feel? Do you prefer to remain standing. numbness.Physical Examination Acute Back Pain Acute back pain: pain for 3 days Acute on top of chronic: pain for 3 hours – old patient Introduction Analysis Impact Red flags DD PMH FH SH OSCE-guide-III. or lie down? That is ok. sit.doc Page 139 of 255 . feel free to do so Os Cf D  What were you doing? What do you do for work? PQRST Impact Red flags DD PMH FH Quality: stiffness Timing: is it worse in morning? Improves with time? Or is it worse at the end of the day?  Does it shoot to your thighs? Toes?  Which is bothering you more. redness / mouth ulcers spondylitis symptoms  Skin changes / nail changes / hx of psoriasis  Repeated attacks of abd pain / diarrhea  Urethral discharge  How does it affect you?  Mets to lungs / liver / brain  Constitutional symptoms  Overweight In addition to your pain. your back or your legs? ↑↓  How about lying down? Stretch your back? Coughing? Moving? Leaning forward or backward?  Any medication? Did it help? AS  Pain other joints (knees / hips / hands) / distribution? Osteoarthritis Sero Pain other joints (knees / hips / hands) / distribution? Ankylosing negative  Eye pain. numbness. did you notice any other symptoms:  Weakness. whenever you want to lie down or rest. tingling  Difficulty with balance. or lie down? That is ok.Physical Examination Chronic Back Pain Pain for 3 months Introduction Analysis Do you prefer to remain standing. falls  Any difficulty passing urine? How about bowel movement? Did you Cauda equine find that you soiled yourself?  Any numbness in the buttocks area?  Do you have morning erection? Any sexual dysfunction?        Trauma Injury to back Osteoarthritis Ankylosing spondylitis Other rheumatic disease SH Osteoarthritis: older patient / worse at evening Ankylosing spondylitis: ▬ Morning stiffness improves by time ▬ LSS x-ray: sacroiliitis OR fusion of SI joints ▬ ESR: ↑ ▬ HLA-B27 tissue antigen: positive Associated symptoms: inflammatory arthritis / Uveitis / psoriasis / IBD / pericarditis / aortic regurgitation Management: ▬ No cure ▬ Regular therapeutic exercises to prevent deformity (swimming / back extension exercises) ▬ NSAIDs: Indomethacin (50 mg PO bid) or Naproxen (250 mg PO bid) ▬ In severe cases: total joint replacement OSCE-guide-III. doc Page 140 of 255 . para-vertebral muscles.Physical Examination Back Joint Examination Neurological screen Physical examination Introduction Can you stand up please? Vital signs Inspection  Gait / balance / stance  Ask the patient to stand up from sitting position  Posture: normal cervical. reduced anal tone) N. lumbo-sacral curvatures  Adam’s forward bend test (if scoliosis: the scapula will be higher) . Motor  Hip flexion (L1/L2/L3) / extension (S1/S2)  Knee flexion (L5/S1/S2) / extension (L2/L3/L4)  Ankle dorsiflexion (L4/L5) / plantar flexion (S1/S2)  Can you walk on your heals? Normal L4/L5 muscles  Can you walk on your toes? Normal S1/S2 muscles Sensory S1: little toe L5: first web L4: medial malleolus L3: knee – med L2: thigh – ant L1: groin T10: umbilicus Reflexes Knee (L2/L3/L4 – mainly L4) / Ankle (S1/S2 mainly S1) / I would like to do the Babinski reflex (positive in UMNL) Pulse Dorsalis pedis Other clinical examinations: DRE. to rule-out cauda equina (sphincter weaknesses. thoracic. Special tests  Occiput-to-wall distance (tragus & nose same level): normally zero  Straight leg raise (irritation of the roots of sciatic n: L4/L5/S1/S2): elevate the lower extremity straight. sacro-iliac joints (medial to dimples of Venus) ROM  Can you touch your toes with your fingers? Without bending knees  Can you arch your back? Without bending knees (stand supported by the bed foot: will not fall. dimples of Venus correspond to PSIS OSCE-guide-III.No scoliosis or kyphosis  SEADS Palpation  Temperature  Tenderness: spinal processes. try to dorsiflex foot  Lasègue sign  Cross straight leg raise test: elevate the other LL  trigger pain  Faber’s test (figure 4 test): to check sacro-iliac joint pathology  Femoral nerve stretch (done for patients c/o pain radiating to the anterior aspect of the thigh): patient prone. knee flexed.B. normally the tips of finger travel > 10 cm)  Cross your arms? Turn to the Rt and Lt (pt sitting on bed)  Modified Schober's test: (midline. between the dimples of Venus) + 5 cm below + 10 cm above  bend forward  N> 6 cm diff. less possibility of knee bending)  Slide your arms on both sides (Rt and Lt)? (stand against wall. when it is painful  where it does hurt?  straight leg test positive  Decrease the angle. and ask him to wrap it again (to make sure he knows how to). OSCE-guide-III. no fractures. history and physical examination are normal. q6h.doc Page 141 of 255 .g. and no lacerations. med lig rupture (eversion)  Squeeze test (squeezing calf  ankle pain). PO. for foot series o If there is any pain in the mid-foot zone and any one of the following:  An inability to bear weight both immediately and in the emergency department for four steps  Bone tenderness at the base of the fifth metatarsal  Bone tenderness at the navicular bone Management:  Complete tear should be evaluated by orthopedics  stat orthopedics consult  RICE: rest (and crutches) / ice for 20 min QID x 3 days / compression (by tensor bandage) / elevation  Pain medication: NSAIDs.Physical Examination Ankle Twist Young man comes with ankle twist. indicates ant talo-fibular lig rupture  Talar tilt: lat calcaneo-fibular lig rupture (inversion). e. Remember: from distal to proximal and 1/3 width overlap.  Show him how to wrap it. remove the wrap. In the next 10 minutes counsel him about the treatment History Mechanism of trauma / injury Noise heard at the time of trauma  Where you able to walk after the injury? Symmetry / SEADS  Temperature  Tenderness: med malleolus / lat malleolus / ant joint line (between malleoli) / lat ligaments (slightly ant and inferior to the lat malleolus) / med ligaments (inferior and slightly ant to the med malleolus) / tendon Achilles / navicular head (medial bony prominence) / fifth meta-tarsal head (lat) / around the meta-tarsal heads / origin of the plantar fascia (inf med aspect of calcaneus )  Crepitus Patient is sitting on the side of the bed  Dorsi-flexion (point toes up) and plantar flexion (point toes down)  Inversion (point the bottoms of your feet towards each other) and eversion (the opposite direction) ROM against resistance / after stabilizing the ankle with your other hand  Talar drawer sign: 1 cm is significant. Ibuprofen 400 mg. if negative  NO ankle fracture   Inspection Palpation ROM Power Special tests Investigations: x-ray  Ottawa ankle rules. for ankle series: o Pain in the malleolar zone and any one of the following:  An inability to bear weight both immediately and in the emergency department for four steps  Bone tenderness along the tip of the medial or lateral malleolus  Ottawa foot rules.  Show him how to use the crutches. Physical Examination Shoulder Joint History .One joint above and one joint below examination (cervical spine / elbow) .Trauma to shoulder / neck? X-ray done? What is your occupation? .Forward flexion (180°) /+/ Backward extension (60°) .Both shoulders symmetrical / clavicle level / scapula level / deltoid .Brief neurological examination of the upper limb . try to let your palm face upwards. I would like to do: .Hands behind your neck (abduction / ext rotation) . do not let me push your arm down 9 Stability testing: For joint stability + Push ant / post + Pull down  sulcus sign 10 Apprehension test (ant and post): for dislocation To complete my exam. discomfort. but against resistance Special tests 1 Painful arc (between 60° and 120°) All these tests are done to 2 Drop arm test  complete tear of supratest for subacromial spinatous tendon impingement of supra3 Neer’s test spinatous 4 Hawkin’s test 5 Jobe’s test (empty can test) 6 Lift-off test: try to push my hand away from Sub-scapularis your back Yergason’s test.Check the pulses of the upper limb (radial / ulnar / brachial) .Adduction  and comment on drop arm test .Passive ROM: If patient is unable to complete the whole range of movements actively. semi-flexed.Abduction  and comment on painful arc test .Tenderness: sternal notch / sterno-clavicular joint / clavicle / acromioclavicular joint / deltoid / long head of the biceps / insertion of the rotator cuff muscles / spine of the scapula / medial border of scapula / spinal processes of the cervical spine .Hands behind back (adduction / int rotation) between shoulder blades.Neurological deficits? How does it affect your life? Vital signs General Patient condition (restlessness. . palm face up test: shake For bicepital tendinitis hands.Like the ROM. and press on your shoulder 8 Speed’s test: supine.SEADS (Swelling / Erythema / Atrophy / Deformity / Scars) Palpation .Crepitus ROM . I will resist you. willingness to move) Inspection .The other shoulder examination 7 OSCE-guide-III.doc Page 142 of 255 .Active ROM: can you copy me please: . complete the ROM passively and comment (in inflammation: passive ROM is > active ROM) Power .External rotation /+/ Internal rotation .Temperature: compare .Another faster way to check: . touch the tip of the contra-lateral scapula. ice packs and NSAIDs may be used for pain relief.Moderate: therapeutic injections of corticosteroid and local anesthetic may be used for persistent impingement syndrome . there are points of tenderness over the lateral epicondyle (origin of the extensor carpi radialis brevis muscle).Normal shoulder exam .Mild: RICE / NSAIDs / PT.  OSCE-guide-III. naproxen or aspirin /+/ Heat or ice o A counter-force brace or "elbow strap" to reduce strain at the elbow epicondyle. Rest (cessation of painful activity). weakness and a loss of movement at the affected shoulder Treatment: . Physiotherapy (PT) focused at maintaining range of movement and avoiding shoulder stiffness. NSAIDs. NSAIDs.The most common symptoms in impingement syndrome are pain. Golfer's elbow (medial epicondylitis) The common tendinous sheath is inserted into the medial epicondyle of the humerus  Treatment: o NSAIDs: ibuprofen. surgery) similar to partial tear / +ve impingement test Elbow ROM: flexion / extension /+/ pronation / supination Tennis elbow (lateral epicondylitis)  With the elbow fully extended.Severe: surgery Investigations: .doc Page 143 of 255 .Physical Examination Rotator cuff muscles: Supra-spinatous Infra-spinatous Teres minor Sub-scapularis Abduction External rotation Internal rotation Lift-off test Impingement syndrome: .Frozen shoulder - - Bicepital tendinitis Repeated ant dislocation Rotator cuff tear o Complete tear o Partial tear Rotator cuff tendinitis Sub-deltoid bursitis stiff. such as fracture or arthritis.  Cozen's test: pain with passive wrist flexion and resistive wrist extension.U/S Possibilities: . steroids. steroids) +ve palm face up test / speed test positive apprehension test drop arm test (ttt: surgery) pain with initiation of movement / +ve empty can test (ttt: physiotherapy. .[  X-rays are used to confirm and distinguish possibilities of existing causes of pain that are not related to Tennis Elbow. with limited active and passive ROM (ttt: physiotherapy. to limit pain provocation and to protect against further damage. IV for 2 wks then oral for 2 wks). stabilize the contra-lateral hip with your left hand: Abduction (45°) /+/ Adduction (30°) . while ant lat hip pain may suggest hip joint pathology . 20 mg/kg IV q8h.Physical Examination Hip Joint + Middle-age male with septic arthritis + Elderly female with osteoarthritis Vital signs General - Patient condition (restlessness. drop foot.Thomas test  put your hand under pt LSS. I would like to do: . discomfort. and try to max flex the contra-lateral knee . greater trochanter of the femur. I am looking for the surroundings SEADS (Swelling / Erythema / Atrophy / Deformity / Scars) Scoliosis / kyphosis / pelvic tilt (level of both iliac crests) Gait: no wide stance. below inguinal lig) ROM Active ROM.Resisted isometric testing (patient lying supine) Special tests .Forward flexion (120°) . The examiner applies counter-pressure at the opposite hemi-pelvis. adjust pending C&S ▬ For small joints: needle aspiration.The other hip examination Septic arthritis: Physical exam: fever / very painful joint / +ve trendlenberg test / restricted movements on all directions DD: Septic arthritis / Osteoarthritis / Osteomyelitis One diagnosis: septic arthritis / One diagnostic test: arthrocentesis (joint aspirate) Management: ▬ IV antibiotics. or vancomycin [if suspecting MRSA. can you lift your thigh (20°) Power . each one followed immediately by passively stressing (increasing) the ROM – while patient is lying . standing on one leg (while the patient is putting his arms on the examiner shoulders). or antalgic gait Balance: Trendelenberg sign. serial if necessary until sterile ▬ For major joints such as knee.One joint above and one joint below examination (LSS / knee) . I would like to check symphysis pubis and inguinal ligament. . for 8 wks]. oxacillin [2 g IV q4h for 4 weeks]. Post hip pain indicates sacro-iliac joint pathology.Create space. iliac crest.Brief neurological examination of the lower limb . empiric therapy. (based on age and risk factors. and applies gentle downward force on the knee. or shoulder: urgent decompression and surgical drainage OSCE-guide-III.Tenderness: ASIS. sacro-iliac joint. the pelvis drops Palpation . combined with ceftriaxone for gram –ve. PSIS.Apparent leg test  from umbilicus to medial malleolus To complete my exam. willingness to move) May I ask for full exposure please? Inspection Hip joint is deeply seated joint.Crepitus: over femoral head (lat to femoral art. shuffling.Figure 4 test (Patrick or Faber’s test)  the leg of the examined side flexed and externally rotated with the ankle resting on the patella of the contra-lateral leg. if suspecting Gonococcal: ceftriaxone.Check the pulses of the lower limb (dorsalis pedis / posterior tibial / popliteal) .doc Page 144 of 255 .Internal rotation (30°) /+/ External rotation (45°) .True leg length  from ASIS (anterior superior iliac spine) to medial malleolus – on both sides .Backward extension (while lying prone): stabilize the lower back by your left hand.Temperature: compare . hip. meniscus Knee lock  torn meniscus Instability  cruciate  DD Tenderness Investigations Treatment OSCE-guide-III.Gait and stance: normal.Anterior drawer test  for anterior cruciate ligament tear .Fluid ballottement test: for moderate amount of effusion .Patellar compression test: tight your thigh please  rough or painful movement: patello-femoral syndrome or osteoarthritis .SEADS (Swelling / Erythema / Atrophy / Deformity / Scars) .Knee giving way . externally rotated foot  extend while applying varus force (from inside outwards) For lateral: maximally flexed knee.Check for the medial / lateral collateral ligaments (stability of knee). while the knee is flexed 90° .Popliteal fossa /+/ Cuff muscles [slightly flexed knee] Knee effusion: .Can not descend stairs Osgood–Schlatter disease Chondromalacia patellae  Pain on tibial tuberosity (↑ by kneeling)  Pain on lateral movement of patella X-ray (AP / LAT / skyline)  Benign self-limited condition  Non-impact activities  Continue activity as tolerated  NSAIDs  NSAIDs  Physiotherapy  Physiotherapy  Surgery for refractory cases Page 145 of 255 . from below and med to upward and lat.Patellar tap: for large amount of effusion Patient lying down: .Flexed knee: tibial plateau / bilateral joint lines /+/ Collateral ligaments /+/ Femoral condyles /+/ patellar crepitus .Bilateral joint exposure (quadriceps) .No genu varum (bow legs) and no genu valgum (knock-knee) deformities .Internal and external rotations: while knee is flexed 90°. point your toes in & out please . General . Then immediately sweep hand down the lateral aspect pushing the fluid back .Inability to continue activity MCL .Patellar movement: medial and lateral .Flexion and extension.doc ACL .Fluid wave or bulge sign (or milking test): for small amount of effusion.Extended knee: tenderness over patella /+/ Lateral movement of patella /+/ quadriceps muscle / quadriceps tendon / patellar ligament / Tibial tuberosity / popliteal fossa and popliteal artery . no antalgic (painful) gait .Temperature: compare . while flexed at 30°  no laxity nor pain . I would like to do … - Painful clicking  cruciate.Physical Examination Knee Joint Inspection Palpation ROM Power Special tests Vital signs.Lachman test: hip / knee semi flexed (30°)  ACL tear .Posterior drawer test  for posterior cruciate ligament tear . internally rotated foot  extend while applying valgus force (from outside inwards) To complete my exam.Flexion (130°) and extension (180°) .McMurray’s test (for medial and lateral meniscus tears)  feel for crepitus / patient feels pain - - For medial: maximally flexed knees. doc Page 146 of 255 .Physical Examination Obstetrics and Gynecology OSCE-guide-III. HRT .doc Page 147 of 255 .Vaginal discharge .PAP smear.C-section (wants to have c-section or wants to have vag delivery after c-section) .Abortion . 16 years old wants to arrange for a PAP smear .Infertility Counselling: .30 yrs old pregnant (36 wks).Amenorrhea .PAP smear: 38 year old had abnormal PAP smear OSCE-guide-III. counsel her .OCPs .Vaginal bleeding .39 years old found she is pregnant.22 years old pregnant  anti-natal counselling . HTN/+++ ptn in urine  counsel for pre-eclampsia .Physical Examination History taking – OB-GYN Introduction CC Analysis of CC HPI Os Cf D COCA ± Blood ↑ ↓ Associated symptoms DD Menstrual M Gynecological G Obstetric O Sexual S PMH FH SH OB/GYN cases History taking: . doc Page 148 of 255 . polyps)? Do you have history of pelvic surgery or instrumentation (e. provided that: . And before that. and by that I mean using condoms every time? For how long you have been together? (> 6 months  stable). were you sexually active? When did you start sexual activity? How many partners have you had for the last 12 months? For the last month? What is your sexual preference? Men/ women/ both? What type of sexual activity? Have you screened or diagnosed before with STIs? HIV? Vaginal discharge? How about your partner? Any fever? Discharge? Burning sensation? Do you feel safe in this relation? What if the male partner does not like condoms? Is it ok to consider it safe sex? Yes. D&C)? Do you use contraception? What method? Since when? When was the last time? Screening: - Have you ever had Pap smear before? When was the last time? Any reason (if long time)? What was the result? (>40 yrs) have you had mammogram done before? When? (Is it painful doctor? Could be.Use alternative reliable contraception (e. we need to apply pressure on the breast to get better image) (>65 yrs) have you had your bone mineral density (BMD) done? Any reason? Obstetrical  GTPAL: - Have you ever been pregnant before? Any abortions (termination)? Or miscarriages (spontaneous abortion)? Number of babies you delivered? Any twins? Any children with congenital abnormalities? For each delivery: was it full term or pre-term? Vaginal or CS? Any complications like high blood pressure / high blood sugar? Family history of: repeated abortions / CS / congenital anomalies / twins Sexual history: - - With whom do you live? If (alone / with family): are you in any relationship? Are you sexually active? Have you ever been sexually active? If with partner: how do you describe the relationship? Is it stable? Are you sexually active? Do you practice safe sex.Strict monogamy relation (no extra-marital affairs) .Physical Examination MGOS history questions: Menstrual: - When was your LMP? First day? Was your LMP similar to the previous ones? Are they regular or not? How often do you have periods? How long does it last? How many days? How about the amount? Is it large / small? How many pads/day? Any blood clots? Are your periods painful? [not painful  anovulatory (PCOS/infertility)] Any spotting / bleeding between periods? When was your first period? Was it regular? For how long it was not regular? Normal to be irregular for up to 18-24 months.Scan the partner for STIs first . Gynecological: - Do you have history or were diagnosed with any gynecological disease (e. OCPs) OSCE-guide-III.g.g.g. Physical Examination History of pregnant lady – third trimester Are you doing regular ante-natal follow-up visits? NO Yes Social issue Deal with the social issue Last visit history / pre-eclampsia    Make sure the mother is stable    Make sure the baby is stable U/S      When was your last f/u visit? What was your BP? Was there any headache? Was there leg swelling? Weight gain? Any abdominal pain? Cramps? Vaginal bleeding? Discharge? Any gush of water? Is your baby kicking like before? > 6 in 2 hrs Have you done your U/S? How many times? When was the last time? Number of babies? Location of the placenta? Amount of fluids? N.doc Page 149 of 255 .B. to make sure the mother and baby are stable: ABCDE  Activity of the baby  Bleeding  Contractions / pain  Dripping / Discharge  EDD (expected date of delivery) OSCE-guide-III. vaccination against HPV 13 Blisters: HSV (Herpes Simplex Virus) / warts: HPV (Human Papilloma Virus) / ulcers: syphilis OSCE-guide-III.Urine changes? Dysuria. speculum exam / PAP smear / swabs for C&S including those for Chlamydia & Gonorrhea / saline slide microscopy / KOH / Whiff test  DD: Gonorrhea.Related to sexual intercourse (bact vaginosis: ↑ discharge post-coitus) .Sore throat? Mouth ulcers? Red eyes? . 1 vag supp od qhs x 3 d o Bacterial vaginosis: Metronidazole 500 mg PO bid x 7 d If pregnant: Amoxicillin 500 mg PO tid x 7 d o Trichomonas: Metronidazole 500 mg PO bid x 7 d  Follow up with in 4 weeks  Her partner(s) to be notified and to come for treatment.Dissemination to liver (pain Rt upper abd) DD .PAP smear! .Physical Examination Vaginal Discharge Teenager / 5 minutes case CC How can I help you?! Analysis of CC Os Cf D COCA ± Blood / color / fishy odour? . Bacterial vaginosis (thin gray. Trichomonas (frothy yellowish / greenish discharge.  Treatment: o Gonorrhea: Ceftriaxone 250 mg IM single dose o Chlamydia: Azithromycin 1g orally single dose o Candidiasis: Miconazole 200 mg vag supp.Itching? Redness? ? Candida DD .HEAD SS / SAD Conclusion: STI because of risky sexual behaviour  Physical examination including pelvic.↑ ↓ . motile organism).? PID  Adnexal tenderness / fever .LMP / regular / how often / similar to previous ones? M Same system HPI .Joint swelling/pain? Skin rash? Reiter’s G .Allergies .How do you support yourself? .Bowel movements changes? GIT symptoms .Inguinal swellings? . frequency? Nearby systems .Abdominal pain  OCD / PQRST / ↑↓ .History of STI / PID? O Complete sexual history for both partners S PMH .Any medications? Recent use of antibiotics . discharge)  Advice regarding safe sex (condoms.IUD .Any pain? With intercourse? AS .doc Page 150 of 255 . multiple partners. ask about sexual health (fever.Any blisters / warts / ulcers13? .DM FH / SH . clue cells). Chlamydia.Related to periods .Constitutional symptoms . STIs)  Chlamydia and Gonorrhea are reportable diseases  HIV testing and other STIs screening if high risk sexual behaviour  Advise regarding PAP smear regularly. Candidiasis (whitish). ↑ ↓ .Contraception history.Constitutional symptoms DD . OCPs / IUD / HRT / mammogram .Any previous pregnancies? Abortions? How many? Route O of delivery? . intermittent / lose track of periods / no pain with periods. SOB.Urine changes? Urinary symptoms? Nearby .Any nausea / vomiting / breast engorgement? Frequency? DD . endometriosis.For the last few weeks.LMP / regular / how often / how much / similar to previous ones? .Related to sexual intercourse (if yes: cervical*) . dizziness.Bleeding disorders/ tendencies? Any bleeding elsewhere (nose.Pelvic fullness / heaviness? . A 48 years patient with vag bleeding and all symptoms will be negative  dysfunctional uterine bleeding (DUB).Abdominal pain: OCD/PQRST (? PID / ectopic) .Thyroid problems? Symptoms? . clots  if large amounts  impact  anemia / dehydration symptoms (pallor.First menstrual period? Regularity? HPI .Breast cancer.Itching? Redness? . how do you distinguish between your regular M periods and the bleeding? Related to periods . with stool.Number of partners / safe sex / when did you start activity? S . (menopausal symptoms / dyspareunia / itchy vulva)  Rule out cancer (endometrial biopsy)  estrogen vaginal cream DD: Fibroid / Cervical polyp / hyper/hypo-thyroidism / Trauma / Coagulopathy / PCOS Investigations: pregnancy test β-HCG / progesterone challenge test / hysteroscope / PAP / U/S / endometrial biopsy / TSH 14 Cervical cancer for prostitutes (risky behaviour) and endometrial cancer for nuns (no pregnancies) OSCE-guide-III. heart racing) . easy bruising)? .Blood thinners? Aspirin? .Any chance you are pregnant? How do you know for sure? Same AS system .Any pain? With intercourse? . HTN FH .How do you support yourself? .Hx pelvic surgeries? Instrumentations? . A 52 years pt / constipation / OCP/HRT / no pregnancies14  endometrial cancer / constitutional A 62 years pt with intermittent bleeding / small amount with secretions  atrophic vaginitis.doc Page 151 of 255 .Grape like tissue (hydatiform mole) . mammogram (if > 40 years). gums.PAP smear! Was it normal? .Hx of STIs PMH .Gynecological cancer / Breast cancer SH .Bowel movements changes? GIT symptoms systems . fainting. gyn cancer? G . fibroids.Physical Examination Vaginal Bleeding – Non-Pregnant / Not-Known Pregnant Analysis of CC Timing: OS Cf D – When did it start to be continuous? COCA / fresh blood vs.Polyps.Any blisters / warts / ulcers? .SAD / HEAD SSS (if teenager) Vaginal bleeding cases: - - Middle age / risky behaviour / old abnormal Pap smear  cervical cancer. Pelvic fullness / heaviness? . come back if more blood or more pain Painless Painful (contractions) . arrange for C/S .Breast cancer. very anxious.Headache.Any pain / discomfort? (OCD / PQRST)? Contractions .Medication / allergies / hospitalization / surgeries / blood transfusion.LMP / regular / how often / how much / similar to previous ones? .Any previous pregnancies? Abortions? How many? How many weeks? Route of delivery? .Trauma.doc Page 152 of 255 .Gynecological cancer / abortions . kidney disease.Otherwise: bed rest / steroids / fetal monitoring / Rhogam / platelets Incomplete abortion: 3 findings on vag exam would confirm the diagnosis: (1) Cervix dilated.Polyps. gyn cancer? .Water gush / grape like tissue (hydatiform mole) .Admit to the hospital .Hx pelvic surgeries? Instrumentations? . Placenta previa Placenta abruption resume normal activity. endometriosis. fainting. .PAP smear! Was it normal? .If she is improving / the fetus is not distressed: she will go home.Bowel movements changes? GIT symptoms systems .50/50% chance to keep or loose the baby . if fetus is still in . blood gp & Rh . (2) Ruptured membranes.Investigations: continuous vitals monitoring / CBC / INR / PTT / . Vaginal exam only after U/S excludes placenta previa .HTN.Bleeding disorders/ tendencies? Any bleeding elsewhere? . SOB.SAD / HEAD SSS (if teenager) Threatened abortion Separation of part of the placenta Will do physical and obstetric examination.Arrange for continuous fetal heart monitoring.Constitutional symptoms DD .How did you know that you are pregnant? Was it planned? system . U/S fibrinogen / Rh status / blood grouping and cross matching / US / and biophysical profile fetal monitoring . IV fluid.Fetal movement .Abdominal pain: OCD/PQRST (? PID / ectopic) . dizziness. hand or feet swelling . (3) Product of conception passed OSCE-guide-III.Management: O2.GTPAL .Physical Examination Vaginal Bleeding – Pregnant / Ante-Partum Hemorrhage    Patient usually sleeping in a left lateral position with oxygen. DM. mammogram .Management: distress. fibroids.What is the result of U/S? .How many weeks? date based on LMP Same . LLP. crying Reassure her & ask her to bear with you for a while to get to the bottom of her problem Ask about her feelings & empathize (impact on her and her partner) Analysis of CC HPI M AS DD G O S PMH FH SH Timing: OS Cf D first time or happened before COCA / fresh blood vs. heart racing) ↑ ↓ . clots  if large amounts  impact  anemia / dehydration symptoms (pallor.Did you have regular antenatal care? F/U visits? U/S? .How do you support yourself? .Urine changes? Urinary symptoms? Nearby .Hx of STIs . sexual intercourse . adolescent): rule out von Willebrand's disease TSH. measure endometrial thickness (postmenopausal) Sonohysterogram (SHG): very sensitive for intrauterine pathology (polyps. DUB is a diagnosis of exclusion. fibroids. ─ Causes: o ↓ progesterone: luteal phase defect (estrogen-dependent DUB) o PCOS o Endocrinal (↓ TSH / ↑ prolactin) o Stress. Anovulatory AUB often used synonymously with DUB. submucous fibroids} Hysterosalpingography (HSG) Pap test Endometrial biopsy: women > 40 years are at higher risk of endometrial cancer  Must do endometrial biopsy in all women presenting with postmenopausal bleeding to exclude endometrial cancer D&C: not for treatment. serum ferritin Coagulation profile (esp.doc Page 153 of 255 . exercise o Liver and kidney disease ─  Investigations for AUB: • o o • • o o o • • • o o o Beta-hCG CBC. free T4 Prolactin if amenorrhea FSH. weight loss. diagnosis only (usually with hysteroscopy) OSCE-guide-III. LH Day 21 (luteal phase) progesterone to confirm ovulation Serum androgens (especially free testosterone) Pelvic U/S: detect polyps.Physical Examination Abnormal Uterine Bleeding (AUB)  Dysfunctional Uterine Bleeding (DUB): Abnormal bleeding not attributable to organic (anatomic / systemic) disease. consider pre-treatment with danazol or GnRH agonists o ─   o If finished childbearing Repeat procedure may be required if symptom recurrence Hysterectomy: definitive treatment Post-coital bleeding / middle age: ─ STIs ─ Cervical (Cervicitis / Cervical polyp / Cervical cancer / Ectropion.g. consider admission Medical treatment: ─ (a) estrogen (Premarin) 25 mg IV q4h x 24h with Gravol 50 mg IV/PO q4h or ─ (b) Ovral15 1 tab PO q4h X 24h with Gravol 50 mg IV /PO q4h Taper Ovral: 1 tab tid X 2d  bid X 2d  OD After (a) or (b). if OCPs) ─ Bleeding tendencies ─ Trauma Management: ─ Vaginal and cervical swabs. consider dysfunctional uterine bleeding (DUB) ─ Medical: o Mild DUB ─       o NSAIDs Anti-fibrinolytic (e. Cyklokapron) at time of menses Combined OCP Progestins (Provera) on first 10-14 days of each month if oligomenorrheic Mirena IUD Danazol (pseudo-menopause) Acute. if positive o Total abdominal hysterectomy and bilateral salpingo-oophorectomy o Adjuvant chemotherapy 15 Ovral is progestin (levonorgestrel) and estrogen (ethinyl estradiol) combination OCP OSCE-guide-III. maintain patient on monophasic OCP for next several months or consider alternative medical treatment Clomiphene citrate: consider in patients who are anovulatory and who wish to get pregnant Surgical: o Endometrial ablation.Physical Examination  Treatment of AUB: Treat underlying disorders / if anatomic lesions and systemic disease have been ruled out.doc Page 154 of 255 . C&S and gram stain ─ Polyp: Sonohysterogram ─ Biopsy ─ If ectropion: stop OCPs. cautery (silver nitrate) Cancer uterus: Risk factors: ─ ─ ─ ─ ─ Early menarche Nulliparity Weight gain HRT / estrogen therapy Unopposed estrogen Management: ─ Endometrial biopsy. severe DUB    Replace fluid losses. OCPs. if she does not want to become pregnant  Dysmenorrhea: ─ DD: PID. was it painful? (painless  anovulatory)  Did you use any contraception? When did you stop? Why? HPI AS  Any chance you are pregnant? How do you know for sure?  Any nausea / vomiting? Breast engorgement? Frequency? DD  For how long have you been trying to get pregnant?  Any previous pregnancies? Abortions?  Constitutional symptoms?  Are you under stress? Hypothalamus  Excessive exercise?  Any concerns about your weight? (anorexia)  Any headache? Vomiting in morning? Visual changes? Pituitary Difficulty seeing to sides? Milk secretions from breast?  History of thyroid disease? Heat/ cold intolerance? Bowel movements? Moist/ dry skin?  Do you have excessive hair growth? Acne? Did you Ovarian notice any weight changes? Hx of DM / thirsty / frequency? Fm Hx of PCOS?  Hx of chemotherapy? Radiotherapy? Hot flushes? Vaginal dryness? Soreness?  Any change in your voice? ↑ Muscle bulk?  Any repeated surgical procedures? D&C? Uterine G  Pelvic surgeries? Instrumentations?  PAP smear!  Any previous pregnancies? Abortions? O S  Hx of STIs PMH .Family hx of PCOS / infertility? SH . (2) hyper-androgenism (hirsutism or ↑ blood level). if she wants to become pregnant o Tranexamic add (Cyklokapron). any irregular bleeding? Spotting?  When was your first period? What age? Was it regular? For how long it was M regular / not regular? How often? How much? LMP?  When it was regular. for menorrhagia only o OCPs.SAD  Investigations: β-HCG / progesterone challenge test / hormonal assay (estrogen / progesterone / FSH / LH / prolactin / thyroid-TSH / serum testosterone. fibroid. ↑ exercise) to ↓ peripheral estrone formation o Metformin 500 mg PO tid o Clomiphene citrate.Any medical conditions? Psychiatric illness? . OSCE-guide-III. (3) PCOS by US ─ Investigations: β-HCG / US / High LH:FSH ratio > 2:1 / Fasting blood sugar ─ Treatment: o Lifestyle modification (↓ BMI.Any medications? Recent use of antibiotics FH .Physical Examination Amenorrhea CC Did not have periods for 6 months?! Did you seek medical attention? Any recent changes? Analysis of CC During these 6 months.doc    Page 155 of 255 .How do you support yourself? . total and free) / US / CBC  Ovarian causes of amenorrhea: PCOS /+/ Premature ovarian failure /+/ Androgen-producing tumours  PCOS (poly-cystic ovarian syndrome): ─ History: previous pregnancy / contraception hx ─ To diagnose PCOS: must have 2 of 3 criteria: (1) oligomenorrhea / irregular menses for 6 months. endometriosis ─ Investigations: U/S to exclude other conditions ─ Treatment: NSAIDs (ibuprofen 400 mg tid). For that reason.Are you aware of sexual cycle and ovulation (both you and your partner) . And I would like to assure you that all the conversation is strictly confidential and I will not release any information without your permission! Introduction CC Analysis of CC For how long have you been trying to conceive? .Does he complain of any testicular problem. but it is important to ask. semen analysis? PMH . e. d 21-22 / US) ─ Tubal patency (HSG / laparoscopy) When to start investigations: ─ If the woman is < 35 years  after 12 months of trying to conceive ─ If the woman is 35-40 yrs  after 6 months of trying to conceive ─ If the woman is > 40 years  start investigations after 1 months of trying to conceive OSCE-guide-III.g.Does he complain of any penile discharge? .g. I am going to ask some questions about your health and your partner health.Have you ever had Pap smear before? When was the last time? Any reason (if long time)? What was the result? .Allergies .Was he sexually active before? Did he father kids from another partner Partner before? history .Have you ever been pregnant before? Any abortions (termination)? Or Obstetric Hx miscarriages (spontaneous abortion)? . varicose veins.Did you use contraception? What? Since when? When was the last time? Hx .Physical Examination Infertility Transitional statement before going in details with the history: In order for a couple to achieve pregnancy. polyps)? Gynecology .Anovulation (irregular cycles / painless / no pre-menstrual syndrome) HPI DD .Family history of: repeated abortions / CS / congenital anomalies / twins Coital history . some of these questions are personal.doc Page 156 of 255 .How frequent? Regularity? .Do you have hx or were diagnosed with any gyn disease (e.Tubal occlusion: surgeries / STIs and PID / IUD .History of chemotherapy and radiotherapy .Did he receive any chemo or radio therapy? .For how long are you in this relation? For how long have you been trying to achieve pregnancy? . inflammation? .Endometriosis: dysuria / dyspareunia / dyschezia / back pain .Did he go through any investigations.PCOS: Do you have excessive hair growth? Acne? Did you notice any weight changes? Hx of DM / thirsty / frequency? Fm Hx of PCOS? .DM FH Infertility SH Investigations: ─ Semen analysis ─ Ovulation documentation (mid-luteal phase progesterone. both partners should be capable of having children and relatively healthy. because this is my first time to see you. is that ok with you? . how did this affect your life? Ethical challenge: travel permission o Actually I am here to get a note. I need to ask you some questions.When was your last f/u visit? .Number of babies? . is your baby kicking like before? .Amount of fluids? Obstetric history: any pregnancy before / any similar conditions? Gynecological history PMH: high blood pressure Social history: SAD / support / home environment OSCE-guide-III. was he hurt? Was anyone else hurt? When was that? It must be difficult.It looks like it is an important trip for you. .Are you under regular follow-up? o Yes  proceed to history o No  any reason? My husband had a car accident! I am sorry to hear that.Location of the placenta? .doc Page 157 of 255 .What was your BP? Was there any headache? .Was there leg swelling? Weight gain? Did they do urine test? .Have you done your U/S? How many times? When was the last time? . before we proceed.Good afternoon Ms … I am Dr … I understand that your blood pressure was measured and urine test was done. . however. to get a better understanding of your health condition. I have the results with me and I will discuss it with you.Dripping / Discharge .Is this you first time to have these checks during your pregnancy? . BP 160/110. Like the B12 results case Introduction I will discuss results with you Ethical challenge: travel permission History Last visit history / pre-eclampsia Make sure the mother and baby are stable U/S Obstetric history / Gynecological history PMH / Social history Counselling Explain what is pre-eclampsia Serious concerns with pre-eclampsia Management Hospitalize If insisting to leave  sign a LAMA Introduction . usually pregnant ladies do not travel during this time of pregnancy! o It is a business trip that would save our financials. and I will start by asking you some questions: History Last visit history / pre-eclampsia .Bleeding . I really need to travel.What type of notes? o Travel note. Manage.I see it is important for you. let me check your health condition first. +++ protein in urine.EDD (expected date of delivery) U/S .Contractions / pain .Physical Examination Counselling – pre-eclampsia 36 weeks pregnant lady comes for f/u visit. However. .How about before being pregnant? Any hx of high blood pressure? Make sure the mother and baby are stable: ABCDE .Activity of the baby. the most likely diagnosis is a medical condition called “pre-eclampsia” OR “pregnancy-induced hypertension”. that is why you have headache. o This includes your brain. visual changes. Ca channel blockers OSCE-guide-III.If all above are stable. the patient will start to seize. that is why you have +++ protein in urine. we need to admit you to the hospital to monitor you. with very serious and may be fatal consequences. urine dips and fetal kick counts as outpatient. liver function tests. just 2 days and I will come back. the most likely explanation for your increased is “pre-eclampsia”. . your dad had HTN. that is why you have ↑ BP. however the only treatment is delivering the baby. and they allow you to take the trip. Serious concerns with pre-eclampsia . I need to travel. subsequently ↓ the amount of oxygen and nutrients.Assess severity including good history and physical exam focusing on heart. . Treatment Plan .Imagine that I give you the note. o But doctor. the obstetrician will assess you and may consider delivering the baby now. will not be able to breath and turn blue.Your blood pressure is 160/110. like giving a doctor note that she needs to be hospitalized.Measure L/S ratio of the baby. which is a serious condition.A condition in which. that is why you may have abdominal pain.doc Page 158 of 255 . all of these are due to this condition. but we have a serious situation here. and lived with it. you start to fall down and seize. but you have “pregnancy-induced” HTN. . o This includes your liver.What we need now is to admit you to the hospital and arrange for obstetrical assessment. and this is a very serious condition.Blood pressure controlled often with labetalol.These are different conditions. lose conscious. On the long term this will lead to some injury and even damage to the baby AND the mother. that is why he is not kicking like before … This is not because of your pregnancy. fetus.What happens is that there is a narrowing of blood vessels. this leads to ↓ the amount of blood reaching the baby. consider daily check. 2 hours later while you are in the plane. Management .I would like to ask more questions to see how it affected you! o My dad had HTN.I understand your concern about traveling. o This includes your heart and blood vessels. o This includes your kidney. If any of above unstable may need to hospitalize as inpatient for close monitoring . What will happen? Nobody will be able to help you. The only resolution for this is delivering the baby. give corticosteroids for lung maturation . Then. Uric Acid) . and the urine test shows protein in large amount (+++) which is not normal.If insisting to leave  sign a LAMA (leaving against medical advice) . . without the proper medical care. . lungs.Suggest solution for her business travel. . which is high. .Physical Examination Counselling Explain what is pre-eclampsia . I am ok. o This includes your baby. reflexes. patients having pre-eclampsia will end up going to the next stage which is “eclampsia”. o Have you had hx of headache?  OCD / PQRST (not detailed) o Nausea / vomiting o Change in your vision? Flashing lights? Flying objects? o Any abdominal pain in your upper right part of your abdomen? o Any bruises? Yellowish discoloration / itching / dark urine / pale stools? o Any chest pain / heart racing / SOB? o Any weakness / numbness? o Any swelling in your body / face/eyes? Did you feel your shoes tight? o Did you gain weight? o Any changes in the urine? Frothy? Burning sensation? . or it might be related to placenta. We believe it is imbalance of hormones.By this you endanger your life and your baby’s life.We do not know exactly why patients have pre-eclampsia.Based on all this. . urine analysis and BW (important CBC.MgSO4 and delivery .The concerns we have is that we can not predict the outcome. do you any idea what is “e0clampsia”? . Did you have your blood sugar checked? How about before pregnancy? Make sure the mother and baby are stable: ABCDE .  Empathy: it looks like you are doing through difficult times! How are you coping?  Offer social support: being pregnant lady without support. I will make sure to connect you with social worker who will help you with proper support (housing / financially / for both of you and the baby) Last visit history / pre-eclampsia .What was your BP? Was there any headache? .Was there leg swelling? Weight gain? Did they do urine test? .Physical Examination Caesarean Section – Counselling – wants to have CS Young 18-20 years old pregnant lady would like to have CS.How do you know it is painful? o I had previous abortion OR o I attended my sister delivery and it was very painful experience . but before that I would like to ask you is there any reason you would like to have CS? o I do not want to have this severe pain! .Dripping / Discharge . we will discuss this in details.How about before being pregnant? Any hx of high blood pressure? .Bleeding . counsel her. is your baby kicking like before? . Introduction Any reason you want to have CS? Social issue History Last visit history / pre-eclampsia Make sure the mother and baby are stable U/S Obstetric history / Gynecological history PMH / Social history Counselling Address patient concerns Why not caesarean section? Management Refer to obstetrician Spend some time to think / stabilize Introduction Any reason you want to have CS? .When was your last f/u visit? . but people differ! And within few minutes I will be explaining different options to control labour pains! History .When was that? Did you attend? o Congratulations! How is your sister doing? How is the baby? o I understand that you saw her in pain.Contractions / pain . was it planned? How do feel about your pregnancy? How is the feeling of your partner? o Are you under regular follow-up? NO! Any reason? There may be social issue here.Activity of the baby.doc Page 159 of 255 .I understand that you are here to discuss the possibility of CS.Let me ask you some questions to assess the condition first! o How do you feel? How is your mood? o How about this pregnancy. you have priority and there are a lot support and resources in the community.EDD (expected date of delivery) OSCE-guide-III. Any pregnancy before? Any abortions or miscarriages? . And if at any time you have any questions or concerns.Physical Examination U/S - Have you done your U/S? How many times? When was the last time? Number of babies? Position of the placenta? Amount of fluids? Obstetric history: GTPAL . - I will give you some brochures and web sites so that you can read more about that. Genital herpes PMH: Medications / allergy / blood transfusion Social history: SAD / support / home environment Counselling Address patient concerns .However. I am not the person who makes the decision. I would recommend you spend some time to think about what I told you.What were the circumstances? How many weeks? . this is very effective and safe.I am going to refer you to the obstetrician. . used when there is a problem or contraindication for vaginal delivery and if there is an emergency situation that necessitates immediate delivery. for both the mother and the baby! . The natural route for delivery is the vaginal delivery. bleeding. and in these cases it is life saving. you can come to see me. Why not caesarean section . . when you start your labour. who will perform further and detailed assessment then discuss the results with you.I know that you are here to talk about CS. DVT. what is your understanding of CS? . and I mean what we call “epidural anesthesia”.doc Page 160 of 255 . and less likely infection. if your concern is the pain there is a lot of options to control it. and it will leave scar in your abdomen. they attend with you and they provide a lot of emotional support o Finally.It is commonly used obstetrical intervention. and if there is no real indication for CS. I appreciate you concerns. try to stabilize yourself emotionally. Where the anesthesiologist puts a needle into your back and injects a freezing substance that helps you to go through the delivery without pain (this is like the dentist freezes your mouth before doing painful procedures). one of them is called “doolas”. higher risk of infection. Management . I will connect you with the social worker. when you go there. you stay longer in the hospital. has the risks of bleeding. it is a major surgery. this should be decided by the obstetrician. .However. we have good measures.Meanwhile.How did you feel about it? How did you cope with that? Gynecological history: Fibroid. OSCE-guide-III. But first let me explain some facts about delivery.I would like to ask you. there are a lot of support groups. It might however cause headache.After all. o We can start by learning some relaxation techniques o And then on the delivery day. we would prefer to go for vaginal delivery. What is your understanding of cord prolapse? .Physical Examination Caesarean Section – Counselling – does not want to have CS Middle aged pregnant lady (36 weeks) is here to have her file. .What was your BP? Was there any headache? .How about before being pregnant? Any hx of high blood pressure? Make sure the mother and baby are stable: ABCDE . and then gets stuck and squeezed by the head.But first I would like to ask you some question. is your baby kicking like before? OSCE-guide-III. I would like to have delivery at home this time.Do you know what the type of your CS was? Classical! History . and I will ask someone to prepare a copy for you.doc Page 161 of 255 . Introduction Any reason you want to have your file? Concern Deal with the patient concern What was the type of your CS? History Last visit history / pre-eclampsia Make sure the mother and baby are stable U/S Obstetric history / Gynecological history PMH / Social history Counselling What is CS? The two types of CS Risks of vaginal delivery post CS Management Prepare a copy of the file Speak with your midwife Introduction Any reason you want to have your file? . This is your right. the baby will suffer from brain damage.That is why they had to do urgent CS. . Now she would like to deliver at home with the midwife.Were there any consequences? o How is the baby doing? How old is he? What can he do? Walk? Talk? o How about you? Any complications? Infections? Scars? Deal with the patient concern: . .What happened last time? o They told me the baby had cord prolapse! . A lot of my friends did it at home with the midwife and they say it is much easier and relax.It is a condition where the umbilical cord goes into the birth canal before the head.Is there any reason you would like to have your file? o Yes. which was a life saving procedure for the baby.This is a very serious condition. she had urgent CS for cord prolapse.I understand that you are here to have a copy of your file for the urgent CS you had 3 years ago.I understand your point of view. I am going to follow up with the midwife. is it ok with you! . Three years ago. o I did not like last time when they did CS at the hospital! . and it had to be done immediately. The cord delivers blood and nutrients to the baby. .Are you seeing obstetrician? Are you doing any regular follow-up visits? Any reason for that? o No.Activity of the baby. .Was there leg swelling? Weight gain? Did they do urine test? . but first let me ask you some questions about your health and your pregnancy! Last visit history / pre-eclampsia . If blocked for long time.When was your last f/u visit? . we can intervene in the proper time. like yours.Even though. I would recommend that you take your file and speak with your midwife. . but it is indicated and actually needed in urgent case. it is done less common. it is always CS.What were the circumstances? How many weeks? .Have you done your U/S? How many times? When was the last time? .doc Page 162 of 255 .Other than the pregnancy that you had CS 3 years ago. what is your understanding of CS? … . In case of counselling transverse CS: . it is the most common type. . if you want to try vaginal delivery. so that just in case any emergency might happen.Speak with your midwife: . to avoid the tearing pressure of the uterine contractions during vaginal delivery.It is commonly used obstetrical intervention. you can come to see me. Because you may end up losing your life and/or losing your baby.Location of the placenta? . o The classical or vertical type. but the risks of having uterine rupture after classical CS is 12%. any pregnancy before? Any abortions or miscarriages? .I will give you some brochures and web sites so that you can read more about that.Due to the scar formed after the CS procedure. . it is always recommended to deliver by CS. and I am sure she will explain the situation to you.If you decide to go for vaginal delivery. my concern is that the scar might undergo severe tearing pressure and might rupture.There are two types of CS: o The transverse (done at the lower segment of uterus).How did you feel about it? How did you cope with that? Gynecological history PMH: Medications / allergy / blood transfusion Social history: SAD / support / home environment Counselling What is CS? The two types of CS . its advantages include: smaller scar and better healing.For that reason: once classical CS. of which 10% of cases end up losing their lives. As it allows quick access and fast delivery. used when there is a problem or contraindication for vaginal delivery and if there is an emergency situation that necessitates immediate delivery.And if at any time you have any questions or concerns. .I would like to ask you. as we cut through the muscle fibers of the uterus it produces weaker scar. . .Number of babies? . because in some cases (like cord prolapse) we can not afford even few minutes more. Risks of vaginal delivery post CS . This is an obstetrical emergency that necessitates immediate intervention. Management .Risks of having uterine rupture after transverse CS is 1%. for both the mother and the baby! . we can not take the risk to try this at home.I do not want to scare you.Amount of fluids? Obstetric history: GTPAL .Physical Examination - Bleeding Contractions / pain Dripping / Discharge EDD (expected date of delivery) U/S . and we share the same guidelines. we can try this in the hospital. OSCE-guide-III. and in these cases it is life saving.I will ask someone to prepare a copy of your file . which will lead to massive bleeding.I am sure that your midwife is highly trained and qualified. Physical Examination OCPs / Contraception Counselling XX is a 16 years old girl. increase thickness ─ Available forms: Copper / hormoneof cervical secretion coated ─ 21 tab + 7 sugar pills  28 days ─ Structural uterine anomalies Contraindications ─ HTN / CAD / Cerebro-vascular disease / DVT ─ History of ectopic pregnancy ─ Breast or gynecological cancer (past personal and family hx) ─ Undiagnosed vaginal Bleeding ─ Undiagnosed vaginal Bleeding ─ History of PID(s) ─ Active liver disease ─ Risky behaviour ─ Smoker > 35 yrs ─ Migraine Benefits ─ Regulate periods ─ Longevity ─ Independence to coitus or compliance ABCD: ─ Improves anemia (↓ bleeding) ─ ↓ benign breast lesions ─ ↓ ovarian cysts and cancer ─ ↓ risk of uterine cancer ─ ↓ dysmenorrhea Available methods OSCE-guide-III. She would like to get information about OCPs. For the next 10 minutes talk to her Introduction Concerns ─ ─ ─ History Exclude pregnancy M G ─ ─ ─ ─ ─ O S ─ ─ ─ ─ Do you have any concerns? Why do you need / think about contraception at this point? Have you used contraception before? What is your previous experience? Why did you stop it? Any chance you are pregnant? How do you know for sure? Any nausea / vomiting? Breast engorgement? Frequency? Painful periods? / Irregular? / Heavy bleeding? Last PAP / any abnormal PAP Previous D&C GTPAL / IUD is not recommended in nullipara DETAILED If risky behaviour: OCPs will not protect against STIs Risky behaviour or previous STI/PID: NO IUD Definition: birth control is an umbrella term for several techniques and methods used to prevent fertilization ─ Hormonal (OCPs / implants / injections) ─ IUD (contragestion: prevents the implant) ─ Barrier methods (condoms / diaphragms) ± spermicidal ─ Behavioural (fertility awareness/timing) / coitus interruptus ─ Post-coital contraception ─ Sterilization (male / female) OCPs IUD Mechanism of ─ OCPs are hormones: estrogen and ─ Mechanically prevents the implantation of action progesterone the fertilized ovum ─ OCPs prevent ovulation.doc Page 163 of 255 . diaphragm.Counsel for safe sex .Discuss various other alternatives.  What happens if I missed 1 or 2 pills (OCPs)? .Bring the patient back after 1 month for the bilateral tubal ligation procedure. and document blood pressure. family coercion) 3. for the next 2 days .1% – 5% ─ 0.1 missed pill  take 2 pills the next day . the operative risks. preferably immediately after menstruation to decrease possibility of pregnancy OSCE-guide-III.Counsel the patient on the permanent nature of the procedure.Because it depends largely on the knowledge.B.5% – 2% OCPs IUD ─ Will do physical and pelvic examination. make sure she knows everything about all of them (OCPs. experience of user and the usage technique. vasectomy) 2. recent emotional trauma.doc Page 164 of 255 . perfect-use versus typical-use failure rates. pap smear ─ Brochures and information ─ Follow up within 1 month ─ How we will start: ─ First day of next menstrual cycle ─ ─ Put package in obvious place that you see frequently ─ Take the pill in the same approximate time every day ─ Use another method of contraception for the first TWO months ─ If you miss a pill: see below ─ ─ Failure rate Plan NOTES  N.OCPs do not protect against STIs  Failure rates are very high with behavioural methods: .2 missed pills  take 2 pills a day.If missed more than 2 consecutive pills: use a backup method of contraception simultaneous to finishing up packet of pills (2 pills a day) until next menses  Woman comes to request sterilization (tubal ligation) 1.: if woman with risky behaviour: . condoms. and the chance of failure (1 in 200) Counsel about the risks of regret for the decision (young age.Physical Examination Side effects Does not prevent STIs ─ Heavy periods ↑ risk of DVT / CAD ─ If hormonal coated: prog side effects: TRANSIENT: headache / wt gain / mastalgia ─ Breast tenderness ─ Weight gain ─ Headache ─ Nausea / vomiting ─ 0. IUDs. ↓ benign breast lesions .Headaches . can combine 2 methods Page 165 of 255 .Breast tenderness .Irregular bleeding .Weight gain .Does not prevent STIs .Amenorrhea after 1 year .Nausea / vomiting .Headache .Effective .Effective Nor-plant .doc Available Combined pills (E+P): low dose estrogen (20.If hormonal coated: prog .Independence to coitus or compliance .NO absolute contraindications .Hormone coated side effects: headache / wt gain / mastalgia 0.Independence - Contraindications Pregnancy Un-dx vag bleeding Cerebro-vascular dis / CAD Active liver disease Hormone-dep cancer Smoker > 35 yrs Hx of DVT / PE Migraine - Pregnancy Un-dx vag bleeding Structural uterine anomalies Hx of PID(s) Risky behaviour Hx of ectopic pregnancy Condoms Diaphragm Behavioural OSCE-guide-III.1% – 5% TRANSIENT: .↓ Dysmenorrhea .Improves anemia (↓ bleeding) .Independence to coitus or compliance Side effects .↓ ovarian cysts and cancer .Longevity .Regulate periods ABCD: . 35.Physical Examination OCPs IUD IM Injection Advantages .Heavy periods .High failure rates up to 25% .↓ risk of uterine cancer .Irregular bleeding .Copper .Mood changes Every 5 years < 1% Depoprovera SC implants .Delayed post-use fertility Every 3 months < 1% - . 50) Failure rate 0.↑ risk of DVT / CAD Un-dx vag bleeding Acute liver disease Thrombophlebitis Thromboembolic disease 14% 20% .Longevity .↓ dysmenorrhea .6% – 2% .To decrease the failure rate. vaginal dryness. heart attacks and breast cancer.doc Estrogen alone Combined estrogen and progestogen Selective estrogen receptor modulator (SERM) Menopausal symptoms For SHORT term only. cholesterol increases the risk of these dis much more than HRT. you do not have the risk factors for cancer. In your case. on the other hand. and will protect you against osteoporosis … In medicine we always weight risk / benefits … Page 166 of 255 . 1 – 2 years Osteoporosis Hx / PE / baseline investigations Media spoke that HRT increases incidence of stroke. obesity. and it will be beneficial for your hot flashes. smoking.OB-GYN HRT counselling Introduction / overview History General How do you describe your general health? Support DETAILED D&C / OCPs / PAP / mammogram / BMD GTPAL Dyspareunia  Irregular menstrual cycles  Vasomotor symptoms: sweating / hot flashes (hot flushes) / palpitations  Uro-genital symptoms: vaginal dryness / soreness / superficial dyspareunia / urinary frequency and urgency  Neurologic symptoms: mood changes / sleep disturbance / depression / anxiety Breast Cancer Uterine Cancer Ovarian Cancer  Early menarche  Obese  Late menopause  Diabetic  OCPs  Nullipara  No breast feeding  PCOS  Age  Past medical history of cancer or biopsy  Family history  Screen the first major risk factors for osteoporosis (see osteoporosis counselling case) What is your understanding about HRT?  Local preparations: creams / pessaries / rings  Systemic formulations: oral drugs / trans-dermal patches and gels / implants   M G O S Menopausal symptoms Risk factors for CANCER Risk factors for Osteoporosis Counselling Common forms / products of therapy Indications for HT   Contraindications to HT Pre-treatment evaluation Adverse effects and risks Any concerns? OSCE-guide-III. this was done by the (Women's Health Initiative). Toward the end of her reproductive years when a woman reaches menopause. Progesterone concentrations rise in a cyclical fashion to prepare the uterus for possible pregnancy and to prepare the breasts for lactation. . Estrogen regulates a woman's monthly menstrual cycle and secondary sexual characteristics (e.Undiagnosed vaginal bleeding . the severity of which can vary widely.There is association between HT and uterine hyperplasia and cancer .Breast tenderness .The reproductive years of a woman’s life are regulated by production of the hormones estrogen and progesterone by the ovaries. breast development and function). which may lead to several symptoms.Mood swings (associated with use of relatively androgenic progestogens) . Heart disease Required baseline investigations .Mammography Possible adverse effects are as follows (similar to OCPs): .OB-GYN Introduction / overview: . and debate regarding its risk-benefit ratio continues Contra-indications of HRT: No absolute contraindications of hormone therapy have been established.Nausea / bloating .HT may slightly increase the risk for breast cancer .Electrocardiography . . However.There is increased risk of thromboembolism with HT OSCE-guide-III.Thromboembolic disorders / DVT .Acute liver disease . HT is designed to replace a woman's depleting hormone levels and thus alleviate her symptoms of menopause.Fasting lipid profile . However.Endometriosis / Fibroids .Pap test .Breakthrough bleeding .CBC .Ultrasonography to measure endometrial thickness and ovarian volume . circulating levels of estrogen and progesterone decrease because of reduced synthesis in the ovary.Fluid retention . HT is relatively contraindicated in certain clinical situations (similar to OCPs): .Urinalysis .Diabetes. HT has been linked to various risks.Weight gain (equivocal finding) .Hormone therapy (HT) involves the administration of synthetic estrogen and progestogen.g.Breast and/or endometrial cancer . HTN.doc Page 167 of 255 . it prepares the body for fertilization and reproduction. In addition.Blood sugar levels . Introduction Concerns History Inform the patient about HIV Plan Workup HIV infection / fatal disease / will impact her family .I am going to speak with the patient now. upset because she had just had needle stick after she gave an IV injection to a patient. OSCE-guide-III.doc Page 168 of 255 . guidelines state we should assume you were exposed and give prophylaxis treatment: 4 weeks of 2 anti-virals (the basic regimen) .Risk of blood transmission is: 0. NO post-exposure prophylaxis is needed . and I will come back to you with his decision. and mother-to-child transmission (MTCT).Risk of female to male transmission is: 0.Complete immunization record. hepatitis C. sexually transmissible virus.If he agrees.Intravenous drug abuse .Human immunodeficiency virus (HIV) is a blood-borne. which can occur during the birth process or during breastfeeding. If he is HIV negative.Hepatitis B vaccination 3 doses + immunoglobulins (immediately) . to simplify this.HIV positive mothers should not breastfeed their babies . .What is HIV? Major pathogens of concern! First of all.Are you pregnant? Risk of intrauterine tx is: 3% with treatment and 30% without treatment .3% for percutaneous exposure .If he refuses or if he is HIV +ve.OB-GYN Needle Stick Counselling – HIV You are about to see Mrs … 33 years old female nurse. shared intravenous drug paraphernalia.03% . Counsel her. .Previous occupational exposure to body fluids . The virus is typically transmitted via sexual intercourse.Any other questions or concerns? What is HIV? .Patient should receive ttt (not teratogenic) .Advancement of HIV treatment Address pregnancy concerns: . and hepatitis D. These pathogens are viruses that require percutaneous or mucosal introduction for infectivity.The major pathogens of concern in occupational body fluid exposure are HIV. including tetanus and hepatitis B .Blood tests for the patient if possible and for the exposed . hepatitis B.Sexual history .Can you tell me what happened? . let me tell you the transmission rates: (no accurate studies) .I will speak with the patient. we will have to assess what is called “exposure code” and match it with “HIV status code”. The major target organs are the immune system (HIV) and the liver (hepatitis). hepatitis A. explain the whole situation and ask him to consent for HIV status . we will know possibility of tx to you. it is not cured. we can ask Dr …. but controllable. we can not do this HIV status test with the patient knowledge and consent. may be he knows! o May be Dr … knows the patient status or may not. we started by giving many pills q4h.doc Page 169 of 255 . at 1 month. with ↑ efficacy and ↓ side effects.It used to be fatal . but if the patient refuses to let us know.A lot of anti-virals were developed since 1990s till now.Victim/health care worker o HIV.Source patient (if available) o HIV o Hepatitis B antigen o Hepatitis C antibody . however. we had just send his blood to the lab! o You are right. we can not ask Dr … about this information unless the patient consents we can ask the surgeon. testing now. OSCE-guide-III.Even if the patient refused. and ask his permission that we do this HIV statue test. To decide when to start treatment Workup: . now it is once or twice a day . his surgeon.We follow the guidelines with monitoring of what is called “viral load” and “cell counts” of the patient immunity cells.Currently.OB-GYN Advancement of HIV treatment: . it is patient confidentiality. I will go to speak with the patient. and at 3-6 months o Hepatitis B surface antibody / titre (if vaccinated) o Hepatitis C antibody. testing now and after 4 and 8 weeks . . o What I am going to do –to help you– is that after we finish. this will save us the whole prophylaxis plan.Prior to initiating retrovirals: o Pregnancy test (stat) – if she is not pregnant o CBC count with differential and platelets o Serum creatinine/BUN levels o Urinalysis with microscopic analysis o AST/ALT levels / Alkaline phosphatase level o Total bilirubin level The ethical questions that might arise: .We can solve all this by calling the lab and adding HIV status check for the list of blood works of the patient. may be in the future they can develop a cure to it . we need to speak to the patient first. explain the whole situation to him. Inguinal swellings? Nearby systems .Smoking AS Same system . is this right? How can I help you today? HPI M  Previous Pap test? How many? How frequent? Any abnormal Pap test? G Any previous colposcopy?  Contraceptive history GTPAL O  RISK factors for cervical dysplasia: S . it undergoes rapid growth. to detect it before it turns into malignant tumour.doc Page 170 of 255 . . frequency? .OB-GYN Counselling – PAP smear CC I understand you are here because you have some inquiries/worries about your last PAP test.Constitutional symptoms PMH Any allergy / medication / disease FH Gynecological tumours SH If teenager: HEAD SSS COUNSELLING: .Have you [read / talked to someone / searched the internet] about this issue? Worried about PAP results .Any blisters / warts / ulcers? .Have you had any experience with … in the past? .Discharge? Itching? Redness? .At the cervix there is transitional zone between two types of cells.Any pain? With intercourse? . that is why we do frequent PAP tests.The results come back from PAP test either ASCUS (Atypical squamous cells of Undetermined Significance) / LG-SIL (low grade squamous intra-epithelial lesion) / or HG-SIL (high grade squamous intra-epithelial lesion) OSCE-guide-III. It takes years from the moment it begins to grow abnormally to the moment it becomes malignant.Bowel movements changes? GIT symptoms .Early age of sexual activity . before it turns to serious disease (to early detect pre-malignant lesions).Abdominal pain  OCD / PQRST / ↑↓ General .What do you know about (LGSIL)? What would you like to know? .Risky behaviour: unprotected sex / multiple partners .PAP smear or test is done to screen for any changes that might happen in the cervix. it might turn malignant. if there is irritation due to HPV. .Urine changes? Dysuria. and you might need to take some OTC medications (Advil) for few days after the procedure. and you might experience a little discomfort and spotting for few days.doc Page 171 of 255 . no douching). .Colposcopy is a magnification of the cervix (10-12 times). application of this acetic acid may give an itchy sensation.Cone biopsy . we will do more investigations to establish a diagnosis and may need to do another larger biopsy called cone biopsy Treatment options: . . the gynaecologist will take a biopsy. and send it for further investigations.Laser . . o If the biopsy is negative. during which. . the procedure may cause some discomfort but is not painful. we will repeat the PAP after 6 months o If the biopsy is positive.You need not to have anything inserted into your vagina for 24 hours before and 2 days after the procedure (no vaginal intercourse.The gynaecologist will insert a speculum (the same instrument used for Pap test).LEEP (loop electrosurgical excision procedure) Colposcopy . OSCE-guide-III. she/he will need to take a biopsy. we will refer you to the gynaecologist who will perform special procedure. and then she/he will use a special magnification device (the colposcopy) to visualize the cervix. you will feel a punching sensation.OB-GYN - - - - For ASCUS: o Woman ≥ 30 yrs  HPV DNA testing  If negative  repeat cytology after 1 year  If positive  colposcopy o Woman < 30 yrs  repeat cytology in 6 months  If negative  repeat after 6 months  still negative  routine screening  If ≥ ASCUS  colposcopy For LG-SIL: o Colposcopy o Or repeat cytology after 6 months  If negative  repeat after 6 months  still negative  routine screening  If ≥ ASCUS  colposcopy For HG-SIL: o We send you for colposcopy For colposcopy.The gynaecologist will apply acetic acid (vinegar) that helps make the vascular patterns more visible.Then if the gynaecologist suspects a lesion. Rubella vaccine. Epilepsy. MR. This is her first experience. Blood Group & Rh group Allergy Medications FH of complicated pregnancies. abortions. exposure to chickenpox as a child.OB-GYN Antenatal Counselling o o Ms XX has missed her period for 2 wks. she did a home preg test which was positive. In the next 10 min. genetic & congenital abnormalities Home. violence.doc HTN. Heart disease. please talk to her and give her necessary advices about her pregnancy. counsel Pregnancy     M G O S PMH        FH  Social Hx      Concerns Was this planned pregnancy? What is your feeling about that? And your partner feeling? congratulations ☺ How do you describe your general health? Symptoms suggesting pregnancy: any nausea / vomiting? Breast engorgement / heaviness? Frequency? LMP. Kidney disease. HBV. A 38 yrs old pregnant lady came to you because she is concerned about problems during advanced-age pregnancy. support Environment: work Activity: exercise / any pet Diet SAD Address any problem you find while you’re taking history Page 172 of 255 . HIV. Diabetes. calculate due date Pap smear?  OSCE-guide-III. Glucose challenge test at 24 weeks Risks of Down syndrome are: 1/400 at 30 yrs of age. no x-rays Smoking increase the risk of abortion. 1200-1500 mg per day / iron: 1 mg/d in T1. a daily caloric increase of -100 cal/d in the 1st trimester. avoid raw or processed meat Haemorrhoids. VDRL. you can take Lactulose for this. swimming. -300 cal/d in the second and third trimesters and -450 cal/d during lactation. 1/200 at 35 yrs of age. premature delivery No safe level of alcohol during pregnancy. β-hCG.doc Page 173 of 255 . 1 pound /week in 2nd half. Hepatitis.OB-GYN Counselling:                   To ensure healthy outcome of the pregnancy I need to see you on scheduled visits. Lytes. heartburn and increased vaginal discharge are common Will gain weight. blood work including CBC. and then every week thereafter and till delivery Today we’ll do physical examination including pelvic exam. connect to support groups and classes for pregnant women OSCE-guide-III. Pap smear if more than 6 months. INR/PTT. you can take daily multi-vitamins (avoid excess vitamin A) Important nutrients during pregnancy: folate. every 4 weeks till the 28th week. avoid strenuous activities Stay away from cats’ litter No medication without asking your doctor.4-5 mg per day / calcium. better to avoid it totally Offer brochures. Canada's Food Guide to Healthy Eating suggests 3-4 servings of milk products daily (greater if multiple gestation). back pain. and 1/100 above 40 yrs of age  we try to anticipate it by US and integrated prenatal screening then confirm it by amniocentesis U/S for nuchal translucency: at 12 weeks IPS I: 11-14 wks /+/ IPS II: 15-18 wks (Maternal serum alpha-fetoprotein. total of 25-35 pounds in average Exercise is OK… walking. Urea. uE3– Unconjugated estrogen) Amniocentesis (U/S-guided trans-abdominal extraction of amniotic fluid / for identification of genetic anomalies): at 15-16 wks.5% risk of spontaneous abortion and risk of fetal limb injury You need a well balanced diet. Creatinine. 5-10 pounds in 1st half. ± ECG. 0. If you do not consume an adequate diet. Blood Type. 0. then every 2 weeks till the 36th week. 4 mg/d in T2 and > 6 mg/d in T3 Pregnant ladies tend to have constipation. Anatomy US at 20 weeks. Urine dip and microscopy. ± HIV. LBW. Rubella antibody. o Why did you have laparoscopy done and what was your doctor’s concern? o You have endometriosis: ─ This means implantation of the interior lining of the uterus somewhere in other places outside the uterus including the ovaries. XX to discuss the result of her laparoscopy & inform her that she has endometriosis. You are about to meet Mrs.g. diarrhea - Menstrual history – brief Gynecological history Obstetric history – GTPAL Sexual history – brief - Past medical history: HTN. the supporting structures of the uterus or on the intestine (draw a picture of the uterus and ovaries for the patient). Allergies / medications / hospitalization / surgeries / blood transfusion Family history of abortion Social history: smoking. For the next 10 minutes. drugs / work / home environments / support - 16 Conclusion: endometriosis Plan: o Will do physical and obstetric examination o Give her treatment options ─ Medical:  NSAIDs – e.OB-GYN Endometriosis You are covering for your colleague Dr. alcohol. Do you need pain killer? ─ Infertility: I understand your frustration. Smith. naproxen sodium (250 mg PO bid) 16  Pseudo-pregnancy: OCPs trial for 6-12 months (Ovral 1 tab PO od)  Pseudo-menopause (only short-term <6 months): Danazole (weak androgen / Side effects: weight gain. fluid retention. ─ Sometimes it runs in the family o I would like to ask about some symptoms (to fit everything together) ─ Pain: Analyze (OCD / PQRST). - Introduction: I would like to discuss the result of your laparoscopy but I need to get some information. dyschezia. please talk to her& address all her concerns. this outside tissue also begins to bleed. blood group & Rh. ─ This may cause infertility in some people. acne. Relation to the period. dyspareunia. voice change) ─ Surgical treatment:  Conservative laparoscopy: laser ablation / resection of implants  Definitive: bilateral salpingo-oophorectomy ± hysterectomy o Brochure & support groups Ovral is progestin (levonorgestrel) and estrogen (ethinyl estradiol) combination OCP OSCE-guide-III.doc Page 174 of 255 . dysuria. kidney disease. hirsutism. Diabetes. This explains the painful periods. Dysmenorrhea. ─ During periods. For how long have you been trying to conceive? Are you currently sexually active? How frequent? ─ Irregular vaginal bleeding  analyze ─ Frequency ─ Blood with stool. OB-GYN Woman wanting an abortion - History Pregnancy Social Abortion Depression - Pregnancy (LMP, symptoms, how did you find out, Rh status) How do you feel? How is the feeling of your partner about the pregnancy? ─ Gynecological history: contraception history, surgeries, infections, PAP ─ Obstetrical history: hx of previous pregnancies / GTPAL ─ Partner involvement? Abuse, rape? ─ Support; family, friends, spoken to any one else about this? ─ Education, Religious beliefs? ─ SAD ─ What are your thoughts regarding abortion? ─ Exclude depression: MI PASS ECG PMH / FH / SH ─ Counselling Pregnancy Social Abortion Depression OSCE-guide-III.doc Be supportive and not judgmental Health while pregnant: ─ Counsel for nutrition, exercise, and activity ─ Social support: it is important to communicate with support groups ─ Does not need to rush decision ─ Abortion options are available in Canada; most hospitals will do less than 14-16 weeks, no problems with some up to 20 weeks. Private clinics also provide the service ─ No legal gestational age where it can no longer be performed ─ Most commonly done via D&C under mild anesthetic ─ Discussion about contraception especially if not the first time (not necessarily at this meeting) ─ Alternative choices: adoption / or rethink about keeping the baby with the help of social support ─ Normalize the patient depressed mood and feelings in view of circumstances ─ Close follow-up ─ May refer to psychiatrist ─ Page 175 of 255 OB-GYN Osteoporosis – Counselling / OR / Short Case Age related decrease in bone mass, starts at 35 years old, accelerated post-menopause, leading to bone fragility and an increased risk of fracture. May manifest later as kyphosis, loss of height and increased abdominal girth Risk factors for osteoporosis: 1- Personal history of fragility fracture (above the age of 40 years) 2- Maternal history of hip fracture 3- Malabsorption syndrome (celiac disease / inflammatory bowel syndrome) 4- Prolonged use of corticosteroids (> 7.5 mg/day for > 1-3 months) 5- Primary hyper-parathyroidism 678910- Weight < 57 Kg Current smoker Premature ovarian failure (female on Tamoxifen for breast cancer / surgical menopause) Male on androgen-deprivation therapy for prostate cancer Heparin or anti-epileptic use or biologics (anti-cancer treatment) Investigations: - BMD: Age group When to do BMD - < 50 years If > 2 of the first (5) risk factors 50 – 65 years If > 2 of any from the list of risk factors > 65 years Always do BMD, screen even there is no C/O Blood work: o Serum calcium and phosphate levels o Alkaline phosphatase o Creatinine o SPEP (serum protein electrophoresis) o PTH (para-thyroid hormone) o Give vitamin D for 2-3 months, then assess the level, if > 0.75 nanogram  it is normal and do not repeat it again Treatment: - Based on BMD, risk factors, age of patient  Fracture Risk Stratification  low, moderate, or high LOW MODERATE HIGH - Life style If fragility fracture (in thoraco-lumbar x-ray) OR prolonged - Life style modifications use of corticosteroids modifications - F/U DEXA Medical NO YES after 5 yrs treatment - Life style modifications - Life style modifications - F/U DEXA after 2 yrs - Medical treatment Life style modifications - Weight-bearing exercises (walking, jogging) - Ca 1200 mg/d (including the dietary intake, avoid ↑↑↑ Ca) - Vit D 1000 IU/d if < 50 yrs, and 2000 IU/d if > 50 yrs - Smoking cessation - ↓ alcohol and caffeine OSCE-guide-III.doc Medical treatment - Bisphosphonates - SERM (Raloxifene): agonistic effect on bone but antagonistic effect on breast and uterus - Parathyroid hormone (PTH) - Calcitonin (if back fragility fracture + pain) - HRT Page 176 of 255 OB-GYN Pediatrics OSCE-guide-III.doc Page 177 of 255 OB-GYN History taking – Pediatrics History taking – Pediatrics   Introduction: Chief complaint 1234567- Analyze the CC Impact Rule out infection Differential diagnosis BINDE Past medical history Family history 1- Analyze the CC - BINDE screening - Mother attitude! - - 2- Impact 3- Red flags Rule out infection - 4- Differential diagnosis 5- BINDE 6- Past medical history 7- Family history 0-6 years BINDE Pregnancy / Birth Immunization Nutrition Developmental Environmental OSCE-guide-III.doc In any pediatrics case: always CONSIDER child ABUSE / NEGLECT  report to CAS TIME: Os Cf D: When did it start? How did it start? Sudden or gradual? - At that time, did your baby have any fever, flu-like symptoms? - Is it continuous or on and off? How often? Day and night? Character: - PQRST - If vomiting or diarrhea: COCA + BLOOD - Timing: is it related to feeding / meals? ↑↓ Factors: is it related to position? Meals? Is he drowsy? Floppy? Does he cry? Is it high pitched cry? Did you notice his suckling is weaker than before? Constitutional symptoms! R/O infection: Did you notice if your child has fever or skin rash? Cough / wheezes? Ear pulling or discharge? Runny nose? Foul smelling urine? Abdominal distension? Diarrhea? Rule out child neglect - BINDE screening - Mother attitude! Scan for risk factors for child abuse / neglect Hospitalizations / surgeries / blood transfusion Illnesses (cancer) / infections Medications / allergies. Travel Family history of similar disease in the family 6-14 years School performance 14-18 years HEAD SSS Home Education Activity Diet Suicide Sexual activity SAD (smoking / alcohol / drugs) Page 178 of 255 OB-GYN BINDE Pregnancy: - Was your pregnancy planned? If no  social issues - Were you having regular follow-up visits? How about U/S? Was it normal? - During your pregnancy, did you have any illness? How about any fever or skin rash? Have you ever been in contact with sick kids? Kids with skin rash or fever? Have you ever been in contacts with pets? - Did you take medications? Even OTC? Did you smoke? Drink alcohol? Have you ever tried recreational drugs? What about before pregnancy? - Were you screened for Hepatitis B virus? HIV? other diseases? There is screening test that we do a vaginal swab at 36 weeks called GBS, did u have it? - What is your blood group? What is your baby blood group? Birth: - Was your baby full term or not? - Was it a vaginal delivery or c-section? o If c-section  why? Was there any complication? Abnormality? o If vaginal  was it difficult labour? Prolonged labour? How many hours? Was there any early gush of water? How many hours? Did you need any help to make it easier; e.g. vacuum? - Did your baby cry immediately or not? Do you know what his Apgar score was? Did he need special attention? When did you leave the hospital? - Were there any bruises or swellings on your baby’s body? - Were you told that your baby had any special features? - After delivery, did you have any fever / discharge? Did you take any medications? Immunization: - Are your baby’s shots up-to-date? o Yes  when was the last shot? o No  any reason for that?  Our religion prohibits vaccination: ok, that is fine  We think vaccines cause autism: correct this info, vaccines are safe  We were busy  neglect concern  what is baby weight? Nutrition: - Weight: o o o o Weight calculation: What is your baby’s weight today? What was his weight at birth? What was his highest weight? Do you have access to growth charts?  If below 3rd percentile: underweight  If (at any time) he crossed (down) two major lines: failure to thrive At birth : x 5 months 1 year 2 years Kg :2x :3x :4x Kg Kg Kg More than 2 years old: [(age X 2) + 8] Kg Even low birth weight, catch up weight later, i.e. @ 1 year they must be around 10 Kg, not only 3 x OSCE-guide-III.doc Page 179 of 255 ¼ X by the second year. so the baby gains ½ X by the first year.OB-GYN - Height: o o - To calculate height:  At birth X cm 50 cm  1 year 1 ½ X cm + 25 75 cm  2 years 1 ¾ X cm + 12.5 94 cm  4 years 2 X cm 100 cm For each year: the baby gains (½) of the previous year increase. and other questions to assess his development.B. Gross motor Fine motor Sit alone / roll over 6 months Draw line 15 months Crawling 9 months Draw cross 2 years Standing / cruising 1 year Draw circle 3 years Walking 15 months Draw square 4 years Go upstairs holding 18 months Draw triangle 5 years Go downstairs 2 feet 2 years Tricycle 3 years Social Social smile Stranger anxiety Separation anxiety Says “NO” 6 weeks 6 months 9 months 2 years Speech / verbal Mama / papa 2 words beyond Ma. biscuits / bread / cereal?  Which started first? The diarrhea or this new food?  How many diapers do you change per day? (normally 5 – 6) Developmental: Now I would like to ask you some questions about the kind of activities that your child can do. No restrictions!  that is fine  Breast feed  if more than 4 months: • Any iron supplement? • Any Vit D supplement?  Formula: • Since when?  if since birth: any reason that you chose formula over breast feeding? • Which formula? Any recent change in formula? For any case of chronic diarrhea?  Do you give him solid food. Pa 2-3 words phrases Short sentences Speaks fluently 9 months 1 year 2 years 3 years 5 years N.5 87. 1/8 X by the third year. (autism / Down syndrome / child abuse): there is no stranger or separation anxiety.doc Page 180 of 255 . Diet: o o What do you feed your baby?  Everything we eat.5 cm  3 years 1 7/8 X cm + 6. OSCE-guide-III. How do you feel being a new mom? How do you feel about your baby? o How is your mood? You look down for me.Do you go to school? Do you like going to school? .What is your weight? What was your weight before? Suicide: .How many partners do you have? Do you practice safe sex? Smoking / Alcohol / Drugs: . Do you know any of your friends doing this? How about you? Have you tried that? .In ABUSE cases: tell me more about your childhood … HEAD SSS Home: . I would like to ask you some personal questions. it is important to ask it.How is the relation between you? Are they supportive? .Do you live in home (basement: mold) or apartment? Is it an old building (lead)? .What kind of hobby do you have? . what marks do you get? What about in the past? Activity: . or use recreational drugs.Sometimes people at your age might start to smoke.Do you or any body in the home smoke? Drink? Use recreational drugs? .With whom do you live? How is the relation between you? o How is the relation between you and the baby? o How is the relation between your partner and the baby? .Are you sexually active? When did you start? When was the last time? .Have you travelled recently? .Any siblings? Education: .With whom do you live? . any chance you are being depressed? Did you have depression before? . and it is confidential.Are you dating? Are you in relationship? .How do you support yourself financially? .How about your diet? What do you eat? Do you follow special diet? .For IV drugs: When was the last time? Did you share needles? OSCE-guide-III.Any chance that you might hurt yourself? Sexual activity: .Any other children in the house? . drink.Now.Is anyone of your family seeing a psychiatrist? Has mental illness? .OB-GYN Environment: .In EPILEPSY case: do you operate machines / drive / go hiking? Diet: .Which grade? Which subjects do you study? .doc Page 181 of 255 . do you Smoke? Drink Alcohol? Have you ever tried recreational drugs? .How about your marks.How is your mood? . Red flags / Rule out infection: . with jaundice since day 2 Introduction Differential diagnosis of newborn jaundice .BINDE .Who noticed it? You or someone else? When? Where did you notice it? How about his eyes? How about his feet? Is it spreading? Is it ↑ or ↓? .Hemolysis (unconjugated) .doc Page 182 of 255 .Birth  pathological unconjugated) .DD .Red flags / rule out infection Pathologic (anytime) .Physiologic (usually days 2-7)  CC unconjugated .Differential diagnosis: Physiological Pathological How do you feed him? Breast milk? Formula? ─ Infection should be ruled out or ─ Breast feeding jaundice: (or “lack of confirmed by now breastfeeding” jaundice): Not enough milk ─ Hemolysis:  dehydration  What is your blood group? Your ─ Breast milk jaundice: is more of a baby blood group? Father blood gp? biochemical problem (inhibition of  Rh incompatibility  IUGR bilirubin conjugation leads to increased  Were you screened for infections levels of bilirubin in the blood).Breast feeding jaundice .Infection  sepsis (conjugated or .OB-GYN Jaundice A new born 5 days old. I would like to know the name of your child? … This is a nice name.Did you notice his suckling is weaker than before? 3.Analyze the CC: .Did you notice if your child has fever or skin rash? Cough / wheezes? Discharge from his ears? Runny nose? Foul smelling urine? Abdominal distension? . I understand that you are here because your son has jaundice (or is yellow). during pregnancy? Treatment: substitute with formula ─ Biliary atresia ─ Hepatitis: neonatal OSCE-guide-III. before I proceed. I am Dr ….Impact / Consequences: . is it pale? 2.Nutrition  physiological FH Introduction: Good morning Mrs ….How about his urine.Impact / consequences .Is he drowsy? Floppy? .When did it start? Early in the 2nd day (or before: pathological) or late (pathological or physiological)? .Does he cry? Is it high pitched cry? .Analyze the jaundice (OCD) . 1. is it darker? And stools.Any night sweats / chills? Any lumps or bumps in his body? Tender points? 4.Breast milk jaundice . In the next few minutes I will be asking you some questions to help me figure out the condition. I am the physician in charge today. I am glad you came here today to figure out 2.OB-GYN 5. and it is usually physiologic jaundice that does not lead to mental retardation Complications of Rh incompatibility: (1) kernicterus (brain damage  seizures).Disease called cystic fibrosis Diagnostic workup: When to suspect pathological jaundice? Treatment - Hemolytic workup: CBC / blood gp (mother and baby) / peripheral blood smear / Coomb’s test / bilirubin (direct and indirect) Septic workup: CBC / differential / blood & urine cultures / TORCH screen TSH and G6PD screening Liver enzymes / bilirubin / and coagulation profile If in the first day (or early second day) of life Bilirubin rises > 85 µmol/L/day Bilirubin level > 220 µmol/L before 4 days of age Conjugated (direct) bilirubin > 35 µmol/L Persistent jaundice lasting beyond 1-2 weeks of age Ensure proper hydration and feeding If sepsis: treat the underlying infection Phototherapy: if total bilirubin is > 300 µmol/L. I first need to ask you some questions. This is a reasonable concern. (2) Hydrops fetalis (generalized edema).Liver disease . What made you think about that? 3.doc Page 183 of 255 . (3) Hypoglycemia OSCE-guide-III. examine him and we may need to do some blood tests.BINDE Birth: Nutritional history: .Breast: o How many times do you feed him? o Do you use 1 breast or both of them? How long each? o After feeding him. and only for unconjugated hyperbilirubinemia. do you feel your breast engorged? .Any other children? Did any of them develop jaundice after birth before? 6.PMH?! 7.FH: .Formula: o Any reason to choose formula feeding? o Which type of formula? Do you know how to prepare it? Environment: . However I would like to inform you that jaundice in newborns is a common condition.How do you feed him? Breast milk? Formula? .Jaundice . it is contraindicated in direct hyperbilirubinemia Exchange transfusion: if total bilirubin is > 400 µmol/L The mother has a concern: will he develop mental retardation? 1.Blood disease . In order to be able to answer this. so. but first tell me. (cocaine during pregnancy  microcephaly. And I would like to ask you some questions regarding your child health. ─ Extreme of age.OB-GYN IUGR A newborn 3 hours old with IUGR. counsel the mother Introduction News BINDE Obstetrical history Mother PMH Good morning … I understand that you just gave birth.How do feel right now? . and others are related to the baby or the environment. IUGR.Have you seen the baby? . On the other hand. it is important NOT to smoke / drink alcohol / use recreational drugs during your pregnancy Possible causes: ─ Smoking / alcohol / cocaine during pregnancy.doc Page 184 of 255 . I would like to ask some questions about your pregnancy! - GTPAL Were you pregnant before? How many times? Any abortions? Miscarriages? Any history of chemo therapy or exposure to radiation Any family history with congenital anomalies + The mother has a concern: the baby is green  This means he passed meconium (baby stool) which means the baby had a stressful delivery + The mother has a concern: is it my mistake?  This condition is related to multiple factors. there are NO safe limits for smoking / drinking alcohol during pregnancy. in the future. some factors are related to pregnancy. MR) ─ TORCH infection. if you decide to become pregnant. . esp. advanced age pregnancy Risks for the next 48 hours: asphyxia / hypoglycemia OSCE-guide-III.Did you pick a name? Your baby has just been diagnosed with a condition called “intra-uterine growth retardation” or “low birth weight” … For that reason. my colleagues are taking care of your baby. doc Page 185 of 255 . without another explanation  reassure ─ Child neglect ─ Feeding problems: overfeeding / hungry Scan for risk factors for child abuse Nutritional ─ How do you feed him? Breast milk? Formula? ─ What about his weight? Environmental ─ With whom do you live? How is the relation? ─ How do you support yourself financially? Do you get enough support? ─ Any other kids? Any repeated visits to the ER? ─ Mental problem ─ Parent SAD ─ ─ Impact Red flags / R/O infection DD BINDE FH Investigations (not including those for suspicious child abuse): CBC / urinalysis / stool analysis OSCE-guide-III. what do you do? o Did you try to hug / hold / burp / sooth / play music / give him a walk? o Did you try to rock him? Shake him? What happened to him? ─ When he cries.OB-GYN Crying Baby Introduction CC Analysis of CC OCD / all the time / day and night? Is he crying > 3 hrs/day for > 3 days/week for > 3 weeks ─ What initiates or increases the crying? o Any chance the baby is hungry? What do you feed him? o Any chance that he is too hot / too cold? Do you adjust the temperature? o Any chance that he is wet? How often do you change his diapers daily? Is there any skin or diapers rash? ─ What improves or decreases the crying? When he cries. between the age of 3 weeks and 3 months. does he pull his legs? Is he passing gases? Is his abdomen distended? Is it related to feeding? How are you coping with this? ─ How does this affect your life? And your partner life? Are you able to go to work? ─ Is he drowsy? Floppy? ─ Did you notice if your child has fever or skin rash? Cough / wheezes? Discharge from his ears? Runny nose? Foul smelling urine? Abdominal distension? Diarrhea? ─ Any infection (there will be other symptoms)  review of systems ─ Infantile colics (crying > 3 hrs/day for > 3 days/week for > 3 weeks). wheezes. nausea / vomiting  Anti-biotic history!  Does he cough to the extent of vomiting or LOC  Did you renew it? From the same doctor? Was  Pertussis vaccination? he examined? Any xrays were done? How did this affect his life? Daily activity? Constitutional symptoms Triggers of Asthma: any thing that ↑ this cough?  Chronic diarrhea  cystic fibrosis  Any allergy Brief Other allergic diseases: atopic dermatitis / allergic rhinitis Allergic diseases: asthma / skin allergies   Differential diagnosis BINDE PMH FH Triggers Infection  Medications   Outdoor Indoor Stress OSCE-guide-III. noisy breathing. we have two issues:  The productive cough & fever 6 weeks ago  pneumonia  The intermittent / dry cough that presented after CC HPI Analysis of the CC Cough Impact Red flags Cough  Os Cf D /+/ COCA + B + Phlegm  Certain time of the day? Night? Acute phase Chronic phase Continuous / productive / Intermittent / dry cough / on and off / fever / loss of appetite no fever  Seen by a doctor? What  SOB. father wants to renew antibiotics Here.doc            Recent chest infection? Flu-like symptoms? Fever / chills? How do you use puffers? Stored properly? Not expired? Did you start new medication? β-blockers? Aspirin? Any recent ↑ in dose of these medications? Exercise Cold air Pollens (is it seasonal?) Dust: construction / smug (smoke/ fog/ exhaust) Do you smoke? Anybody around you? Do you have pets? People around you? Fabrics related: carpets floor? Any change in linen? Pillows? Blankets? Mattress? Curtains? Relation to any type of food? + Perfumes Do you live in a house (basement  mold)? Any construction renovation? Exposure to chemicals? Any new stressful situations? Page 186 of 255 .OB-GYN Chronic Cough – Asthma Child complains of cough for 6 weeks. diagnosis? Treatment? chest tightness. post-pneumonia. ─ When the child becomes older than 6 years. Case: 9 weeks history of cough on Amoxil for 2 weeks ─ DD: asthma / bronchitis / cystic fibrosis / recurrent pneumonia ─ Investigations: CXR / CBC. It is a term used to describe asthma-like symptoms in infants (< 6 years old) that may later be confirmed to be asthma when they become old enough to participate in asthma tests (spirometry and bronchodilators). ─ It may be self limited. ─ If this condition happens in adults.OB-GYN Questions: Diagnosis: Investigations: Treatment: hyper-reactive airways disease x-ray steroids puffer for 4 weeks Counselling: ─ The most likely explanation for that is a condition called: hyper-reactive airways disease. we need to start treatment with puffer (steroids puffer for 4 weeks). we treat with puffer for 4 weeks. and is usually triggered by infection (acute bronchitis or pneumonia). it may last up to 10 weeks after infection. ─ This is a common problem.doc Page 187 of 255 . however. we send the child for investigations (spirometry and bronchodilators) to confirm the diagnosis of bronchial asthma. lytes / sweat chloride test OSCE-guide-III. differential. and if the condition is still persistent for more than 10 weeks. if no improvement. we send to investigate for asthma (spirometry and bronchodilators then metacholine challenge test). Does he cry? Is it high pitched cry? .Constitutional symptoms! infection .Impact .OB-GYN Anemia 6-9 months. what about your partner? Are you related by blood to your partner? Investigations: lab works. CBC / differential / lytes / serum iron studies (ferritin.Is he active / playful like before? What can he do? Is he crawling? anemia .Os Cf D . that is why I need to ask you about your background. TIBC) / hemoglobin electrophoresis / KFTs / INR / PTT Treatment: iron supplement OSCE-guide-III.Bleeding disorders: nose / gums / coughing / vomiting / bruises  Iron def.Did you notice if your child has fever or skin rash? Cough / wheezes? Ear pulling or discharge? Runny nose? Foul smelling urine? Abdominal distension? Diarrhea? 4. if old place.Is he gaining weight? 3.Clarify CC: What do you mean he is pale? Is he yellow? .If he is doing activity. have you ever seen him eating the paint scales? Any heart / lung / kidney / liver disease? Hospitalizations / surgeries / illnesses (cancer) / infections Medications (Sulpha drugs – G6PD deficiency) / allergies Travel Family history of similar disease in the family Any bleeding disorder Any repeated surgeries? (cholecystectomy / splenectomy) Ethnicity: some blood diseases are more common in certain parts of the world.BINDE   - 6.Did you notice his suckling is weaker than before? Signs of . did you notice any SOB? Fainting? . does he complain of tender  Bleeding disorders points in his body] / cough / repeated infection  Chronic diseases Lead intoxication Leukemia 5.Family history - - Scan for risk factors for child abuse / neglect N: What are you feeding him? Breast milk? From the beginning? Do you give him any iron supplements or iron fortified cereals? B: was he term or not? E: with whom do you live? How do you support yourself financially?  offer social support Where do you live.Who noticed it? You or someone else? Is there any chance that he had this pallor before and you were not aware of it? 2.  Hemolytic disorders does he limp? if you carry him.doc Page 188 of 255 . anemia on body / blood in urine / stools / joint swelling  Thalassemia Leukemia: Constitutional symptoms / Bone pain [if he walks.Red flags: rule out . mother complains he is pale? 1.Rule out child neglect .Is he drowsy? Floppy? .Past medical history - 7.Analyze the CC .Diff diagnosis: . do you fell he is still hungry? General condition after vomiting ↑↓ Factors: is it related to position? When lying down? Other GIT symptoms: diarrhea Is he drowsy? Floppy? Does he cry? Is it high pitched cry? Did you notice his suckling is weaker than before? Dehydration: do you feel his lips dry / skin dry? Does he tear? How many diapers Failure to thrive: what about his weight.Pyloric stenosis - - Any infection / meningitis Brain tumour - - GERD Wrong formula OR not preparing it well Overfeeding OR NOT burping - - OSCE-guide-III.Impact - 3.Analyze the CC 2. did your baby have any fever.OB-GYN Vomiting The mother of (6 weeks – 3 months) old baby came to the clinic complaining of child’s repeated vomiting. Introduction Chief complaint 1. do you know his weight? What was his weight at birth? Do you have access to his growth charts? Constitutional symptoms! R/O infection: Did you notice if your child has fever or skin rash? Cough / wheezes? Ear pulling or discharge? Runny nose? Foul smelling urine? Abdominal distension? Diarrhea? Differential diagnosis of 6 weeks vomiting Rule out child neglect . flu-like symptoms? Is it continuous or on and off? How often? Day and night? COCA + BLOOD: what do you feed him? Is he vomiting the entire amount? Is it watery or curdy? Character: is the vomiting forcible? Projectile? Timing: is it related to feeding / meals? After vomiting.doc BINDE screening Mother attitude! Family history of pyloric stenosis Appear at age of 2 – 4 weeks Projectile / non-bilious / baby still hungry after feeds Continuous Other symptoms / neurological: weakness / neck stiffness / seizures After feeds Not all feeds No ↓ in weight Page 189 of 255 .Red flags - 4.DD When did it start? How did it start? Sudden or gradual? At that time. Full blood work / CBC / albumin level .How do you feel being a new mom? How do you feel about your baby? .Weight analysis! Environment: .SAD parents (smoking / alcohol / drug use) .If dehydrated: admission .How is your mood? You look down for me. lytes.Fundoscopy . ABG) / US .How do you support yourself financially? .BINDE - Scan for risk factors for child abuse / neglect Nutritional history: .How do you feed him? Breast milk? Formula? Which type? Do you know how to prepare it properly? Any solid food? .OB-GYN 5.Pregnancy not planned .Difficult child .PMH 7.Do you get enough support? 6.FH Pyloric stenosis DD: duodenal atresia / tracheo-esophageal fistula Management plan: . the vomiting or switching to the new formula? .Baby who needed special attention after delivery .Which happened first.doc CONTACT CAS Page 190 of 255 .Investigations: lab works (CBC.Young couple .Parents with history of abuse .Skeletal survey OSCE-guide-III.With whom do you live? How is the relation? .Stress or financial difficulties in the family Investigations for child neglect .Preterm baby .If suspicious child neglect: contact CAS Potential risk factors for child abuse: .Separation from the child .Congenital anomalies . any chance you are being depressed? Have you ever been depressed before? . Differential DD for ACUTE diarrhea: diagnosis .Poor appetite .Os Cf D CC .Use of antibiotics .Dehydration: do you feel his lips / skin dry? Does he tear? How many diapers .Toddler’s diarrhea: does he drink too much juice daily? .Constitutional symptoms! (R/O infection) .Any body else at home with diarrhea? .OB-GYN Diarrhea Diarrhea Failure to thrive – FTT  What about his/her appetite?  What other associated symptoms? (Respiratory / Gluten) Cystic fibrosis Celiac disease Milk protein HIV allergy From cow milk .Lactose intolerance: .Did you notice his suckling is weaker than before? .Infectious: .Does he go to day care? DD for CHRONIC diarrhea without failure to thrive: .doc Page 191 of 255 .Does he pass a lot gas? . do you know his weight? What was his weight at birth? Do you have access to his growth charts? .Impact .Difficult to wipe? .Any undigested food .Did you notice if your child has fever or skin rash? Cough / wheezes? Ear pulling or discharge? Runny nose? Foul smelling urine? Abdominal distension? Diarrhea? 4.↑↓ Factors: Juice (Excess fruit juice) .Infectious – parasitic / traveller’s diarrhea .Failure to thrive: what about his weight.APPETITE 2.Watery / loose / bulky .Red flags: .Analyze the .Identify FTT – weight: What is weight today? At birth? Last visit? The highest weight? Not gaining weight? .Gluten Should not be given < 1 year - NO FTT Toddler’s diarrhea Infections Lactase Deficiency (lactose intolerance) A 50 years old father comes with 9 months child with 6 weeks of diarrhea (CHRONIC) 1.Other GIT symptoms: vomiting .Does he have any redness / skin rash at his buttocks? OSCE-guide-III.Does he cry? Is it high pitched cry? .Long period  malabsorption  anemia and rickets 3.Camping / travelling .Good appetite .Is he drowsy? Floppy? .COCA + BLOOD + others: .Respiratory . HIV (if susceptible): was he screened for HIV .Did you start to give him solid food? What type of food? How about bread.BINDE DD for CHRONIC diarrhea WITH failure to thrive: .Scan for risk factors for child abuse / neglect .Cystic fibrosis . and you are speaking with one of the new parents: o Are you the biological mother/father? o Is this adoption or foster home? o When was the child adopted? At which age? From where? o What were the circumstances? o Do you have information about the biological parents? o Was he screened for HIV? OSCE-guide-III.OB-GYN 5.Does he have flat buttocks? Thin legs and arms? Distended abdomen? . It might be the adopting father. biscuits? Which started first. does he have abdominal distension? Gases? Pulling his legs? .Poor appetite .What is your differential diagnosis: o Cystic fibrosis o Celiac disease .If the child was adopted.doc Page 192 of 255 . do you tell her or no? o In order to determine whether I should release any information or no.Does he have any bulging through formula? Was it changed his buttocks? recently? .What do you feed your son? How .Do you know how to prepare the . I would like first to know who has the legal custody (guardian) of this child. cereals.Nutritional history .After you feed him. the diarrhea or the new food? .If the biological mother called.Repeated chest infections? Cough? .Weight analysis 6.Celiac disease Cystic fibrosis Celiac disease .PMH 7. a social worker (case manager) … Notes: .Milk protein allergy: cow milk given before 1 year .bulky foul-smelling stools .Did he have yellow discoloration many times? after birth? For how long? .When did he poo the first time? . want to know about her son.FH Questions: .Good appetite . Weight today. CF.Who is the primary care giver? For how long have you been with him? .Not gaining weight .Differential diagnosis - 5.Impact - 3.FH - Is he drowsy? Floppy? Is he playful? Active like before? Any limitations? Does he turn blue with activity? Constitutional symptoms! Any congenital or long term disease? Review of systems: cardiac/ chest/ GIT/ urinary/ MSK/ skin/ allergy/ pale/ bleeding Rule out child neglect Difficulty swallowing (CP.Most common cause is inadequate intake OSCE-guide-III.Congenital: everything is small / short.Height today.PMH 7.Weight < 3rd percentile or falls across 2 majors percentiles .OB-GYN Mother worried about her child weight .Analyze the CC Weight analysis: .Failure to thrive (FTT): weight decreases first then height will be affected later . last visit 2. last visit.doc Page 193 of 255 . birth. birth. I need to contact his family physician to take a look at his charts Height analysis: .Endocrine causes: fat and short .Rule out any serious condition: chronic inf/ dis / malignancy 4.When did you start to worry about that? Why? . thin with small head Failure to thrive .Who noticed it? . highest .Do you have his growth chart? If no: if you do not mind. pees a lot. tired) Chromosomal abnormalities / inborn error of metabolism Scan for risk factors of potential child abuse / neglect Apgar score at birth Diet in details: breast feeding/ formula/ cow milk? Any reason? For how long? Any supplements? Any solid food? Developmental milestones Environment: with whom do you live? Who takes care of the baby? Is he/she capable of doing this? SAD during pregnancy and now Under weight: .Not eating well 1. Cleft Palate) Chronic loss: chronic Diarrhea (Celiac disease. pancreatic insufficiency) / chronic vomiting (pyloric stenosis in a younger child) Diabetes mellitus (drinks too much water.BINDE - 6. allergic skin reaction? Family history of allergy? Asthma. dinner and snacks o Review of systems  will be negative ─ ─ ─ ─ ─ Plan. ─ Is it first time to eat peanuts? Any similar reaction before? Any known food allergy? ─ Review of systems  will be negative ─ ─ ─ Past history of asthma. TIBC) Differential diagnosis: o Stresses int the family o Child abuse / neglect o Failure to thrive Case: A 6 years old developed severe allergy to peanut.OB-GYN Case: A 2 years old boy does not want to eat. it is full of candy and a coke. allergic rhinitis.doc Page 194 of 255 . The father carries a bag! ─ History: o When you ask about the bag. allergic rhinitis. counsel the father. to improve the health and diet education of the father: o Educate about nutritious food and supplementation o Give brochures and Canada dietary guidelines o Refer to dietician specialist or nurse Recommend supplementation: o Iron o Multi-vitamins Examine the baby Blood works: CBC / lytes / serum iron studies (ferritin. child is now stabilized. he says it is for the boy lunch. allergic skin reaction? Other siblings with allergic reactions? Management: ─ Will send the boy for allergic testing ─ Strict avoidance of allergens ─ Epi-pen OSCE-guide-III. lunch. o Details about breakfast. Any flu at that time? .Do you measure it? How many times daily? How do you measure it? .Chest: cough / phlegm / SOB / wheezes . pale stools) / urinary (urine changes.Did you notice any skin rash?  OCD / distribution / color / do you feel it elevated?  Are his shots up-to-date? . distension.Other persons at home with the same symptoms? .doc Page 195 of 255 .Analyze the CC 2.Is it the first time? .Differential diagnosis: Review of systems FEVER .Trunk  vesiculo-papular  chickenpox . crying while peeing.OB-GYN Fever Introduction CC 1.Did you notice his suckling is weaker than before? .Red flags SKIN RASH 4.Joints: pain / swelling / mouth ulcers .Scan for risk factors for child abuse / neglect 5.Does he cry? Is it high pitched cry? .The fever and constitutional symptoms are already analyzed . dark urine.Impact 3.Any diurnal variation? More at morning or night? .Buttocks / abdomen  henoch schonlein purpura / Investigations: urinalysis Treatment: steroids .Any special pattern? More every 2nd or 3rd day? . diarrhea) / liver (yellow color. loin pain) .Review of systems: DD .Did you try to give any medications to help? Did it help? .Is he drowsy? Floppy? .Os Cf D .Other constitutional symptoms .BINDE 6.Headache / drowsy / neck pain / rigidity / nausea / vomiting? Does he recognize you? Talk to you? .Abdomen (pain.Past medical history 7.Cheek: fifth disease .Is he tired? .ENT .Face: measles / rubella .Family history OSCE-guide-III. itching. To BOTH the mother and the child .FH Physical exam OSCE-guide-III.Impact .doc - Allergic rhinitis: runny nose related to seasons.COCA . no fever Viral flu: respiratory symptoms / joints & muscles ache Viral common cold Scan for the risk factors of potential abuse Immunization School performance Any congenital or long term disease? Other members in the family with symptoms? School contacts? Mouth ENT LNs Chest exam Page 196 of 255 .BINDE 6.OB-GYN Runny Nose / Flu / URTI 8-15 years child is coming to see you with his mom.Is this the first time? Or did it happen before? 2.Skin rash 4.What ↑ or ↓ .Constitutional symptoms Review of systems: .Rule out infection: Any recent flu-like symptoms? Do you feel tired/ fatigue? History of sinusitis / Pain in your face? Any sneezing? Red eyes? Pain/discharge in ears? Any sore throat/ oral ulcers/ tooth pain? Pale / bleeding .Red flags . recurrent.During the encounter. distribute the questions and interaction between both the mother and the child 1. c/o: runny nose / flu / URTI? Introduction .Os Cf D .Cardiac / chest / GIT / urinary / MSK / allergy .Analyze the CC .DD 5.PMH 7.R/O meningitis: Neck stiffness / pain? Headache? N/V? .Is he playful? Active like before? Any limitations? 3. malaise Measles Appearance: erythematous maculo-papular rash. rash 3 days after start of symptoms Distribution: starts at hairline. can affect trunk Roseola Appearance: pink maculo-papular rash (faint). mucosa. diagnosis by liver biopsy  Associated with aspirin ingestion by children with varicella or influenza infection. no fever (infectivity: 4 days prerash) Infectivity: 7 days pre-rash to 5 days postrash Infectivity: 1-2 days pre-rash until vesicles have crusted over Infectivity: prior to onset of rash Reye Syndrome:  Acute hepatic encephalopathy and non-inflammatory fatty infiltration of liver and kidney  Mitochondrial injury of unknown etiology results in reduction of hepatic mitochondrial enzymes. Timing: 14-21 day incubation. extremities.  1-3 days prodrome: (fever and respiratory symptoms).OB-GYN Rash Clinical Presentation Fever. rash 1-5 days after start of symptoms. myalgia. parotitis (bilateral. headache. trunk. Koplik spots Timing: 10-14 days incubation. Timing: 5-15 days incubation. rash 3-5 days after symptoms. Erythema Appearance: uniform. Chickenpox Appearance: macules  papules  vesicles  crusting. Distribution: starts on face spreading to neck and trunk.  40% mortality OSCE-guide-III. rash 10-17 days after symptoms (fifth disease) Distribution: bilateral cheeks with circum-oral sparing. spreading downwards.  Then rash Distribution: face.doc Page 197 of 255 . palms and soles typically not involved Rubella Appearance: pink. erythematous maculo-papular rash Infectiosum Timing: 4-14 days incubation. all stages apparent (varicella) at once (polymorphous rash)  very pruritic Timing:  10-21 days incubation. maculo-papular rash. pushes earlobes up and out). Distribution: starts at neck and trunk spreading to face and extremities Management: rest / anti-pyretics / fluids / good nutrition Mumps When does the kid go back to school? Infectivity: 7 days pre-parotitis to 7 days post-parotitis No rash. palms and soles. Is he aggressive? Does he play with other kids? .How do you describe his hearing? Does he have hearing difficulties? .Did he take any medications? Any antibiotics (aminoglycosides)? .Screen for neglect: how many hours you spend with him? Is he a difficult child? . when he was born? AUTISM: .Any family history of autism? BINDE: .Was he ever screened for hearing test. would he respond and reply? What if you are behind him? What if you are in another room? .Did you have skin rash during pregnancy? TORCH infection? SAD during preg? .Is the child able to speak at all? How many words is your child capable of using? When did he start to say it? Can he use many words in one sentence? .doc Page 198 of 255 .If you call him.Was he able to use more words (talk better) and lost them? .Was it complicated labour? Apgar score? .Did he get repeated ear infections? Fluids in the ears? Discharge? .Does he maintain eye contact? Does he show emotions? .Does he have a favourite toy? How does he play with it? (train / spinning wheels) .Pediatrics Delayed Speech Introduction Verbal assessment Rule out any serious condition BINDE PMH FH Clarification: is it not gaining words.What can he do? When did he start to sit? Crawl? Stand? Walk? Climb stairs? . or head banging? .Does he control his urine / bowel movements? Environment: .When did you start to have concerns? Did you seek medical attention before? . losing words OR not speaking at all? - Hearing loss Autism Anatomical: tie tongue / cleft palate .Start with the development: to rule out MR Developmental (mile stones): .Does he do repeated movements like rocking.Family history of deafness or hearing loss VERBAL ASSESSMENT .History of meningitis / jaundice .Did you notice that he keep increasing the volume of the TV? .Did he have any special features? Congenital malformations? Cleft palate? OSCE-guide-III.Family factor: how many languages do parents and other family speak at home? Pregnancy / Birth: . did strangers make him nervous? .How can he communicate with you? What does he do if he wants something? I would like to ask you some questions in order to reach to the cause of this condition: HEARING: .Would you please tell me more about that! .As a child. the most likely explanation of your child seizures is a medical condition we call “benign febrile seizure” ─ What do you know about “febrile seizures”? Do you want me to clarify some information about it? In details? Febrile seizures: ─ This condition usually affects kids 6 months to 6 years. to seek medical attention and to decrease the fever ASAP using Tylenol or cold foments. ─ Any other questions or concerns. Then find the source of fever and treat. typical attack is less than 15 minutes. ─ We do not know exactly the reason for it. OSCE-guide-III.doc Page 199 of 255 . for each 100 child who got 1 febrile seizure attack: o 65 children will not have it again o 30 children will have another attack o 3 children will have many other attacks even without fever o 2 children will develop seizure disorder ○ The best treatment for it is the prevention that is why it is important to make sure that whenever he gets a fever. ─ Usually it is self-limited. and will not recur in 24 hours. it is not uncommon. ○ In case it happens again: ○ Turn the child on his side / protect him from hitting any nearby object / do not force objects into his mouth ○ Bring to ER if seizure does not stop within 15 minutes ○ Diazepam 5 mg PR suppository ○ If repeated attacks. we may consider prophylactic anti-convulsion therapy ○ Will do CT. and a lot of children (around 3%) might have attacks. benign. Most children will outgrow their condition after the age of 6 years. and can not tolerate high fevers.Pediatrics Seizing child counselling Refer to the seizing child phone call case in the emergency medicine section for analysis of the event History Analyze the event Fever Rule out BINDE PMH FH Counselling During / before / after Analyze the fever Rule out meningitis / pneumonia Febrile seizure / epilepsy Introduction Febrile seizures Introduction: ─ Based on what you have told me. ─ Another attack(s): ○ From the studies we know it might happen again. EEG ─ I will give you some brochures and web sites in case you want more information. but it is related to fever and may be because the children brain is not fully developed at that age. without problems with the law. paediatrician.Actually yes.Specific learning disability PMH . this is crucial for children. let me first ask you some questions to see if your child meets the criteria of ADHD or any other developmental challenge: Diagnosis (hyperactive / inattentive / impulsive): . it helps them to focus as increases their concentration and channels their energy. Introduction To diagnose ADHD: .Seizures (petit-mal epilepsy) FH  ADHD / MR / autism / depression . it has its side effects.Can he focus on one subject for > 30 minutes? Can he finish his tasks (e. However.Depression Conclusion Introduction: . it is the same family.ODD /+/ Conduct disorder BINDE .> 6 months duration  Diagnosis (symptoms of ADHD) . but to make a diagnosis a psychiatrist. school performance.How about before? Did anyone mention that or no? IMPACT: . that include: insomnia (that is why we give it early).Impact on functioning. Counsel for 10 minutes.How much time do you spend with him? How about the mother. is she involved? .< 7 years old child  Impact Differential diagnosis:  Differential diagnosis . Before talking further about ADHD and Ritalin. it is called “methylphenidate” and it is approved for this indication. It is not addictive.g.In children. so that they can have career and live independently in the future. Even though we might not be able to cure all children with ADHD. . and it is effective and has been used for long time.Did you notice that yourself? . like any other medication. by we try to help them with education.Did the teachers complain that your child is full of energy? Spinning all the time? Refuse to stand still? Talk all the time? Answers even if he is not asked? Does he stand in-line or does he break the queues? .Who diagnosed it? Usually teachers recognize it first (pick it). or a specialized nurse assessment is needed Concern – do you give Ritalin (which is amphetamine) to children? . and not all children improve on it. it is the first line of treatment for ADHD. . . A lot of children use Ritalin.Pediatrics ADHD counselling The father comes to you saying that his son was diagnosed with ADHD two days ago and he has concerns about ADHD and Ritalin.It is generally a safe medication in children. and we can stop it at any time.doc Page 200 of 255 . the homework)? Does he jump from one activity to another without finishing it? Does he lose his stuff? Does he forget his belongings? .It is not exactly amphetamine.Is this only at school or also at home? . as it allows them to do better in schools. relationship with peers OSCE-guide-III.2 settings (school / home) Address concerns . abdominal pain. doc Page 201 of 255 . and there is medical treatment for it.Based on what you have told me. Notes: . He has symptoms: . And in his age group. his symptoms and behavioural changes could be related to the loss of his mother.Counsel on Ritalin.On the other hand. it must be difficult for children in his age to go through all that.Pediatrics Differential diagnosis: ODD Conduct disorder Learning disability Petit-mal epilepsy Depression Autism MR - Does he like not to follow the instructions? Does he like to challenge his teachers and other family members? Is he aggressive? Does he fight a lot with other children? Does he have a pet? How does he treat his pet / or other pets? Did you notice that he takes others’ belongings without telling them? Does he tell the truth all the time? Does he like to set fires? Does he like to go to school? Does he have specific difficulty in reading / writing / mathematics? Does he have a history of seizures? LOC? Abnormal movements? Was he stressed recently? Any loss of a beloved one? Is he sad? Crying? Nightmares? Losing weight? BINDE: detailed developmental history Conclusion: . . . How is he/she coping with that? How are you coping? OSCE-guide-III. in which the first line is Ritalin.We can refer your child to a child psychiatrist who can help him deal with that. However. it must be difficult for children in his age to go through all that. this is not uncommon condition. How is he/she coping with that? He has NO symptoms OR does not fulfill the criteria: . . depression may manifest as behavioural changes like what he is experiencing right now.Based on what you have told me.I am really sorry for this loss. I am sorry to hear that. your child symptoms meet the criteria for diagnosis with ADHD.Whenever you hear that one of the parents has passed away  show empathy. which might lead to depression. it is less likely that he has ADHD. and the fact that it happened > 8 years old. for how long have you been here? How do you feel being here? Identify the language barrier . .Pediatrics Vaccination counselling New comer to Canada. welcome to Canada.doc Page 202 of 255 .What gave you this feeling? Concerns? .I can assure you that there is no connection between vaccines and autism.Before we proceed.Pose frequently and ask if she has any questions . the explanation of that was a bias in the selection by the author and the study was conducted to favour this outcome.When we tried to figure out why the first study found the connection. in large number of countries. But because the world is a becoming more and more a small village. OSCE-guide-III. the study found there is a connection between autism and 1 type of vaccines.And because we take vaccines very seriously.Now. Any questions till now? Why to vaccine against diseases not preset in Canada? . am I clear. a much larger study was done. and people travel easily from a place to another.Vaccination Candidacy for vaccination Mother vaccination Otherwise. namely the MMR. can you tell me more about your concerns? . o Diphtheria is also coming back because of lack of vaccinations. and you came to the clinic because you have some concerns about vaccinations. And the origin of this misinformation is a study done in England many years ago. speak neutrally ○ What are vaccines? ○ How do we vaccine? ○ Side effects of vaccines Introduction / welcome her / how do you feel? .Pap smear . Vaccines cause autism! . and some children ended up losing their lives. The only relation is a coincidence between the age in which parents start to notice autism symptoms and the age we start to give MMR. or do you have specific concerns? o I heard that vaccines cause autism! o I think we do not have these diseases in Canada.Deal with concerns one by one . and this is because we have a successful vaccination program. comes to you as she has some concerns about vaccinations Introduction / welcome her / how do you feel? Speak with enthusiasm (to encourage) with three Identify the language barrier counselling sessions: Identify concerns . Now we found for sure that there is no connection between vaccines and autism. why should we give the vaccines for diseases not common here? Thanks for coming here to discuss your concerns with me. Identify concerns . we do not want your child to be vulnerable to preventable diseases if there is any outbreak anywhere.Breast feeding . they started to have measles infections again. including very large number of children. And I will give you some examples: o In England.There is misinformation among the general population that there is a connection between vaccines and autism. . We will discuss all you concerns. or do I need to talk slower? We can arrange for an interpreter or a family member. Another theory to explain the connection was the preservative used in the vaccine (Thiomersal) and it contains mercury.Do you need general information. even in many European and developed countries. First of all.Good evening Mrs …vich. people stopped to vaccinate their children for MMR. if you would like to. my name is Dr … I understand that you are a new comer to Canada. .Even though we do not have these diseases commonly in Canada. However Canadian vaccines do not contain it. and some children become floppy. the injury is vulnerable to be contaminated with tetanus. if a child is injured.We have large number of diseases and infections. we can set up a follow-up meeting to discuss in details your vaccination status and find what vaccination(s) you might need to take. OSCE-guide-III. . As children are vulnerable to be infected with many infections.Vaccines are generally safe. which has serious fatal consequences. It helped to save the lives of large number of children all over the world. we wait.You do not need to worry about memorizing them. this is rare. . fever.Was he diagnosed with any neurological diseases? . sickness. redness and swelling. .doc Page 203 of 255 . flu-like symptoms? If high fever. these anti-bodies will protect him. it is in the soil everywhere. pertussis and hemophilus influenza B. millions and millions of children were and are vaccinated and it saved the lives of large number of them. tetanus.As a child. . and each one needs more than one shot.Any questions or concerns do you have? Side effects of vaccines . very rarely.Does he have any allergy? Based on what you told me. caused by large number of bugs. and that is why we keep the children under medical supervision for 20 minutes after vaccination. at the site of injection. it may cause fever. I would like to make sure that your child is a good candidate for vaccination. We try to protect against many of it.Pediatrics Tetanus is universal. Do you have any questions or any more concerns? o - Candidacy for vaccination / history – BINDE: Before I proceed further. we try to decrease the number of shots to be given. Did he get recurrent infections before? (if not very young) . again. 4. Later in life. Any other ideas or concerns you need to talk about? Thanks for coming and see you next visit for the first vaccination shot.Does your child have any illness. it protects against diphtheria.Vaccines are one of the most significant preventive interventions in medicine. children experience a serious allergic reaction.For example. And these vaccines are covered by OHIP. . 6. . even thought it might affect adults. . Usually the body reacts by forming “anti-bodies” which are protective chemicals against these bugs. I will give you a schedule of the required immunizations to follow with his family doctor. like any other medical intervention. I hope that I answered you concerns. vaccines have some side effects.Lastly. were you vaccinated? How do you feel about that? .There are some few rare side effects that include prolonged crying. What are vaccines? . like pain. 12 and 18 months and another booster dose at around 5 years. there is a shot called “pentacel” that stimulates the body to produce protective antibodies against 5 diseases. then we inject it into our bodies by a needle. . and we either kill it or we make it so weak so it will not cause any harm. but usually with no serious consequences as compared to children! . It is to be given at 2.If it is ok with you. you need to seek medical attention immediately. polio. .Is it ok till now? Mother vaccination: . and now you have a better idea about vaccinations. and we usually start vaccination at 8 weeks.It may cause minor issues.However. seizure.Was he scanned for HIV? Were you scanned for HIV? Is there any risk that you might have HIV? . We take the different bugs (bacteria and viruses). some of them cause serious illnesses.Because we have large number of diseases.Was I clear? Any questions? How do we vaccine? . your child is a candidate for vaccination. but if this happens. which we deal with Tylenol for pediatrics. . if the child will get exposed to the real bugs. tachycardia.Symptoms of hypoglycemia may be categorized as neurogenic (adrenergic) or neuroglycopenic. major. difficulty with concentration. dehydration. coma and death OSCE-guide-III. shakiness. Sympathoadrenal activation symptoms include sweating. blurred vision.Hypoglycemia is a syndrome characterized by a reduction in either plasma glucose concentration or its tissue utilization to a level that may induce symptoms or signs such as altered mental status and/or sympathetic nervous system stimulation.Signs of acidosis may include shallow rapid breathing or air hunger (Kussmaul or sighing respiration).Pediatrics Child with DM counselling Father of 6 years old boy comes to you as he has some concerns about his diabetic son History - Counselling - Complications Management When was the child diagnosed? How? What type of insulin? Is DM controlled? Regular measure of blood glucose? Hb A1c! Symptoms of DM Symptoms of DKA Symptoms of complications: nephropathy / retinopathy / neuropathy (usually develops more than 5 years after diagnosis) Do you have special concerns? What do you know about diabetes? Did you read? Hear? Anything? Type 1 DM Insulin DKA /+/ Hypoglycemia /+/ Prevention of complications Macro and micro vascular complications Meal plan. hypotension. in extreme cases. psychological support Insulin injections with BS monitoring Children are more prone to brain damage with hypoglycaemia. Hypoglycemia: .DKA is a state of absolute or relative insulin deficiency aggravated by ensuing hyperglycemia. . but it is not uncommon in some patients with type 2 diabetes.Signs of dehydration include a weak and rapid pulse. and acidosis The most common causes are underlying infection. polyuria. and nocturia. and disturbance of consciousness. DKA mainly occurs in patients with type 1 diabetes. therefore high target range in younger and tight control in older kids Diabetic ketoacidosis (DKA): . life-threatening complication of diabetes. . and new onset of diabetes.doc Page 204 of 255 . confusion. Neuroglycopenic symptoms include weakness. and increased capillary refill time. . . polydipsia.Symptoms of hyperglycemia associated with diabetic ketoacidosis may include thirst. or dizziness. . education. exercise. and a sensation of hunger. disruption of insulin treatment. anxiety. . dry tongue and skin. tiredness. and.DKA is defined clinically as an acute state of severe uncontrolled diabetes associated with ketoacidosis that requires emergency treatment with insulin and intravenous fluids.Diabetic ketoacidosis (DKA) is an acute. inappropriate behavior (sometimes mistaken for inebriation). abdominal tenderness. DD - - 5.When did it start? How did it start? Sudden or gradual? .Home / school change . who else does live with them at home.Pediatrics Bed wetting counselling / Nocturnal Enuresis Introduction 1. any sisters or brothers? Kidney disease Kidney disease Bed wetting DM Seizure disorder Page 205 of 255 .doc - OS CF D: .New sibling .PMH 7.Diabetes insipidus (history of meningitis / brain infection / head trauma) .Impact 4.Is it continuous or on and off? How often? Day and night? Every day? Every night? ↑↓ Factors: stress / drinking too much fluids before bedtime How does Mom feel about it? How does the child feel about it (impact of this on child)? Constitutional symptoms! Did you notice if your child has fever or skin rash? Odd smell or colour of urine? Pulls his penis? Cries while peeing? Rule out child neglect / abuse Medical conditions - BINDE screening Parent attitude! DM (drinking too much water / going more often to pee / feeling tired / losing weight) .Seizure disorder Stressors .FH OSCE-guide-III.Rule out infection 2.School performance Very briefly because the child is more than 6 years old .Analyze the CC - 3. is he the only child.BINDE - 6.Scan for risk factors for child abuse / neglect How is his school performance? Who is the primary care giver.Frequency .Primary or secondary (dry period(s) of time)? .UTI (detailed in No 3) .Neurological: trauma or surgery to back / bowel dysfunction / leg weakness or numbness . .More common in boys than girls May be it is due to regression of his development because of the current stresses in his life Condition is usually self limiting & you need to give the child some time & he will adapt very well to the changes Try behavioural modification for 3 months: .Pediatrics Counselling - Management - - OSCE-guide-III. 5-10% of children have this problem .doc The condition is common.Document dry nights and use reward system. the only serious side effect is seizures due to water intoxication . it ↓ bladder contraction therefore it leads to ↑ bladder capacity Page 206 of 255 .Desmopressin (DDAVP): 1 hour before bedtime. .By the age of 4 years. 25% of children have this problem . Alarm therapy Pharmacological measures: . Be careful to water intake.Avoid drinking late before sleep.Try to wake him up to go to the washroom.Oxybutynin: at bed time.By the age of 7 years. . anti-convulsions) & allergies Social history: smoking. worm.B.doc Page 207 of 255 .Highly nutritional.Engorgement (pump) . sterile . antibiotics. however.Less allergic . alcohol. active T.Contains antibodies to help your child fight infections . a method of contraception   Problems with breast feeding that might happen to the mother: .Sore nipple (clean & apply soothing lotion) . .N. breast milk contains: more vitamin C. such nuclear medicine therapies require only a temporary interruption in breastfeeding . economic.The mother has been infected with HIV / is taking antiretroviral medications . emotional satisfaction for the mother and creates sense of security for the baby .Secures bonding between mom and baby. Do you have any concerns? I need to ask some questions ─ If still pregnant: ○ How the pregnancy is going so far? ○ Pregnancy questions of BINDE ○ Ante-natal care follow-up (from OB-GYN) ─ If already delivered: ○ Birth questions of BINDE PMH: medications (lithium.Help mom reduces weight.Ready. providing all elements baby needs (especially colostrum).Rule out contra-indications for breast feeding: . less protein load on the baby . a rare genetic metabolic disorder .An infant diagnosed with galactosemia. clean.The mother is using or is dependent upon an illicit drug .The mother has active herpes simplex on breast .Mastitis (treat & do not stop feeding) OSCE-guide-III.: Hepatitis B is NOT a contraindication to breastfeeding 2. drugs / work and home environments 1. how do you feel being a (prospective) mother? It is good to hear that you plan to breast feed.The mother has untreated.The mother is taking prescribed cancer chemotherapy agents. easily absorbable iron.Counselling: Advantages of breast feeding . such as antimetabolites that interfere with DNA replication and cell division OR is undergoing radiotherapy.B.Pediatrics Breast feeding counselling     A lot of enthusiasm Congratulation for the news. Do you have any questions or concerns? 4. I will give you a table with the recommended time and types to start solid food . vitamins and rest.Care of the breast: frequent cleaning with water and proper hygiene. redness.Advice: .Use pump & keep the milk in a bottle for 3 – 6 hours outside and 24 hours in a fridge.Wetting 5 – 6 diapers daily What do you do if you do not have enough milk? (You can supplement with formula)   How can you breast feed & work at the same time? .I will give you the immunization schedule so that you remember to bring him for followup and for vaccination .Mother can use OCP but it will reduce amount of milk OR use an IUD . warning signs: engorgement.doc Page 208 of 255 . you can keep it in the freezer OSCE-guide-III.On demand at the beginning . .Then the child will adapt to a schedule of every 3 – 4 hours   How long should the baby stay on each breast? (10 minutes)  How do you know that your baby is feeding well? .Avoid smoking & alcohol . tenderness.I will give brochures & information about BF classes .Frequently asked questions about breast feeding: How often you should feed your child? . fluids.Mother should get enough nutrition.Pediatrics 3. hotness .Sleeping well .Gaining weight .Avoid using any medication without asking your Doctor .Give supplementations of: o Vitamin D from day 1 o Iron from 4 – 6 months o Start solid food from 4 – 6 months. Pediatrics Psychiatry OSCE-guide-III.doc Page 209 of 255 . fluent. cooperative (not). (2) He has (good / poor) eye contact. (8) Judgement (good / poor). OSCE-guide-III. Psychomotor agitation Eye contact * Cooperative * Hostile * Abnormal movements / jerks / tics / lip smacking (anti-psychotics) Speech Volume (low / normal / loud) (process of Tone (parkinsonism: monotonous) talking) Articulation Fluent Pressured speech Mood How is your mood? How do you feel? Write it in patient “own words” Affect Quality (by pt observation) Elevated Euthymic Depressed Anxious Congruency Appropriateness Others Stable / range / labile / flat Perception Normal perception Illusions Hallucinations Thought Processing (how does the patient connect Goal directed ideas). (6) There are no delusions or hallucinations. and his appearance matches his chronological age. (4) His mood is … (5) His thoughts are organized (or disorganized).g. not slurred. tone. (7) There is no suicidal ideation or homicidal thoughts.doc Page 210 of 255 . with psychomotor … (retardation / agitation) (3) His speech is of normal volume.Pediatrics Mental status exam – the psychiatry interview ABS MAP TCIJ ALWAYS clarify! What do you mean by that? Appearance Dressed Dishevelled: dressed and groomed poorly Groomed Given age matches his chronological age Dressing matches the weather Behaviour Psychomotor retardation vs. e. and not pressured. insight (intact / lost). well groomed. how did you come here today? Thought block Flight of ideas Loose association Tangentiality Circumstantiality Contents Obsessions Delusions Suicidal / homicidal ideation Cognition Mini mental exam Insight Judgement Brief comment: (1) The patient is well dressed. if he resists  ↑ anxiety  take actions to try to ↓ anxiety (compulsions) .Auditory: o Usually schizophrenia o Do you hear voices / things that other people do not hear? When you are alone.If the patient is suicidal / homicidal / can not take care of himself  admit.doc Page 211 of 255 .Pediatrics Perception Hallucinations: . do you hear voices coming from your head? o How many voices o Are they familiar or not? o Are they talking to you or about you? What are they telling you? o Did they ever ask you to harm yourself or somebody else? What is preventing you from doing this? o How do you feel about these voices? . . if he/she refuses  form 1 (for involuntarily admission – for 3 days – for psychiatric assessment – by another physician). I have concerns about safety of the patient and other people.I want to file form 1 for the patient and call the hospital security to bring the patient back.Visual: o Usually organic (tumour / epilepsy / cocaine and amphetamine) o Brain tumour /+/ alcohol intoxication / DT /+/ cocaine / hallucinogens o Do you see objects / things that others do not see? o Can you describe what do you see? o Do they give you any messages? o Are these messages asking you to harm yourself or anyone else? . and he can not resist.Smell: usually epilepsy Though Processing: o How did you come here today? Content: + Obsessions: . OSCE-guide-III. Form 1 has to be filled within 1 week from seeing the patient. contamination / order / checking / … o Do you have any repeated thoughts or images that you find difficult to resist? About what? What do you do? + Suicidal / homicidal ideation: o Do you have any thoughts or ideas of harming yourself? o Or harming other people? o Any access to weapons? .Repeated intrusive thoughts that the patient knows it is wrong.Tactile: o Cocaine chronic use (most probably) OR delirium tremens o Do you feel ants / insects crawl on your body / skin? .Mostly regarding: cleanliness. playing something)? .You can not convince the patient it is wrong.If there is a fire in the building. concrete thinking! Insight: .If you find a stamped and addressed envelop on the ground. do you believe that they are talking about you? Delusion of reference o Do you believe that you are a special person? With a special talents? Or special power? Do you believe that you have a special mission to do in life? Do you think you deserve to be treated specially? Grandiosity o Do you feel other people are falling in love with you? Eromantic o Do you believe any part of your body is rotten? Cognition: . certain hobby.g.Pediatrics + Delusions: .The ideas o Believable (could be) – non bizarre o Unbelievable (could never be) – bizarre o Do you believe that other people would like to harm you? OR conspire against you? o Do you think that others would like to control you? o Read your mind? Thought broadcasting o Put thoughts into your head? Thought insertion o Steal thoughts from your head? Thought withdrawal o If you are watching the TV or reading the newspaper.Do you think that you are doing well? Or do you need help? Judgement: .Depression: o What is your mood? How do you feel? o Did you lose interest in things that were interesting to you before (e.Are you becoming forgetful? Are you losing your staff? . that do not match with the patient cultural and religious background .doc Page 212 of 255 .Anxiety: o Are you the kind of person who worries too much? o Do you have excessive fears or worries? .Psychosis: o Do you hear voices or see things that others do not? o Do you think that someone else would like to hurt you? OSCE-guide-III.False fixed believes.Assess abstract vs. what are you going to do? . even with proof . what would you do? General screening: . near the mail box. including major mental / psychiatric disorders. i. such as Autism.Schizophrenia: o 4 positive symptoms: hallucinations. were coded on Axis II in the previous edition. Panic attack might be one or more attacks c.Axis V: Global Assessment of Functioning or Children's Global Assessment Scale for children and teens under the age of 18 (a questionnaire) Example of a full proper psychiatric diagnosis: . disorganized speech. panic disorder: a.doc Page 213 of 255 .Axis II: Personality disorders and intellectual disabilities (although developmental disorders. .Depression: o MI PASS ECG or MIS GE CAPS o You need to find at least 5 of the 9 for > 2 weeks. Substance Use Disorders . If patient is avoiding going outside  with agoraphobia 2. negative symptoms: mood. there is at least ONE panic attack with at least ONE month of worries and fears of having it again b.Axis III: Acute medical conditions and physical disorders .Phobias specific to certain objects 3. we can replace it with agitation OR drop in school performance + other 4 criteria. psychosocial and environmental factors contributing to the disorder . catatonia … o At least 1 month of active symptoms (2 of 5) + 6 months of deterioration in functioning. disorganized behaviour.GAD: excessive unrealistic fears for more than 6 months PLUS other manifestations 5. o 1 other category. these disorders are now included on Axis I) . In panic disorder.OCD 4.Panic attack vs.Axis IV: Recent stressors. o 1 active symptom (not 2) is accepted in the following cases:  If the hallucinations are > 2 voices (commanding or commenting)  The delusions are bizarre .Bipolar I / Anti-social personality / DM+HTN / Divorce / global assessment was not done because the patient was not cooperative Diagnosis of diseases based on DSM-IV-TR is based on CRITERIA and TIME. and learning disorders. o If not fulfilling these criteria: non-specified mood disorder o In teenagers: we do not need M or I. including at least one of the Mode or Interest. delusions.PTSD (acute or chronic): Have you ever encountered a situation in which your personal or mental safety and wellbeing were endangered? When? Do you have flashbacks or nightmares? OSCE-guide-III.Anxiety: 1.e.Pediatrics DSM-IV-TR Diagnostic and Statistical Manual of Mental Disorders 4th Ed/2000 – Text Revision Multi-axial system (5 axes) The DSM-IV organizes each psychiatric diagnosis into five dimensions (axes) relating to different aspects of disorder or disability: . .Axis I: Clinical disorders. suicide. e. including constitutional symptoms 3..OCD . medications and specific diseases Thyroid disease Mitral valve prolapse Brain tumour / HIV Anxiety / psychosis Mood / psychosis Mood / anxiety Serious conditions (red flags): .SAD if IV drug use: check for liver (hepatitis) / constitutional symptoms (HIV) 2. homicide.1st time or did you have it .1st time or did you have it O 4. panic Criteria (1 month of 2-5 active E  High: DIG FAST + disorder symptoms + 6 month of R MI PASS ECG 2.PTSD ..doc Page 214 of 255 .Panic attack vs. P Criteria  Low: MI PASS ECG 1. add: HEADSSS Family history of psychiatric illness: suicide / depression / SAD / seen by psychiatrist OSCE-guide-III.g.Self care (are you eating / sleeping well?) .With whom do you live? Family support? For teenagers.Pediatrics History taking – Psychiatry MOAPS: mood / organic / anxiety / psychosis / serious conditions (self care.Specific phobias function deterioration) S 3.Rule out medical conditions as DD.How do you support yourself financially? . support) / HEADSSS Major psychiatric illness Suicide Minor psychiatric illness Personality disorder / Drinking / addiction / Eating / sleeping S disorders / Somatic disorders / A Mood Anxiety Psychosis Cognitive (delirium / D dementia) .What about the opposite? S Dx: one of the mood disorders Past psychiatric history Past psychiatric history MOAPS Organic: 1.PMH.Suicidal / homicidal ideation Social history: .GAD before? before? N 5. Psychiatry Mood disorders: OSCE-guide-III.doc Page 215 of 255 . Talking to some body. marijuana. why do you have this? In addition to ….CT / MRI brain Pay attention to patient cues . vomiting) / Brain tumour (N/V)  HIV  Thyrotoxicosis  Medications: thyroxin. tingling? OS: When did it start? What were you doing? C: Is it all the time.CBC / toxicology screen . do you have any other strange feelings? Any hallucinations: What? For how long? Any (tactile or visual) hallucinations  will be mostly organic  cocaine (substance abuse) until proven otherwise Have you seen a psychiatrist before? Cover the following:  Head injury / trauma  Brain infection (fever. amphetamines) Constitutional symptoms / self care / suicide Differential diagnosis: .Post-partum psychosis . numbness. anti-parkinsonism (L-dopa)  SAD (cocaine.HIV .Looking at wall or ceiling . It could be very disabling) .Poor hygiene .Mood disorder Investigations: .Schizophrenia (a mental disorder that impairs the way you perceive reality.Delirium / dementia .Brief psychotic disorder .Brain tumour . or on and off? Any specific setting? In you opinion.doc Page 216 of 255 . OSCE-guide-III.Paranoid .Septic workup .Drug-induced .Psychiatry Psychosis Pt comes to the clinic complaining of strange feelings in his right hand Clarify the CC OSCD Criteria :  Hallucinations  Delusions 1st time or did you have it before? Past psychiatric history Organic Mood / Anxiety Serious conditions Social history Family history 123456  Can you tell me what is going on! What do you mean? Is it pain.HIV / syphilis test . HIV (repeated infections / repeated diarrhea) If the patient came because his parents or roommate have concerns. we call that schizophrenia. mother is calling you?! What do you tell her? Talk to him to attract his attention ─ The doctor should get the phone number and address and ask the nurse to call 911 ─ Ask her if you can speak with the patient  psychosis patient ─ OSCE-guide-III. haloperidol). stand by your chair o I would like to assure you that you are safe here. Explain about side effects: weight gain / ↑ blood glucose level / ↑ cholesterol / drowsiness ─ Arrange follow up visit ─ Information e. did you see a gay patient today doctor? o Why are you concerned about that? o As a physician. sexual orientation or anything else! Do you think I am crazy doctor? o There is no medical term called “crazy”. are you Egyptian doctor? o Why are you concerned about that? o Whether I am Egyptian or not will make no difference in this situation I do not like “gays”. how do you feel about that? ─ Will start medication which is helpful in reducing the symptoms (Risperidone). religion.g.doc Page 217 of 255 . by the way. you can ask the patient: what kind of concerns does … have? Difficult situations: ─ ─ ─ ─ ─ ─ If the patient with hallucinations tells you that he sees a radiation and gives you a photo and asks: do you see it doctor?  For me it does not look like radiation. that we can treat with medications Case: A young man can not move his neck. it looks like your schizophrenia is relapsing. wants to fly. DD acute dystonia: ─ Trauma ─ Meningitis ─ Subarachnoid hemorrhage ─ Cervical disc ─ Muscle spasm ─ Anti-psychotic medication (e. I deal with all patients.: support groups / brochures Notes: ─ ─ Whenever you suspect substance abuse: after you ask “have you ever tried recreational drugs?” ask “what about crack cocaine? Do you sniff? Do you inject? Did you share needles” o If shared needles  scan for hepatitis (liver symptoms). treatment: lorazepam Case: Patient is in the balcony. that is why we need to admit you and refer you to psychiatrist to reassess your condition. no one will harm you I do not like “Egyptian people”.g. but I can understand that you see this as radiation At any time the patient starts to agitate and worries about special hallucinations! o You are safe here. no body will harm/hurt you If the patient is away: o Do not chase him/her around the room. sex.Psychiatry Management: ─ Will examine and do some tests o Because you have stopped your medications. by the way. regardless their race. However sometimes some people have difficulties in the way they handle their thoughts and the way they interact with and perceive reality. It is a mental illness like any other illness that can affect the body. Will you hook me to the cleaning machine that cleans the blood? I am glad you came here today. .doc Page 218 of 255 . but not with the machine.Psychiatry Schizotypal personality disorder - Delusions Magical believes Limited number of friends that share the same believes Ethical challenges: . OSCE-guide-III.Will you admit me doctor? We need further psychiatrist assessment then we may need to admit you. I think you need help. do you feel: . then comment to the examiner: “it looks regular / irregular for me” OSCE-guide-III. ask the patient “can you tap it for me”.doc Page 219 of 255 .light headed .Psychiatry Panic attack Patient comes to the clinic complaining of dizziness Clarify the CC Analysis of CC HPI Criteria during attack (AS) Between attacks Anxiety MOAPS When you say dizziness.Spinning Os Cf D Analyze the attack: How did it end? How many attacks? Are they similar? What were you doing? When was your last one?  ↑ cardiac: heart racing17 / chest pain / tightness / excessive sweating  ↑ respiratory: SOB / wheezing  ↑ neurology: dizziness / numbness / tingling / weakness / shaky / buzzing sounds / headache / vision changes / difficulty balance  ↑ GIT: nausea / vomiting / difficulty swallowing  Depersonalization: you feel that you are outside of your body  Derealisation: feel things around you are strange / not real  Excessive fears of: losing control / going crazy / dying  Do you have fears of having other attacks?  How does it affect you? Do you avoid going out? (Relation to agoraphobia?)  Are you the kind of person who worries a lot? Excessive fear  Are you under any stress in your life? How can you cope with this?  Any special fears? High altitudes? Closed places? Talking in public? Pets?  Have you ever encountered a situation in which your personal or mental safety and wellbeing were endangered? When? Do you have flashbacks or nightmares?       Past psychiatric history Serious conditions Hypoglycemia Thyroid disease Pheochromocytoma SAD (cocaine / amphetamine / alcohol withdrawal) / Caffeine Arrhythmias / MVP Anemia (fatigue / light headedness / heavy menses / PMH anemia) Constitutional symptoms Self care  Suicide AMPLE Heart diseases / thyroid / abdominal tumours   PMH Family history Social history 17 Any heart racing. in attacks. imagine you are crossing the road. o Now Mr … what do you know about “panic attacks”? o Do you want me to explain this in details over the next few minutes? Inform the patient: o Explain the pathophysiology: panic attack or panic disorder is a kind of severe anxiety. o The same reaction might happen suddenly without any external trigger. This is normal and useful reaction. TCAs (nortriptyline). breathing techniques / meditation) Treatment: o Like many other conditions.Psychiatry COUNSELLING - - - - - With what I heard from you today. headache. rise in blood pressure. occurring more days than not for at least 6 months. Usually it is related to stress.5 mg qhs x 2 weeks (it is important to use it on schedule. mindfulness. no PRN) o Buspirone (tid dosing) o Others: SSRIs/SNRI (paroxetine). it happens suddenly. beta-blockers o Avoid Bupropion due to stimulating effects OSCE-guide-III. work / school)  The person finds it difficult to control the worry  Treatment: o Lifestyle: caffeine and alcohol avoidance.g. it could be treated.g. GIT disturbances. not irregularly) • SSRIs: Paroxetine 10 mg od x 4 weeks – similar to what we usually use with depression. urine analysis. some sexual dysfunction. electrical tracing of your heart (ECG). and this would be stressful. sleep hygiene o Psychological: psychotherapy. and this is what we call a “panic attack”. they have their side effects. the most likely diagnosis to your symptoms is a medical condition that we call “panic attack”. regular schedule. o Consequences: this might happen again / may cause significant limitations Preventive measure: o Life style modification (↓caffeine and alcohol / better sleep hygiene) o Relaxation techniques (e. long half-life. • Follow-up 2-3 weeks Offer more information: brochures / web sites Whenever you suspect social problems  involve the social workers Generalized Anxiety Disorder (GAD)  Excessive anxiety and worry (apprehensive expectation). o It is due sympathetic over-activity. relaxation. to exclude other medical conditions and to confirm our diagnosis. and CBT o Pharmacological:  Benzodiazepines (short term. and you feel alert. Like any other medication. some investigations like blood works. And this improves by time.doc Page 220 of 255 . which leads to some changes: increase in the heart rate. about a number of events or activities (e. and a speedy car is approaching you. our body reacts to this by enhancing the sympathetic nervous system. o Treatment varieties include:  Talk therapy  Medications: 2 types • Anti-anxiety: Lorazepam 0. low dose. normally. We still need to do physical examination. CVS screen .PAD / impotence N.Morning or all day: ?depression .doc - Blood sugar measured Emergencies Page 221 of 255 .Lung .Cardiovascular .Urinary .Limitation of activity? How many blocks are you able to walk? .Constitutional symptoms .Weight loss .Do you wake up during night? .Drink more .Tender points in your body All systems review (head to toe): .B.Symptoms: Complications (chronic) vascular MICRO MACRO .Endocrine (thyroid / DM) ↓ sleep / insomnia ? depression Criteria : MI PASS ECG 1st time or did you have it before? What about the opposite? Past psychiatric history Organic …/…/… Anxiety / psychosis Serious conditions Social history Family history Counselling on depression MI – mood / interest PMH of cancer Social history – SAD Family history Diabetes Mellitus: .Pee more .Retinopathy .Blurred vision DKA .Not being refreshed after sleep? Do you have any special concerns? Timing: .Lack of energy? Tiredness? . hx / pregnancy Cancer colon for males .How many hours? And before? .Eat more .Is it weakness? Can not do? .Anemia / bleeding ± LMP / mens.Hx of DM Fluctuations (acute) . β-blockers are contraindicated in DM: it causes hyperglycemia / and it masks hypoglycemia OSCE-guide-III.Nephropathy .CAD .Tired Hypoglycemia .GIT / liver .Psychiatry Tiredness OR weight loss Introduction CC Clarify the CC Os Cf D Ask about sleep Tiredness .MSK / skin / rheumatology / autoimmune . do you feel refreshed? Do you need naps? ↑ sleep /or/ the same ? organic .End of the day: organic .When you wake up.Find difficulty falling asleep? .Neuropathy . doc 4. Depression (alcohol / depression / suicide) is common combination  Treatment for depression (or most of the psychiatric diseases): i.Abdominal pain .Headache .Young female: think menorrhagia .Depression:  Psychomotor question: do you think things take more time to do now? Compared to before?  Pancreatic cancer  depression  Whenever you find alcoholic patient  check for complications: i.Insomnia / sleeping pills .Fatigue .How many hours? How about before? .Do you eat before sleeping? Heavy meals? Late meals? .Diabetes Mellitus.Old person: think cancer & occult blood . GIT: upper GIT bleeding / peptic ulcer perforation iv.Anemia: .Do you sleep alone? Or do you have sleep partner? o Does he notice you are snoring? Do jerky movements? o Does he snore? Does he do jerky movements? .Sleep hygiene questionnaire: .Do you exercise before sleep? .Find difficulty falling asleep? How long does it take you? .Domestic abuse presentations . Medications Usually in combination 3.Do you drink before sleep? Alcohols? Coffee? . Liver damage (↑ liver enzymes) / hepatitis / cirrhosis / carcinoma iii.Anorexia nervosa Page 222 of 255 . Talk therapy iii.Domestic abuse .Depression / PTSD .Do you wake up during night? Any reason? Can you sleep again? .Do you work on shifts? 2. polyuria .When you wake up.Fatigue OSCE-guide-III. do you feel non-refreshed? Do you need naps? . Cancer pancreas ii.Do you wake up early? .Hypothyroidism .Do you have dreams? Nightmares? After .When do you go to bed? .Psychiatry Sleep / fatigue notes 1. Life style modification ii.Do you sleep in dark room? Lit room? .Do you read in bed? Watch TV? During .How does this affect your life? Do you work night shifts? empathy Before .Vaginal bleeding .Fibromyalgia . More at certain time of the week? .Difficulty falling sleep . my kids need me. if positive  screen MI PASS ECG PMH Social .doc Page 223 of 255 .Children? . taper benzodiazepines OSCE-guide-III.Do you wake up with nightmares? Depression .With whom do you live? Support?  Screen for domestic violence or spouse abuse .Waking up Analysis CC: Os Cf D . o What about if they are not around? Maybe! o This means: implicit yes to suicidal ideation  Lady looking for renewal of benzodiazepines: o Renew it and tell her that she needs to use it properly o Tell her that she is in grief.Financial support? Notes  Did you ever think to hurt yourself? NO.Psychiatry Insomnia A lady complaining of insomnia Common presentation to: domestic abuse / depression / anxiety Introduction CC Clarify the CC Insomnia / Tiredness .Did you try anything to help? Did it work? Ask about sleep Sleep hygiene questionnaire Anxiety . start SSRIs. she needs SSRIs.Screen with MI.Do you have too many worries? .Any changes / stresses in your life? .What comes in your mind before falling asleep? . Psychiatry Domestic Violence – Spouse Abuse Screen for domestic violence or spouse abuse: - - - ASSURE confidentiality: I would like to assure you that our conversation is completely confidential.Did he try to push you? Hit you? How many times? . I will not release any information.Did he ever get angry to the extent that he became physical? .Do you have access to financials? Do you take permission? .Does he start to shout at you? Swear at you? . whatever you will tell me here.Did he ever force you to do sexual activity against your will? How do you feel? - Duration of abuse? Severity? Hospitalized! How does this affect you? Are you pregnant now? Do you work now? How do you support yourself financially? Children involvement: .Any visits to the ER? When was the last time? Financial: .Did he ever to try to take you money against your wishes? Sexual: . unless otherwise required by the law! With whom do you live? How do you describe this relationship? Supportive? o How long have you been in this relation? o Do you feel safe at home? In this relationship? Do you or your partner go through stressful times? o Do you sometimes have conflicts? Arguments? Is there any chance that you partner drinks or uses drugs? How often? When he drinks.Did he try to put you down? Does he try to control you? How did this affect you? .Did you ever talk to anyone about this? OUTCOME: .Does he call you names? How does this affect your self-esteem? Physical: . does he become angry? Lose control? When was the last time? Verbal / emotional: .The patient decides to end the relationship and leave  you must provide support and shelter .doc Page 224 of 255 .Did he ever mistreat / abuse the children? Fatality: .The patient decides to continue: either with OR without police involvement OSCE-guide-III.Who is controlling the spending at home? .Did he ever mistreat / abuse you in front of the children? .Do you have access to weapons at home? .Did you ever have thoughts to put an end to this all by ending your life or his life? . and you should not feel guilty about that. if you want to continue the relationship.The husband needs help.Not answering directly for questions about the relationship with partner . and you do not need to accept this.It is not your mistake. It is an illegal crime.Psychiatry Wrap-up: . . in the city.I hate drinking . they will be able to help with housing.She is avoiding eye contact .It is important that you consider reporting the situation to the police for your safety. it will deteriorate. It is difficult decision to leave or stay.He is a great father. and it is against the law. . on the contrary. and may put charges against him. likes his daughters.Consequence to the children (if any): psychological trauma . and works very hard. …)  Easily accessible (you can pick it in second and leave) o We will schedule a follow-up visit within few days Is it ok with you if I document this? Support: o Police is not the only option. . they are free. they will come. The studies show that the longer you stay in this relationship.Based on what you have told me. you can still involve the police.I wish I can be a better mom / I am a failure / do you think too I am clumsy. do some investigations. .g. OSCE-guide-III. - - - - If you like to end the relationship and leave: o You can call the police. they will come and arrest him. We know from the studies that if you involve the police. speak with him. and then. things might deteriorate and get out of control and one of you might end up losing her/his life. the police can give a restraining order o I will connect you with the social services and support groups. and it is common. there are community recourses: shelter. bank documents. IDs.We know from studies that the situation will not improve. hotline. your husband will be obliged to attend special training courses: o Anger control o Stress management and relaxation techniques o Drinking problem rehabilitation o Marital counselling My concern is that if you go back without taking any measures. by law. o Give social support group numbers. and confidential. situation will improve. and nobody deserves to be treated in this way. you do not need to go through all of this by yourself. Clues for domestic abuse: . at least: o You need to prepare an escape plan:  Where to go when you don’t feel safe!  A bag with essential belongings (e. the higher the chance of abuse. It is unacceptable. check willingness to get counsel. . financial support for both of you and the kids However. and you do not need to worry that he might hurt you. legal aid. what you are experiencing (or have gone through) is called domestic violence or spouse abuse. then they will investigate the case.doc Page 225 of 255 . Do you have other children? .Other injuries before or visits to ER? .doc Page 226 of 255 .I: Are his shots up-to-date? If no.D: Is he hyperactive baby? Challenges you most of time? .B: screen for the risk factors for child abuse:  Was this pregnancy planned? Regular f/u visits?  Was he a term baby? Did he need special attention?  Has he had congenital anomalies?  Do you think he is a difficult baby? Fussy baby?  SAD for both partners! .Who witnessed it? Anybody else? .N: What is his weight? Do you know about his growth charts? Regular f/u visits? .Analyze each event .When did this happen? When did you come to the ER? .E:  How do you support yourself financially? Any support from the biological father?  Anybody at home seeing a psychiatrist? Illness?  Tell me more about your childhood .Psychiatry Child Abuse18 The child came to the ER with femur fracture. and after we finish I would accompany you to see him. the skeletal survey showed multiple healing fractures.Can you describe what happened? What he was doing? . counsel Introduction Analyze the event Is it the first time? BINDE Other children PMH of the child … I assure you that he is ok. is it ok with you. Before this I would like to ask you some questions to know more about his condition / fracture .Did you take him to the same hospital?  Are you the biological mother?  Is your current partner the biological father? .Repeated visits to ER? Chronic illness / bone or metabolic diseases Screen for domestic violence or spouse abuse 18 Good TWO screening questions: immunization (not up-to-date) / weight (FTT or under nutrition) OSCE-guide-III. any reason? . children at that age have flexible bones.It is not your mistake. and it is difficult to explain the nature of his fracture(s) only by jumping from a couch. and it is common. and you should not feel guilty about that. this is not their first priority Their first concern is the safety of your child. . especially if you do not have enough support.doc Page 227 of 255 . and nobody deserves to be treated in this way. what you are experiencing (or have gone through) is called domestic violence or spouse abuse. and you do not need to accept this. and it is against the law. It is an illegal crime. on the contrary. . The studies show that the longer you stay in this relationship. OSCE-guide-III. it will deteriorate.It is important that you consider reporting the situation to the police for your safety. the higher the chance of abuse.Also.Psychiatry Wrap-up: - - - How do you feel your child has so many fractures? … I know that you are concerned about your son. In the same time. what will happen is that they will: o Ask you some questions about what happened! o Come to visit you at home o Talk with your partner Then they will take their next step based on the results of these meetings I am sure you are sharing my concerns about … (the child name) safety! If there is spouse abuse / domestic violence: . sometimes it is challenging to look after a child. In these situations we usually involve the children aid society (CAS). based on what you have told me. It is unacceptable. this is a kind of social services devoted to the safety and well being of children o Please do not do this? Why? o They will take my son! Why r u saying so? Any experience with them? o Not necessarily that they take your child. .We know from studies that the situation will not improve. whose wife is recovering in the ER. I will ask her. Analysis:  Do you have any idea how did she end up having all these bruises?  Was there any argument / disagreement / shouting? Did you lose control? Did it end up that you physically hurt her?  Is this the first time or happened before? Any repeated visits to the ER before? Social history: How long have you been together? What is the nature of your relationship? Stable? Was there and significant conflicts before?  Was there any recent change or stressor in your life? How do you support yourselves financially? Do you have enough resources?  Do you have anybody else at home? Any family support? Do you have children? How is the relation with them?  SAD  Safety:  Criminal record / access to weapons at home  If you go home now and face the same situation. and he asked to speak with you. if that is ok with her. how would you react?  Any chance that you might hurt yourself or any other one? OSCE-guide-III. In the next 10 minutes counsel him Introduction Analysis SH / Safety Counsel  Domestic violence   Anger control Stress management and relaxation techniques Drinking problem rehabilitation Marital counselling  Offer social support if there is a need   Introduction:  If the patient asked to see you: I understand that you are here because you are accompanying your wife. I can take you there. How can I help you today?  If the patient is inquiring about her status: I can assure you that she is stable and in safe hands now.  If the patient asks to see her: After we will finish.doc Page 228 of 255 .Psychiatry Domestic abuser You are bout to see a 55/60 years old gentleman. she has bruises. she has bruises and my colleagues are taking care of her right now. Psychiatry Counselling:      I can see that you are going through stressful period of time. It must be difficult for you and your wife. Sometimes this stress might present by changes in behaviour and/or personality. If you do not have enough support at home, things might get out of control. What happened is what we call “domestic violence”; it is a kind of “physical abuse”. It is not acceptable, and it is considered illegal crime. However, this is your wife decision. If she chooses to report you, that is her right, and nobody can prevent her. She can press charges against you, and they will take you to the court, in this case you might need legal help, this might have serious consequences. On the other hand, if she decides not to take any measure, may be you should try to improve the situation by taking steps to decrease the stress in your life, and you can consider reducing your alcohol drinking. Drinking alcohol might leads to what we call “disinhibition” in which one might lose control on his reactions and usually this leads to violent and serious consequences. I can help you by referring you to attend: o Alcohol rehabilitation programs o Stress management and anger control programs  I recommend also that you consider attending family marital counselling; they have good experience in dealing with couples going through difficult times.  Finally, I can help you to contact the social services. They might be able to help; you can speak with them and see what they might be able to do! Is that ok with you? OSCE-guide-III.doc Page 229 of 255 Psychiatry Depression Screen: MI PASS ECG Organic: Illness: hypothyroid, anemia / pernicious anemia, M.S, cancer / cancer pancreas  Medication B Blockers, Anti-parkinsonian  SAD  Depression management / counselling 7. Inform the patient a. Based on what you have told me, the most likely explanation for your condition is a medical condition called “depression”, what do you know about depression? Did you read anything about it? It is the most common mood disorder, in which you feel low, upset and lack of energy. It is a common problem, and it is treatable. b. Explain the pathophysiology: it is related to imbalance of the chemicals in our brain, most likely related to decreased serotonin c. Consequences / complications of the condition: it affects functionality, leads to decreased concentration and ability to work, and in severe cases in susceptible persons, it might lead them to suicide 8. Treatment (outpatient): a. Talk therapy (psychologist / psychiatrist) b. Medications: SSRIs (which are very effective medications) take effect after few weeks  Cipralex 10 mg PO od x 3 weeks (side effects include: 1. weight gain, 2. GIT symptoms, 3. sexual dysfunction). These side effects improve by time and to reduce it, we start increasing the dose gradually (start low, go slow).  Because depression has serious (fatal) consequences, if you do not take this medication, you will be compromising your safety.  DO NOT stop it on your own; we can start to gradually decrease the dose after the proper period of treatment. You need to continue on it for at least 6 months after symptoms improve. c. Follow up visit after 2-3 weeks d. Contract: sometimes when the anti-depressant starts to work, the energy level improves while the mood is still low, that is why sometimes there is increase in suicidal ideation. Usually happens 2-3 weeks, you need to promise me that if this happens with you, call 911 or call me immediately and come to see me 9. Offer more info: brochures / web sites 10. Break every 30-60 seconds and ask the patient: does that make sense? Is this acceptable? Reasonable? Is it clear? 11. Treatment (hospitalization): for suicidal patients a. From what you have told me, you are meeting the criteria of what we call “…” and I have concerns about your safety, because you have more than THREE risk factors for suicide as per the screening test. Do you mind to stay with us in the hospital for few days, so we can do the required investigations and start the medications, until you feel ok, what do you think about that? ─ No doctor, I am not staying in the hospital!!! ─ Actually, Mr … as I told you, I have concerns about your safety, we can not compromise your safety. And by allowing you to leave today, we will be compromising your safety. OSCE-guide-III.doc Page 230 of 255 Psychiatry Dysthymia COMMON CASE IN THE EXAM  Depression presentations: o Sad (low mode), weight loss, insomnia, tired  Scale the sadness 0 – 10 o Indecisiveness: difficulty making decisions o Low self esteem  how do you feel about yourself? o If good days: ask for periods (check for gaps ≤ 2 months)  How does it affect your life? o Then assess functionality; what do you do?!  Screen MI: o If positive  MI PASS ECG  If positive  assess SAD PERSONS  Any relation to menstrual periods  pre-menstrual dysphoric disorder  Counselling: similar to depression, but mention that Dysthymia is a milder form of depression, with longer duration, and does not interfere with life functionality. OSCE-guide-III.doc Page 231 of 255 Psychiatry Premenstrual Dysphoric Disorder (PMDD) DSM-IV-TR Diagnostic Criteria for Premenstrual Dysphoric Disorder A. In most menstrual cycles during the past year, five (or more) of the following symptoms were present for most of the time during the last week of the luteal phase, began to remit within a few days after the onset of the follicular phase, and were absent in the week post-menses, with at least one of the symptoms being one of the first four listed 1. Markedly depressed mood. Feelings of hopelessness, or self-deprecating thoughts 2. Marked anxiety, tension, feeling of being "keyed up" or "on edge» 3. Marked affective lability 4. Persistent and marked anger, irritability, or increased interpersonal conflicts 5. Decreased interest in usual activities 6. Difficulty concentrating 7. Lethargy, easily fatigued, lack of energy 8. Change in appetite – overeating or specific food cravings 9. Hypersomnia or insomnia 10. A sense of being overwhelmed or out of control 11. Physical symptoms – breast tenderness or swelling, headaches, joint or muscle pain, sensation of bloating or weight gain B. The disturbance markedly interferes with work, school, social activities or relationships with others C. The disturbance is not merely an exacerbation of the symptoms of another disorder such as Major Depressive Disorder, Panic Disorder, Dysthymic Disorder or Personality Disorder D. Criteria A, B and C must be confirmed by prospective daily recordings and/or ratings during at least two consecutive symptomatic cycles (how to diagnose) Treatment  1st line: SSRIs highly effective in treating PMDD o Fluoxetine (20 mg od) and sertraline (50 mg od) most studied o Can be used intermittently in luteal phase (mid cycle  onset of menstruation – pre-menstrual) for 14 days  2nd line o Alpraxolam (Xanax) for anxiety symptoms  3rd line o ± OCP containing progesterone drospirenone (e.g. Yasmin) o GnRH agonists (e.g. leuprolide) o If GnRH agonist completely relieves symptoms, may consider definitive surgery (i.e. Total abdominal hysterectomy+ bilateral salpingo-oophorectomy) OSCE-guide-III.doc Page 232 of 255 Psychiatry Abdominal Pain / Headache Abdominal pain for few weeks, and was seen by a surgeon last week, comes to your office (± to have MRI OR to renew medication). Headache for 7 months, young man, comes to renew Tylenol 3 Chief complain ± a request (investigations OR medication renewal) HPI Analyze the CC     AS    Impact Red flags      Os Cf D / PQRST / ↑↓ / 1st time When did the headache (pain) start? Did you seek medical attention? What was the diagnosis? Did you take any medication? When did you start Tylenol 3? Why? Analyze previous visits: is the pain different from before? How? Other pains / headache GIT / liver Genito / urinary How does this affect your life? How are you coping? Constitutional symptoms Screen red flags for headache:  Trauma  Worse at night  Nausea / vomiting  Bothered by light /+/ Neck pain / stiffness  Weakness / numbness / tingling in body / seizures Are you under stress? Support systems DD Medical problem PMH FH SH Somatisation MOAPS screening (screen for depression) PMH: HEAD SSS FH of psychiatric disease SH Counselling Physical examination Notes: - Somatisation disorder: (4 pains / 2 GIT / 1 neuro / 1 sexual) complains - If the pain is only during the day, and not nights  mostly non-organic + If requesting MRI  NO MRI + Actions for Tylenol 3: - If using it for few weeks  stop it / do not worry about withdrawal symptoms - If using it for long time  counsel / renew / promise to cut down gradually - If using it for depression  start SSRIs / taper Tylenol 3 (decrease gradually) / then brief counselling for BOTH Tylenol 3 & depression - If drug seeker  DO NOT give any narcotics / rehabilitation + Always renew the medication, except for drug seeker (very anxious to renew the narcotic / will not accept another alternative / making stories to rash you to prescribe it). + If not drug seeker: reassure the patient that you will prescribe her pain medication before the end of the session, but in order to prescribe the proper medication, you need to ask some questions, can you bear with me for few minutes? OSCE-guide-III.doc Page 233 of 255 external benefits obtained or unpleasant duties avoided (e. co-ordinate necessary investigations  Biofeedback  Psychotherapy: conflict resolution  Minimize psychotropic drugs: anxiolytics in short term only. serves to reduce anxiety and conflict. aquatic exercises). antidepressants for depressive symptoms Somatization disorder ─ Conversion disorder ─ ─ Pain disorder ─ ─ ─ Hypochondriasis ─ Fibromyalgia ─ ─ ─ ─ ─ Lots of symptoms: ≥ 8 physical symptoms that have no organic pathology: 4 pain + 2 GIT + 1 sexual + 1 pseudo-neurology One or more symptoms or deficits affecting voluntary motor or sensory function that mimic a neurological or general medical condition (e. stretching. lat thigh. brain tumour) based on a misinterpretation of one or more bodily signs or symptoms Onset often after car accident Wake from sleep feeling un-refreshed Wide spread pain. or the idea that one has. low cervical C5C7. impaired co-ordination. seizures or convulsions La belle indifference Pain is primary symptom and is of sufficient severity to warrant medical attention Post-traumatic / post-surgical Exacerbated by psychic factors Preoccupation with fear of having. post neck. a serious disease (e.Psychiatry Somatoform disorders DD General Characteristics:  Physical signs and symptoms lacking a known medical basis in the presence of psychological factors  Cause significant distress or impairment in functioning  Symptoms are produced unconsciously  Symptoms are not the result of malingering or factitious disorder which are under conscious control  Primary gain: somatic symptom represents a symbolic resolution of an unconscious psychological conflict.g. CBT Amitriptyline 10 – 25 mg qhs Gabapentin 300 mg tid Similar to fibromyalgia but FATIGUE is the predominant feature Associated with sleep apnea / irritable bowel syndrome Page 234 of 255 . massage) Stress reduction. muscle strengthening. both sides of the body Characteristic reproducible tender points: occiput. local paralysis.doc ─ ─ Patient education Exercise program (walking.g. work) Management of Somatoform Disorders:  Brief frequent visits  Limit number of physicians involved in care  Focus on psychosocial not physical symptoms  Minimize medical investigations. lateral border of the sternum. med knee Treatment:      Chronic fatigue syndrome Factitious disorder / malingering OSCE-guide-III. physical therapy (good posture. double vision.g. no external incentive  Secondary gain: the sick role. lateral epicondyle 2cm below that point. above and below waist. . but we believe that because some patients are more sensitive to pain than others.It is not uncommon condition. but before that. - If there was a suicidal attempt: However. these stresses may manifest as painful experiences (symptoms). I would like to explain the findings in your case.Psychiatry Counselling for somatisation disorder: . in these situations.Based on what you have told me. I would like to explain the findings in your case. but we will agree that you will gradually cut it down. and medications (SSRIs.I understand that you are here because of …. . it is important to have only one physician dealing with all the investigations so that he can get better understanding of the whole situation. I do not have a family physician! o I can be your family physician. it belongs to a family of medications called TCA (tri-cyclic antidepressants) but we use it for pain control Counselling if the patient is depressed: .For Tylenol 3 we will not stop it suddenly. till the other medication (SSRIs) kicks in. during which we will review underlying symptoms. behavioural modifications. . and to renew medication and we will discuss that. I will renew it for you. . or may be due to patients difficulty in handling stresses in their lives. the most likely explanation to your symptoms is a medical condition called “depression”. . because of the suicidal attempt 2 days ago. over the next few days.doc Page 235 of 255 .I understand that you are here because of …. to make sure we are not missing any serious condition. o We will review the stress in your life and see how we can help you with it:  I can refer you to psychiatrist to help you deal better with any stress / conflict in your life  And we can consider some medications (Amitriptyline 25 mg PO qhs).What do you know about “somatisation disorder”? Would like me to explain? . and then you can arrange a meeting with your family physician and discuss the follow up with him. and to (renew medication / do MRI / …) and we will discuss that. That means we will set a follow-up visits every 4 weeks. TCA)]  depression counselling .We need to treat the depression with [talk therapy. but before discussing this.  NO. and we do not know the exact explanation for it. if you would like to. we would like you to stay with us in the hospital for few days so that we can start the treatment OSCE-guide-III.Do you have a family doctor?  YES o I will explain some points for you now.Based on the symptoms (± and the surgeries you had) the most likely explanation to your pain (headache) is a medical condition called “somatisation disorder”. how do you feel? What if you do not take it. which might have been harmful to you Introduction HPI Analyze the CC      AS    Impact      Red flags     Analyze Tylenol 3 Other medications Why are you taking it? What was the diagnosis? Os Cf D / PQRST / ↑↓ / 1st time When did the headache (pain) start? Did you seek medical attention? What was the diagnosis? Did you take any medication? When did you start Tylenol 3? Why? Analyze previous visits: is the pain different from before? How? Other pains / headache GIT / liver Genito / urinary How does this headache affect your life? How are you coping? Have the medications been impacting your life? Relationship with family Education. as I am a little bit concerned about the amount you have been taking.doc Page 236 of 255 . how do you feel? Shaking? Heart racing? You feel you are on the edge? Do you renew it from the same doctor or different doctors? Why you did not go to him this time? Is it ok that I contact him? Do you renew it from the same pharmacy or different pharmacies? Is it ok that I contact the pharmacy? Did you ever obtain the medication from the street? OSCE-guide-III. firmly ask him to stop. do you take any other meds? Sleeping pills? MOAPS screening PMH: HEAD SSS FH of psychiatric disease SH Counselling Analyze Tylenol 3 - So you told me you are taking it for … Who prescribed it to you? Who renewed it to you? Why? When was the last renewal? Can you show me your last bottle? How many tablets do you use now? And before? When did you start to ↑ the use? When you take it.Psychiatry Drug seeker If you find a man searching in the drawers of the hospital. beside for the headache relief. tell him this is private property and he is not allowed to go through this medical stuff I wish it could be that simple. Employment Legal problems. police involvement? Constitutional symptoms Screen red flags for headache:  Trauma  Worse at night  Nausea / vomiting  Bothered by light /+/ Neck pain / stiffness  Weakness / numbness / tingling in body / seizures Are you under stress? Support systems In addition to Tylenol 3. but I need more information and physical exam before I can write any prescriptions to you. if there is no strong indication to use it. OSCE-guide-III. Not only that. proper sleep hygiene. For that reason it is not the right step to renew it. It is an excellent pain killer if used for short term. running nose. and the problem will keep increasing. with weaning from caffeine and alcohol. We must stop the drug o I can help you with “sick note” o I can give you another non-narcotic medication that can help you with your pain . and can be used for long time. they will need to keep increasing the dose.Tylenol itself is a safe and effective drug. and heart racing. tearing. The patient may also develop “analgesic headache” syndrome. It is a “habit-forming” medication. but before that let me ask you: what is your understanding of Tylenol 3? . shaking.Tylenol 3 is a good medication when it is used for particular indication.I understand that you are here to renew your Tylenol 3. exercise. We call that “a habit forming medication”. sweats. however. and stress management. you will have “withdrawal” symptoms. o Also. in which inappropriately used analgesics actually cause headache. and can not stop it. diarrhoea. Counselling: . similar to that you have now. . muscle aches. Suggest a “drug holiday”. N/V. The seeker may be seeking “Fiorinal” - Fiorinal is a combination preparation of (barbiturate / caffeine / ASA) properly used only for the relief of occasional tension headaches.For these reasons. The fact that patient consumes a lot suggests overuse due to dependence. I am wondering that if you would be interested in talking to one of our social works here. I can refer you to a detoxification center. There are also some numbers you can call. o Can you give me just few pills. if it is used for long term. drowsiness. they are professionals to help people deal with medications or drugs. I would like to perform a brief neuro screening exam  move on. it is better to ↓ it as it might cause liver and kidney injury. diet control. Do you know what does it contain? It contains 2 medications: o One of them is the regular Tylenol as you buy it from the pharmacy o The other one is codeine . I have a very important interview? o Even if I give you few pills.On the other hand. full neurological examination is not indicated. do you know why? o First of all.Psychiatry Given the benign history with no suspicion of ↑ ICP or focal deficits. but. but based on what you described.If I renew your medication. who is expert to find out the community resources for you. the other medication “codeine” it is a drug belongs to the family we call “narcotics” which is similar to morphine. we will discuss that. and description of headache consistent with the common tension headache. . the more you will need it. .doc Page 237 of 255 . this will be temporarily. that can precipitate withdrawal symptoms: agitation. I will not be helping you. this is not the solution. you are having “dependence” on narcotics. if you stop using it suddenly. delirium and seizures.It sounds like you have been going through a lot of stress in your life. we call that “tolerance”. the more dependent you will be on it. and the more I renew your medication. it will be like a vicious circle. Or if you like. where they will help you to quit.I appreciate your trust to give me all the information. people get easily hooked on Tylenol 3. people need to keep increasing the dose in order to obtain the same effect. this is concerning for us. . What is the medication you want to stop? Why would you like to stop your medication? I am glad you came here to discuss it. sleep. any other concerns When were you diagnosed? How? Any serious consequences? Were you hospitalized? For how long?  Were you seen by a psychiatrist? Regular f/u? Do you feel: DIG FAST (distractibility. talkative) MI PASS ECG   Today Lithium Scan for depression History Do you renew your medications on regular basis? How much lithium do you take? From the beginning?  Are you taking it regularly?  Do you measure lithium level? On regular basis? When was the last time? What was it? What is your target?  Are you still taking it? Did you stop?  How do you feel about lithium?  Hypothyroidism: do you have your thyroid hormone measured? Do you feel cold? Dry skin? Constipation?  give thyroxin  Diabetes insipidus: do you feel thirsty? Drink more? Pee more? Got your urine checked? ttt: thiazides  Abdominal pain? Nausea / vomiting?  Neuro – shaking/tremors: β-blockers  Neuro – ataxia/balance/seizure: stop it I know that you have been asked all these questions before. grandiosity. activity. let me ask it for another time!   Side effects MOAPS Do you feel: DIG FAST (distractibility. impulsiveness. talkative) D Do you have a lot of projects? Were you able to finish it to the end? Can you focus on multiple projects?  Are you spending more money than before? Are you borrowing money that you can not I pay back? Are you over-using your credit cards?  With whom do you live? Many sexual partners?  SAD: what started 1st. sleep. impulsiveness (with painful consequences). feeling high or talking drugs?  Have you had problems with the law? Fighting? Arrest? Speeding tickets? G Do you feel very special? Have special mission? Do you feel a lot of thoughts? Ideas? F A How much time do you spend on your projects? How many hours do you sleep? Any changes? S T Did anybody mention that you are talking fast? OSCE-guide-III.Psychiatry Lithium discontinuity Introduction     Mania History Have been diagnosed with bipolar 3 years ago. activity. grandiosity. flight of ideas. flight of ideas.doc Page 238 of 255 . and would like to discontinue your medication. thought block is not a side effect of lithium. . this is not related to lithium.2). diarrhea .Agranulocytosis (Carbamazepine): check CBC every week. which is within therapeutic target (0. start to spend too much. we can measure it today and we can try to decrease it gradually to check if you are feeling good and closely monitor you. irritability .Based on your lithium level.I understand you are here because you would like to discontinue the lithium. then every 2 weeks. . We can treat and control it. You can see it is increasing. But you have to promise me that at anytime you feel high mood.This is not the case for mania/bipolar. but we can not cure it.Nausea.doc Page 239 of 255 . vomiting. I would like to know your understanding about mania and mood disorders! . then every month Counselling: . .Mood disorders are common.Headache . and the most common of them is depression where people feel low and do not concentrate and its treatment include the talk therapy and medications that could be used for 6-12 months and could be stopped if the condition improved and in some times we need to give the treatment for longer periods of time. You may try some relaxation techniques to help you concentrate more. may be one day in the future we will be able to do this.Liver toxicity .5 – 1. talk fast or start not to sleep well. OSCE-guide-III.Regarding your inability to write. and after the third time it goes higher to 90%. however before we discuss that.Insomnia. you have to come to see me or go to the nearest ER and inform them.Psychiatry Side effects of any medication: .Your chance of relapse if you stop it is 60% and after the second time this goes up to 80%. . moving around  Ask whether the patient has been on medication before or not.: Lithium  Ask about any side effects of lithium medication N/V / Diarrhea / tremors / polyuria  Obtain history in the usual format Introduction Ask about the Mood Assure the patient Red flags HPI MOAPS PMH / FH SH Assure the patient: you are safe here. symptoms (heart racing. If Lithium is causing some troubles. laughing. visual field changes) Psychiatric disease N. sweating. This is because he stopped taking the Lithium.If high mood: manic attack. if any patient has mood disorder. e. hyperthyroidism: history of thyroid problems.If no high mood: brief psychotic disorder / schizophreniform Usually patient brought by police or family member or asked to come by family members  Patient is talking fast and a lot. go through DIG FAST and MI PASS ECG Management:  Explain that the patient has recurrence of his mania or bipolar. with psychotic feature . you are meeting the criteria of what we call “manic episode” and I have concerns about your safety).  Usually you need to admit the patient to control the symptoms of mania (from what you have told me. we can start another medication.B.g.doc Page 240 of 255 . heat intolerance.  Will examine and do some tests. neck swelling.Psychiatry Manic patient If the patient is psychotic: First step is to detect early what is his mood? . you are in the hospital and no one will hurt you Fever / headache / nausea & vomiting / head injury  OCD  Mania (DIG FAST)  Depression (MI PASS ECG)  Suicide (SAD PERSONS)  If you leave what will happen? What would you like to do? Screen  SAD: alcohol / substance abuse / amphetamine  Medical conditions. OSCE-guide-III.  Will start medication. Do you feel safe at home? Then ask gradually. was it organized? Or it was an impulse? Did you leave a note? Recently. if there is a chance that this parent might get angry when he drinks? May shout. o If happy. ask about the relations with him and with other parents. may push.Psychiatry Suicidal attempt Introduction - Analyze the event - Before After Psychiatric assessment Risk MOAPS - - LOTS OF EMPATHY … And to see what should be the next step. and may hurt? . may swear at. are you going to hurt yourself? Were you seen by a psychiatrist? Were you given a diagnosis? Do you see your psychiatrist? Take meds? Assess the risk factors: Analyze SAD PERSONS Screen for anxiety Screen for psychosis Screen for suicidal / homicidal ideation / self care Past medical history / allergy / medications / … Decision Conclusion / Counselling SAD PERSONS S A D Sex Ag Depressio Mal e n e > 65 P E Previou Ethano s l attempts SAD 3-4 >5 E R N S R Rationa l thinkin g lost S Suicid e in the family O Organize d plan N NO suppor t S Seriou s illness HEAD PMH SSS Release if enough support Hospitalize - SAD What did you think will achieve by ending your life? Sometimes people hear voices asking them to end their life. did you hear this? HEAD SSS H: With whom do you live? … Anybody else? … Anybody else? If there is a stepparent in the image. I would like to know how you feel about being saved. what are your plans? Where do you want to go? What do you want to do? If another crisis may happen. first.Past medical history OSCE-guide-III.doc Page 241 of 255 . have you been giving your belongings away? What is going in your mind now? If you leave the hospital. I am glad for that o No! Assure confidentiality Can you tell me more about what happened? What is the name of the medication? How many tablets? Any alcohol with it? Why did you do that? Is it the first time? Who saw you and brought you to the hospital? Assess the plan here. you can come to my office or call 911.I can arrange a meeting with a social worker. I have concerns about your safety. o Why do you think this would help? “She will not be angry” I see.g. regrets the accidents.Based on our interview. or we can ask a nurse or a social worker to be there. .Whenever you hear “car accident”  show empathy / did you hurt yourself / ask about who was in the car / was any one injured? .doc Page 242 of 255 .If no eye contact. life is full of challenges.The girl asks you to tell her mother that she crashed mother’s new car! She does not want to directly (herself) inform the mother! o I can not do this. what do you think about that? RELEASE . It is important that you learn how to deal with challenges. e.Psychiatry Decision: .However. OSCE-guide-III. however. she regrets what happened. Do you mind to stay with us in the hospital for few days. but you have to arrange a follow up meeting with your family doctor within 3 days.I would also like you to promise me that if at any time you want to harm yourself or end your life. we can arrange a meeting with your mother. no SAD PERSONS risk factors  she is competent  respect her wishes. If you feel over whelmed. wasting time. talk to somebody.It she is ok. she is happy to be saved. If multiple suicidal attempts  borderline personality disorder  do NOT admit Notes: . so we can do the required investigations and start the medications.Based on our interview. no pt interaction  assure confidentiality .If the person driving was < 18 and was driving alone  be curious  this must be an important meeting / person that you really did not want to miss! . it is ok if you would like to leave. I would like you to know that life sometimes could be challenging. a psychiatrist! . because you have more than THREE risk factors for suicide as per the screening test. o We can help you to tell your mother by yourself. and you may face challenges in the future. until you feel ok.If still depressed and/or SAD PERSONS (>3-5)  admit . no SAD PERSONS  release Conclusion / Counselling: HOSPITALIZE . you will seek medical help immediately. and ask for help . I can be present.The girl does not want to inform her parents that she did attempt suicide! o You assess her and if she is to be released. . it is better that you try to learn how to deal with challenges yourself. S: HEAD SSS FH Eating disorder / psychiatric illness / suicide Weight analysis: .What is your weight today? . How do you feel about that? I am glad that you came: .To assure your parents (if she is not ok) Weight analysis Diet Exercise Extra measures Impact MOAPS .Do you calculate calories? How much calories do you eat per day? . her parents brought her because they have concerns about her weight Anorexia nervosa Restrictive Bulimia nervosa Binge-purge Under weight Distorted self image Amenorrhea Disturbed perception  loss of insight  incompetence  inform parents and admit involuntarily Introduction Average low weight Binge-purge (> 3 times / week) > 3 months Lose control  over-eat  react (purge) Your parents brought you …. .When do you look at yourself in the mirror.Mood: scan for depression .When did you start to lose weight? What was your weight at that time? How much did you lose? What was your highest weight? What is your target weight? .Do you eat alone or with other people? . how do you perceive yourself? How do you perceive your weight? . I would like to know how did you achieve that? Diet: Let us talk about your diet.AP: screen for anxiety / psychosis .Organic: DM / hyperthyroidism / constitutional symptoms (cancer) .Why are you losing weight? .How many meals do you eat per day? How about snacks? o What do you eat in breakfast? How about the amount? .It looks like you lost a lot of weight in short period of time.Psychiatry Eating disorder Young female.Do you like to dress in baggie? .To figure this out (if she is ok) .doc Page 243 of 255 .Do you like to collect recipes? To cook? OSCE-guide-III.Are you losing weight alone? Or someone else is encouraging you? . .Psychotherapy (individual/group/family): addressing food and body perception.How many times a week? . recognition of health risks) Notes: So doctor do you agree with me that I am overweight? Or do you see me like my parents I am not good? .Do you feel cold / tired / swelling in your legs? .Anorexia patient is to be admitted to hospital if: o <65% of standard body weight (<85% of standard body weight for adolescents).Heart racing? Light headedness.Do you take stool softeners? Do you take water pills? .Do you dance? Practice any sports? Extra measures: . o Heart rate <40 bpm o Abnormal serum chemistry or if o Actively suicidal .Do you take anything else to help you to lose weight? . family therapy.Muscle cramps? Calf pain? . it looks like you lost significant weight in short period of time. psychological (cognitive behavioural therapy.Pigmentation on your skin? Fine hair growth? Skin changes? .Do you sometimes exceed the amount of food you intended to eat? How many times a week? .If the patient lost interest  slow down  summarize and start again slowly OSCE-guide-III.Treatment: biological (treatment of starvation effects.doc Page 244 of 255 .I will share your parents concern.It is affecting your body.Monitor for complications of AN .Did you try before to induce vomiting? . SSRIs).Psychiatry Exercise .How about exercise? Do you exercise? . and this is concerning. health effects . o Hypovolemia requiring intravenous fluid.Any bony pains? Fractures? .Do you have amenorrhea? When was the LMP? Regular? . dizziness.I am concerned that you have a condition called “Anorexia Nervosa” (explain) .Agree on target weight on admission and reassure this weight will not be surpassed . coping mechanisms.Monitor for re-feeding syndrome: a potentially life-threatening metabolic response to refeeding in severely malnourished patients resulting in severe shifts in fluid and electrolyte Bulimia nervosa: . fainting? Conclusion: . It is a tough task but I will refer you to a multi disciplinary team to start treatment .Would you like to discuss that with your parents Management of anorexia nervosa: .How do you feel after that? How do you compensate? Impact / consequences: Because you have lost a lot of weight. without treatment it could be fatal .Criteria for admission: significant electrolyte abnormalities .The treatment is to start eating and to gain weight. I would like to know the impact of this on you! . and I will ask you about it later! (penny/ tree/ car) 14-18 / Concentration: can you spell the word “world” backwards? He gets -1 for each nonmatching letter (first check if he can spell it correctly forward) 19-21 / 3 words recall – delayed: can you tell me the 3 words that I told you before 22-24 / 3 steps command: give all the instructions at once.doc Page 245 of 255 . are you left or right handed? Can you please take this paper by the … hand / fold it into halves / give it back to me? 25-26 / Aphasia (pen / watch): what is the name of this? What is this? 27 / Read and execute: can you read this sentence and do what is written in it! 28 / Write: can you write a sentence for me! 29 / Copy: can you copy these two shapes! 30 / Repeat: can you repeat after me. in which I am going to ask you some questions. or buts”! MMS score < 24  incompetent OSCE-guide-III. please do as much as you can!  Prepare this list before you go to the room in cases of delirium / dementia / post-concussion. Some of these questions are easy. ands. we will do a mental exercise. “no ifs. ands or buts 17 o 18 w 3 3 words recall – immediate 5 world – backwards 3 3 words recall – delayed 3 2 1 1 1 1 3 steps command Aphasia (pen / watch) Read / execute Write Copy Repeat 1-5 / Orientation to place: do you know which country we are in? Province? City? Hospital (or street) name? Which floor (or suit number)? 6-10 / Orientation to time: do you know which year we are in? Season? Month? Day of the month? Day of the week? 11-13 / 3 words recall – immediate: I am going to tell you 3 objects. Then you can mark the correct or the wrong ones 1 2 3 4 5 5 Orientation to place 6 7 8 9 10 5 Orientation to time 11 14 19 22 25 27 28 29 30 d 12 15 20 l 13 16 21 r 23 24 26 Close your eyes! No ifs.Psychiatry Mini-mental status exam: Introduction: Mr … Now. and some questions are difficult. and I would like you to repeat after me and memorize it. Accounting: who is responsible for banking at home? Did you ever give cheque without balance? .Are you having difficulty memorizing numbers? .Hygiene: difficulty dressing and undressing yourself? do you remember to get all your meals? Or do you skip meals? do you have difficulty moving around? how about urination? Have you ever lost control or wet yourself? any difficulty having showers? IADL – SHAFT: .Any fluctuations in memory level? . sleep at day.Eating: .Ambulatory: .Toileting: .Do you have difficulty finding words? .Do you make lists to remind you to do things you used to do on regular basis? Do you have difficulty organizing your schedule? . or old events? o Recent: What did you have for breakfast? Confirm from partner! o Remote: Who was the USA president during WWII? (Roosevelt) ADL – DEATH: .Do you lose your stuff? . what are you going to do?  How about your sleep? (dementia: fragmented sleep /+/ delirium: reversed sleep cycle.Do you have difficulty reading? Writing? Calculating? . awake at night) MMS DEATH SHAFT MOAPS Let us take a day of your life.Psychiatry Dementia Difficulty with memory for 6 months Introduction Analysis of CC Behavioural changes I would like to ask some questions. then we will do a mental exercise Memory assessment  Did anybody tell you that you have changes in your personality? Being short temper? More arguments?  If there is a fire in this building.This deterioration is gradual slowly progressive.Dressing: .Shopping: who is responsible for shopping? You or your wife? . like tying a tie? .Traffic: do you drive? Difficulty driving? Have you ever lost your way? OSCE-guide-III. are you able to help your wife? .Do you feel difficulty for new events. or is it you feel ok for a while then you have attack then you are fine then you have another attack? (step ladder) . I would like to see how did it affect your life? Activities of daily living (ADL) Instrumental Activities of Daily Living (IADL) Organic in details and screen the rest (especially mood for pseudo-dementia) Memory assessment: Can you tell me more about this difficulty! OCD + .Do you have difficulty doing tasks you used to do before.House keeping: how about house keeping.Food: do you cook? Did you ever forget the stove on? .doc Page 246 of 255 . Thyroid disease (especially if pt is younger than 60 years) .Any history of thyroid disease? Symptoms of hypothyroidism?  Hx of surgeries? In stomach?  Are you vegetarian? For how long? Do you take supplements?  pernicious anemia Anxiety Psychosis Self care / suicide Dementia cases: . . The cases could be: .e.NPH (normal pressure hydrocephalus): if the patient has difficulty in AT of the “DEATH”. In the next 5 minutes. but during taking history.SAD  History of stroke? Difficult with vision / hearing? Weakness / numbness? Loss of balance? Urinary incontinence?  Head trauma? Injury?  Brain tumour / infection . falls due to ataxia and urinary incontinence OSCE-guide-III. . comes to your clinic complaining of difficulty with memory. take history  thyroid.doc Page 247 of 255 .Do you have nay long term disease? Kidney? Lung? Heart? .Depression – pseudo-dementia .Dementia . In the next 10 minutes take history and assess (make MMS exam)  Dementia.Medications? OTC? Sleeping pills? .HIV . the examiner will give you the score)  Alzheimer.Pernicious anemia .55 years old patient comes to your clinic because he has difficulty in memory.Depression – pseudo-dementia? Organic: .Psychiatry MOAPS screening: Mood: . His MMS score is 21.67 years old man.69 years old man comes to your clinic because he is keeping forgetting for the last few months. and if you mention: I would like to do the MMS exam. take history and assess (this is too long for 5 minutes.Alzheimer disease . i. In the next 5 minutes. Is he hypertensive? Controlled? Regular measurements? .Psychiatry Delirium Delirium cases: . Counsel him! (insulin induced hypoglycemia  stressful event  decompensate a border line delirium) .Sleeping pills. he needs to be taken to the nearest ER.doc Page 248 of 255 .A middle aged gentleman comes to your clinic because his dad is not himself for the last 3 days. we will need to decrease or stop some of his medications. Your dad needs to be seen by a doctor ASAP. Theophylline (for asthma): stop and take beta 2 agonist instead Erythromycin (for pneumonia): change the antibiotic Lorazepam: discontinue OSCE-guide-III.A middle aged gentleman comes to your clinic because his mom is in seniors home. for how long? Controlled? Regular f/u and measurements? HbA1c? . not feeling himself. Take history by proxy .Patient has surgery 3 days ago.Cholesterol / Water pills / Anti-depressants . not feeling himself.I can see that he is diabetic. Patient will be aggressive. do you have a list with medications? Go one by one! . and she is not herself. and restart them gradually. . If he is too far. ask if it was prescribed by the same doctor . they gave her 15 units of insulin instead of 5 units. if more than 1.Patient has surgery 3 days ago.Erythromycin!!! Why was he taking it? Pneumonia! Conclusion It looks like your dad has a medical condition called “delirium” it is a serious condition. can you bring him to see me. Patient will keep repeating: “I do not know”!  mini-mental status exam Case 1: Dad has not been himself / not sleeping well Introduction I will ask some questions in order to reach a working plan Analysis of the CC  How old is he? What are your concerns?  Tell me more! Any recent stress? OCD  Did you notice if your dad is angry / aggressive?  Does it look like your dad is seeing things do not exist? Hearing voices? Complaining of insects crawling on his skin?  Does he sleep during night? What a bout during the day?  Is he eating? Taking care of himself?  With whom does he live? How is he capable of keeping life? How does this affect his / their life?  Is it first time?  Constitutional symptoms Causes DD  Any headache / vomiting / neck pain / skin rash / red eyes / any ear discharge / runny nose / teeth pain / diff swallowing / SOB / cough / Infection urine changes / abd pain / calf pain / swelling Trauma  Head trauma? Injury? Surgery  Recent surgeries? Pain at site of injection? Dressing change? SAD  SAD Medications  What about medications. impairment of LOC.For delirium. however.It the patient is not cooperative. It is reversible.Mental status exam = psychiatric interview . but I can assure you that nothing will hurt you! Analysis of CC  I can see you are scratching. Romberg test. keeps repeating “I do not know”.If the patient is starring at the wall. for follow-up If confused patient (long case – examination)  GCS: only if the patient is poorly responsive  MMS  Cranial nerves  Body: . they will go with you. we do the MMS exam every 3-6 months.Cerebellar tests: finger to nose. After you finish. it does not look like spiders. you can continue the rest of your exam .  Will give medication to help you calm down  Will have a nurse close by if you need any thing  Will keep the room quiet and well lit  Will come back again to see you Notes: . do you have a list with medications? Any sleeping pills?  Do you have nay long term disease? Kidney? Lung? Heart? Conclusion It looks like you have a medical condition called “delirium” it is a serious condition. planter flexion power  Patient supine: tone OSCE-guide-III.Power / sensation / reflexes  Patient standing: gait.For dementia.Hoffman’s reflex – thumb flexion  UMNL . delirious and uncooperative I can assure you that are safe here.Psychiatry Case 2: DT Patient is agitated. we do the MMS exam daily until he improves . start to ask the questions of the MMS exam. do you see anything there? .Pronator drift . you are in the hospital and no one will Introduction hurt you. do you feel anything? Do you hear / see anything?  Do you think any one would like to hurt you? Assure safety!  When did that start? OCD?  How was your sleep? Full MMS exam  Constitutional symptoms Causes DD  Any headache / vomiting / neck pain / skin rash / red eyes / any ear discharge / runny nose / teeth pain / diff swallowing / SOB / cough / Infection urine changes / abd pain / calf pain / swelling Trauma  Head trauma? Injury? Surgery  Recent surgeries? Pain at site of injection? Dressing change? SAD  SAD: any shaking / sweating Medications  What about medications. do you see anything? Do you see anything else? Do you hear voices?  Doctor. ask him: I can see that you are looking to the wall. do you see the spiders I see? For me.doc Page 249 of 255 . fluctuating. I understand that you can see them at the moment. we would like to help you  I can see that you are looking to the wall. rapid alternating movements . It affects 25% of Hospitalized people. Nicotine gums iii. car. If she/he would like to know more information or need help.Previous attempts: How many times? Why did you fail? When was the last time? EMPATHY: “failure” is a normal part of trying to stop 5. and weight gain) iv.Red flags:  Constitutional symptoms  Risk factors (personal history or family history) of:  Heart disease / attack / HTN  Diabetes mellitus / hyper-cholesterolemia 8. Remove cigarettes and other tobacco products (e.5 mg bid x 4 d then 1 mg bid x 3 months  Investigations: i.Is there any other smoker in your home? Is she/he willing to quit? It will be a great idea if both of you tried to quit at the same time. ashtrays) from your home. anxiety. 7 mg] ii. nausea and a craving for tobacco. or if there is hemoptysis: chest x-ray OSCE-guide-III. we can arrange a meeting 6. this will increase the success rate of your trial. I need to have the bigger picture about your smoking. Nicotine inhaler  Psychological support for smoking cessation (to ↓ the craving): i.5 mg qAM x 3 d then 0. I will be more than happy to meet her/him. insomnia.g. Tell your family. but first let me ask you some questions. tongue. seizure disorder. Anticipate the challenges you will face (nicotine-withdrawal effects: headache.Impact (complications of smoking):  Cancer (lung – hemoptysis. then stop smoking + 150 mg qAM x 3 days then 150 mg bid x 3 months + Contra-indications: epilepsy.Plan:  STAR: i.Psychiatry Smoking Cessation – counselling 1. irritability. and this will help us to figure out the best plan to achieve our goal 2. nasopharynx. and work  Nicotine Replacement Therapy: i. Zyban (Bupropion): + used with tapering smoking for 2 weeks. they will be your support iii. print papers with this date and stick it under your vision so that you see it frequently during the day ii. urinary bladder. … We will speak in details about how we can work together to achieve this healthy goal. If patient is worried.Congratulations. Champix (Varenicline): ↓ urge to smoke and ↓ withdrawal symptoms + 0. patients undergoing abrupt discontinuation of ethanol or sedatives ii.Smoking history:  When did you start smoking? For how many years?  How many cigarettes per day? 3. Set a quit date. CBC / urinalysis / lipid profile ii. If there is risk factors for heart diseases: stress ECG test iii.doc Page 250 of 255 . 14 mg. friends. Nicotine patch [21 mg (if smoking > 25 cig/day). eating disorders. other cancers)  Cardio vascular hazards (myocardial ischemia) 7.Reasons (motivations): to seek smoking cessation 4. I am going refusal for spiritual therapy! OR whatever cause! Ask for reason: Why? How long have you been thinking that way? Did you talk about this with your family? Assess competency.Family history of lung cancers Support . the surgeon thinks that “chemo” and/or “radio” therapy are the best line of treatment for you.How are you coping? History Brief history . counsel her. I do not want to contaminate my body with chemicals. she can refuse treatment . do you want to do arrange that? . .The side effects of treatments and the complications of not getting treatment (terminal illness) .Based on your condition and stage of cancer. I will arrange the family meeting. I would love to respect your wish. . and you go for spiritual therapy! . I will come back to talk to you again tomorrow In not depressed  she is competent. so we have radio / chemo .Give it more thoughts. was recently diagnosed with lung cancer.When we talk about decision-making.Once diagnosed.You sound depressed to me. usually surgery is late to be done.Available treatment .What about arranging a meeting with some one who has had the same medical condition.After all this is your decision.No doctor. refusing the surgery because her husband died 30 years ago in a surgery (she does not know why) / she thinks that people die from the anesthesia OSCE-guide-III. rule out depression: . You will get better insight into the disease and you will see the results of treatment. we need to rule out certain conditions and to know more about your health.What is your understanding of … (lung cancer)? . Introduction .With whom do you live? .Why not to try both? We start the medical therapy that we are sure it works. death. and speak with him/her. this is based on the many clinical trials and evidence-based medicine.Explain the condition. so that he can explain condition in more details? - A 71 years old with cancer colon. social worker. Treatment . but I would be more comfortable to follow that after you talk to psychiatrist. Can you please tell me how has your mood been lately? Do you find yourself cry easily? Interests? Suicidal ideation? Any major event.Do you know which type you have? .How do you support yourself financially? Lung cancer . the available treatments .SAD . I just want to make sure you know the available treatments that were proved to be beneficial in treating lung cancer. .Psychiatry Refusal to treatment – counselling Mrs … 56 yrs old.Any family support? . patient concerns: afraid of living with colostomy / concerned about being a burden to the family / afraid of complications of the surgery (a friend died in similar surgery) A 70 years old lady with lung cancer.Now. I would like to explain the treatment options we have.How about arranging a meeting for you and your family members (if you wish) with the surgeon.doc Page 251 of 255 . accident in the family recently? MI PASS ECG … Counsel If depressed  assess suicide and psychiatric consult .I understand you were diagnosed recently with lung cancer . and psychiatrist. OSCE-guide-III.Psychiatry Truth telling Usually a son or daughter asking you not to inform the patient (parent / grandparent) about his terminal illness or advanced condition Introduction: .Patient may need to start some arrangements Explain the implications not to tell: . it is not unusual for families to have that request! .We need to discuss the treatment options. fragile personality. . people will go through different stages when they react to bad news. if they need someone to talk to cope.Well. you know your loved one the best. the patient has the right to know.I can give the family a little bit more time to think and we will talk again. Explain the reasons to tell: . it is a team work. will eventually know or find out . it is reasonable to have that concern. I can arrange that if they want . it must be very hard on the family as well.Patient will have suspicion about his own condition .Does the patient have advanced directive? Will? Have discussed this before? Explain patient must be told: .Will talk to the patient to see if she/he wants to know all the details or not?!! o If yes.Well.I can tell that the patient has a very caring family.On the other hand. usually. given her/his previous reaction.In all cases.Patient has the right to know . and it is the decision on the patient .It is difficult to hide.Why you do not what her/him to know? (can’t handle the bad news. if the patient asks. …) . besides. we will ask if she/he would like us to inform someone else . depression.Patient will lose the trust to doctors in general Decision: . patient will eventually need to know the truth. we can not hold this right. we have to tell her/him Conclusion: . we have specialists here who can really help the patient and family to go through these stages. we have to tell her/him o If no.doc Page 252 of 255 . When he came in. won’t affect the face Address any questions or concerns OSCE-guide-III.If he had advanced will. Explain the condition: brain death .doc Page 253 of 255 . his heart was not beating. we did the compression . decision should be made within the next 24 hrs . driver license .You will be notified which organ used and where to go.Many organs can be used .We have a team to do that. (check with the relatives if they know this terminology) o Irreversible brain damage. will never gain consciousness. .He was healthy.Show empathy: sorry for the loss. he does not breathe o Legal term for death. to pain o No spontaneous breathing. no functioning at all o Not responding to light. the decision is a “life gift” Explain how to do it . good candidate for organ donation .Patient is in a state called the brain death. patient is not responding to treatment due to the severe trauma to the head. patient was young / healthy / family needs him / … Give time to family to recall how active and how nice he was Bring the issue of organ donation: .What is the outcome.Still can have the open casket.There is time limit. they will respond very quickly .Psychiatry Organ Donation Explain resuscitation effort .Confirmed by two nerve specialists: neurologist / neurosurgeon: .It won’t affect the arrangement for funeral .What family’s view about this .What has been done to the patient “resuscitation” . we put down a tube. but you won’t get the individual’s name Explain funeral . when we stop the machine. he was not breathing.It is family’s decision now. Impact on life. religious. .Type of obsession: dirt and contamination. checking and rechecking? .Screen for psychosis . orderliness.  MOAPS: .Psychiatry OCD - Address body language Why do you wear gloves and hat? And refuse hand shaking? Start with an open ended question. OCD / trigger / …  Obsessions: .Screen for other types of anxiety disorder.Screen for mood disorders .Screen for organic causes . home.Do you feel that these obsessions are not real? .Screen for suicide. alcohol. drugs.Do you want to get rid of them? . work. work. homicide.What do you do to overcome the stress created by these ideas? .doc Page 254 of 255 . support   OSCE-guide-III.How many times do you wash your hands? How long do you take in a shower? . self care  Past medical history / medication & allergies Family history Social Hx: smoking. Psychiatry NOTES OSCE-guide-III.doc Page 255 of 255 .
Copyright © 2024 DOKUMEN.SITE Inc.